Mega Part 1

¡Supera tus tareas y exámenes ahora con Quizwiz!

b. contains less than or equal to 3 g of fat per serving. (CH 11 Krause, pg 186) Nutrient content terms such as low-fat must now meet government definitions that apply to all foods. To qualify as low-fat, it must be 3 g or less per serving. When a product contains 25% less fat than a reference product, it can be labeled as "reduced fat." -

"Low-fat" on a food label means that food a. contains 25% less fat than a reference product. b. contains less than or equal to 3 g of fat per serving. c. contains one-third fewer fat calories than a reference product. d. is naturally low in fat.

d. Thiamin (CH 29 Krause

, p. 566) Wernicke encephalopathy is a common side effect of alcoholism and involves damage to the central nervous system caused by thiamin deficiency. The classic triad of disturbances includes encephalopathy, nystagmus (constant, involuntary movement of the eyeball), and ataxia (impaired muscular movement or gait). - - =A deficiency of which nutrient is associated with Wernicke encephalopathy? a. Copper b. Essential fatty acid c. Tryptophan d. Thiamin

d. to prevent antibiotic resistance. (CH 9 Krause) Antibiotic-resistant strains of pathogenic bacteria is a growing problem. Antibiotics are in widespread use in the meat and dairy industry. Organic meats and dairy products are raised without the use of antibiotics. The American Medical Association has gone on record to oppose the nontherapeutic use of antibiotics in agriculture. -

. One of the strongest arguments for the use of organic food is a. that the cost of organic food is decreasing. b. organic food is not more susceptible to contamination. c. to prevent sensitivity to pesticide contamination. d. to prevent antibiotic resistance.

a system of cost efficiency. (CH 10 Kruase, pg 169) Utilization management strives for cost efficiency by eliminating or reducing unnecessary tests, procedures, and services. HIPAA is a system for protecting personal health information. A critical pathway identifies essential elements that occur in a patient's care. -

. Utilization management is a. a system to protect personal health information. b. a system of cost efficiency. c. a critical pathway. d. a medical staffing system.

Escherichia coli (CH 9 Krause, pg 149) E. coli contamination was connected to undercooked ground beef in one fast-food restaurant chain in the 1990s. A common source for this pathogen is water contaminated with human waste. C. jejuni can cause bloody diarrhea but comes from consuming raw milk and undercooked chicken and shellfish. S. aureus promotes nausea and vomiting and affects foods that may be mixed by hand, such as meat and pasta salads. Y. enterocolitica can cause fever and bloody diarrhea in children but more often comes from contaminated and undercooked pork products. -

. Which pathogen causes hemorrhagic colitis (painful, bloody diarrhea), which can result from eating undercooked ground beef and meats? a. Escherichia coli b. Campylobacter jejuni c. Staphylococcus aureus d. Yersinia enterocolitica

b. Osteoporosis (CH 21 Krause, pg. 390) Conditions such as diabetes, heart disease, hypertension, gallbladder disease, sleep apnea, osteoarthritis, and certain cancers worsen as the degree of obesity increases. Women of slight build and those who are underweight have a higher risk of developing osteoporosis. - -

1. An increased incidence of which of the following diseases is NOT associated with obesity? a. Heart disease b. Osteoporosis c. Gallbladder disease d. Sleep apnea

may indicate deficiency before clinical or anthropometric data does.(Ch 7 Krause pg 99) Laboratory-based nutrition testing, used to estimate nutrient concentration in biologic fluids and tissues, is critical for assessment of both clinical and subclinical nutrient deficiencies. Most of these states can be assessed in the laboratory so that nutritional intervention can occur before a clinical or anthropometric change or a frank deficiency occurs. Single test results may be useful for screening or to confirm an assessment based on changing clinical, anthropometric, and dietary status. The best biologic medium to test depends on the specific nutrient. Laboratory assessment is stringently controlled. It involves comparing control samples with predetermined substance or chemical constituent concentrations with every patient specimen. REF: p. 99 -

1. Laboratory tests for nutrients a. are not useful if you have only a single test result. b. are always best done on whole blood. c. may indicate deficiency before clinical or anthropometric data does. d. are currently not well controlled.

Ensuring that effective service delivery systems are in place (CH 9 Krause, pg 140) In the model of shared responsibility, the local health agency is most responsible for ensuring that services are available. The federal government functions in a supportive role, both in providing funding and in the development and dissemination of public health knowledge. Although community organizations and leaders may be involved, the state public health agency is responsible for assessment and monitoring activities in relation to the achievement of goals and objectives. -

18. Although responsibility for public health is shared across different levels of government, which responsibility is usually handled by the local health agency? a. Providing funding to support the capacity to carry out public health functions b. Ensuring that effective service delivery systems are in place c. Supporting the development and dissemination of public health knowledge d. Assessing the capacity to perform essential public health functions

protein (Ch 7 Krause pg 102) Small amounts of protein, from 2 to 8 mg/dL, are expected in normal urine. Larger amounts are associated with damage to the kidneys. Results for glucose, ketones, and blood in the urine are expected to be negative. -

19. In urinalysis tests, which of the following is expected to appear at some level in normal people? a. Glucose b. Ketones c. Blood d. Protein

Reliance on the patient's memory (ch 4, Krause) Both the food frequency and the 24-hour food recall depend on the patient's memory of past intake. This can be of particular difficulty when trying to evaluate the diet history of an elderly person or a child. To their benefit, both of these methods require little time or materials to administer, and they do not require the patient to write down and maintain his or her own record of intake. REF: p. 60 -

3. What is a limitation of using the 24-hour recall and the food frequency? a. The patient's level of literacy b. The time required to collect data c. The lack of tools for conducting these d. Reliance on the patient's memory

b. Hepatitis B (CH 29 Krause, p. 562) Hepatitis B and C are transmitted via blood, blood products, semen, and saliva. Hepatitis D also may be transported by blood and body fluids; however, it is rare in the United States and depends on hepatitis B virus for survival and propagation in humans. Hepatitis A and E are the infectious forms that are transmitted by the fecal-oral route. - -

3. Which is the most common hepatitis virus that is primarily transmitted by blood and body fluids? a. Hepatitis A b. Hepatitis B c. Hepatitis D d. Hepatitis E

a. Metoclopramide (CH 27 Krause, p. 512) Metoclopramide is a prokinetic agent. It increases the contractility of the stomach and shortens gastric emptying time. - -

A patient with gastroparesis may benefit by which of these drugs? a. Metoclopramide b. Aspirin c. Acarbose d. Simethicone

all of the above. (ch 5, Krause) ANS: D Many legislators and legal experts believe the Americans with Disabilities Act sufficiently protects against discrimination, but as an added measure of protection, the GINA was passed by Congress and went into effect on November 21, 2009. GINA defines genetic testing and genetic information, bans discrimination based on genetic information, and penalizes those who violate the provisions of this law. Consumers and health care professionals can feel comfortable in adopting this new service. REF: p. 66 | p. 81 -

9. The purpose of the Genetic Information Nondiscrimination Act (GINA) is to a. define genetic testing and genetic information. b. ban discrimination based on genetic information. c. ensure the fair and positive use of genetic testing. d. all of the above.

d. Food intolerance (CH 26 Krause, p. 480) Food intolerance is a physiologic reaction that does not include immune responses. Patients with food allergies or hypersensitivities would have a response by the patient's immune system. A food aversion is an individual dislike or avoidance of a food for reasons that may be based more on knowledge and perception. - -

A child develops gastrointestinal symptoms after drinking fruit juice. This is an example of what type of condition? a. Food allergy b. Food aversion c. Food hypersensitivity d. Food intolerance

b. The child is at risk for becoming overweight. (CH 17 Krause, pg 324) A child's BMI between the 85th and 95th percentile is defined as being at risk for becoming overweight. An overweight child is one whose BMI is above the 95th percentile. For a child to be considered obese, the BMI has to be significantly above the 95th percentile. -

A child's BMI is plotted on a growth chart at the 90th percentile. How would this child's BMI be interpreted? a. The child has a normal BMI. b. The child is at risk for becoming overweight. c. The child is overweight. d. The child is obese.

b. Parathyroid hormone (CH 24 Krause, p. 457) Vitamin D deficiency is associated with secondary hyperparathyroidism. This results in excessive parathyroid hormone secretion, which then stimulates bone resorption to maintain serum calcium concentrations. Excessive use of thyroid hormone or corticosteroids as medications can also contribute to bone loss. Glucagon has not been associated with risk of osteopenia. - -

A chronic elevation of which circulating hormone typically leads to an increase in bone resorption and bone loss? a. Thyroid hormone b. Parathyroid hormone c. Corticosteroid hormone d. Glucagon

b. Contemplation (CH 14 Krause, pg 228) During contemplation, the client goes back and forth in considering the advantages and disadvantages of making a behavior change. In precontemplation, the client is not even aware that a problem may exist. During preparation, the client needs help in finding an acceptable change strategy to address the problem. In relapse, the client needs help to restart doing the actions that she had previously been doing to bring about the behavior change. -

A client is aware that a dietary behavior problem exists but is not sure whether to do something about it. What stage of change is this client in? a. Precontemplation b. Contemplation c. Preparation d. Relapse

c. diarrhea. (CH 13 Krause, pg 211) Although diarrhea is the most commonly reported complication associated with enteral feeding, often the enteral nutrition is not the cause. More likely reasons for the diarrhea to occur include bacterial overgrowth, antibiotic use, gastrointestinal motility disorders, and administration of hyperosmolar medications and electrolyte supplements. Adjusting the medications and their administration can correct the diarrhea. The use of prebiotics and probiotics, pectin, bulking agents, and antidiarrheal medications are other ways to treat the diarrhea. -

A common complication of enteral tube feeding is a. hyponatremia. b. refeeding syndrome. c. diarrhea. d. hypoalbuminemia.

d. VO2 max (CH 23 Krause, p. 428) A daily intake of less than one-third of the RDA for thiamin, riboflavin, B6, and vitamin C, even when other vitamins are supplemented, may lead to a significant decrease in VO2 max and the anaerobic threshold in less than 4 weeks - -

A daily intake of less than one-third of the RDA for thiamin, riboflavin, B6, and vitamin C can lead to a significant decrease in a. muscle tone. b. glutathione peroxidase. c. antioxidant levels. d. VO2 max.

b. 200 to 350 g of carbohydrate and is low in fat. (CH 23 Krause, p. 434) Providing 200 to 350 g of carbohydrate (or 4 g/kg) is sufficient to replenish muscle glycogen and provide for additional blood sugar. The meal should be eaten 3 to 4 hours before the event, and it should not provide more than 25% of total calories from fat. This is to ensure adequate emptying of the stomach before the activity. Some protein can be included but should not be emphasized, particularly in relation to animal sources, because these can increase the fat intake during the meal. - -

A desirable pre-event meal 3 to 4 hours before competition is one that provides a. high fiber and vitamin content. b. 200 to 350 g of carbohydrate and is low in fat. c. an equal mix of protein and carbohydrate. d. 200 to 350 g of protein and fat and is low in carbohydrate.

d. functional food. (CH 11 Krause, pg 173) Functional food constituents have a biologic effect and may influence health and susceptibility to disease. Sustainable food and organic food are agricultural terms. A cariogenic food is one that promotes tooth decay. -

A food constituent that is not defined as a nutrient but has a biologic effect that may influence health is a a. sustainable food. b. organic food. c. cariogenic food. d. functional food.

b. eats dairy products and eggs but no other animal products. (CH 11 Krause, pg 187) Vegetarians eliminate meat and poultry, but may eat fish. A vegan eats nothing of animal origin. A lactoovovegetarian eats nonanimal products and eggs and dairy. -

A lactoovovegetarian a. eats only eggs and vegetables. b. eats dairy products and eggs but no other animal products. c. does not eat any food of animal origin. d. is a vegetarian who occasionally eats poultry.

c. Canada's Food Guide recognizes cultural, spiritual, and physical importance of Aboriginal foods. (CH 11 Krause, pg 182) The Canadian Food Guide recognizes the cultural, spiritual, and physical importance of traditional Aboriginal foods as well as the role of nontraditional foods in contemporary diets. MyPyramid does not make the same distinction. -

A major difference between MyPyramid in the United States and Canada's Food Guide to Healthy Eating is a. the Canadian guide recommends daily servings of milk. b. MyPyramid emphasizes eating vegetables. c. Canada's Food Guide recognizes cultural, spiritual, and physical importance of Aboriginal foods. d. only MyPyramid includes a recommendation for physical activity.

Consistency-modified diet (Ch 10 Krause, pg 170) When a patient has poorly fitting dentures, it affects the patient's ability to chew. Difficulties with chewing and swallowing are best accommodated by a consistency-modified diet. The regular house diet, although nutritionally complete, may include foods that the patient will have difficulty chewing and swallowing, and this may be reflected by a decreased or incomplete intake. Liquid diets tend to not be nutritionally complete, and in particular, they lack fiber. A person with poorly fitting dentures would need extra calories only in the instances when either increased energy needs are determined or there exists a concern in regard to the patient's weight. -

A newly admitted patient is observed to have poorly fitting dental plates. Which type of diet would be most appropriate for this patient? a. General or regular house diet b. Liquid diet c. Consistency-modified diet d. Diet increased in energy value

72 (ch 4, Krause) With the availability of complete intake records, 72 hours is usually enough to reflect an individual's average intake. With incomplete records, the NIA may need to be extended beyond 72 hours. In a hospital setting, between the patient's health status and the foods and menus offered, intake will vary from meal to meal and from day to day. By tracking the intake across 3 days, the assessment can account for this variation. REF: p. 60 -

A nutrient intake analysis (NIA) or calorie count should be recorded for _____ hours. a. 48 b. 72 c. 24 d. 36

Cheese, salami, and Chianti wine (Ch 8 Krause, pg 131) Aged cheese and meats and fermented foods and beverages contain tyramine, a pressor agent that can promote a hypertensive crisis when eaten while a patient is receiving MAO inhibitors. In general, fresh fruits and vegetables, meats and poultry, and milk and unfermented cheeses are acceptable. -

A patient is being treated for depression with a monoamine oxidase inhibitor. Which of the following should the patient be instructed to avoid while taking this medication? a. Milk, soy protein, and grapefruit b. Cheese, salami, and Chianti wine c. Tomatoes, onions, and poultry d. Cantaloupe, canned pineapple, and peanuts

c. 408 kcal (CH 13 Krause, pg 218-219) As dextrose monohydrate is a molecule of glucose and water, it has a lower calorie concentration than other carbohydrates (3.4 kcal/g vs. 4 kcal/g). 120 g ⋅ 3.4 kcal/g = 408 kcal. Ten percent lipid emulsions provide 1.1 kcal/ml solution. Twenty percent lipid emulsions provide 2 kcal/g solution. -

A patient is receiving an intravenous solution that provides 120 g of dextrose monohydrate. How many calories does this provide the patient? a. 132 kcal b. 240 kcal c. 408 kcal d. 480 kcal

Maintain consistent vitamin K intake. (Ch 8 Krause, page 137) For warfarin to be effective as an anticoagulant, the dose of the drug must be balanced with the amount of vitamin K consumed. Therefore, the patient needs to maintain a consistent intake of vitamin K. Decreasing the intake of vitamin K runs the risk of the patient being unable to stop bleeding. Vitamin D is not affected by warfarin therapy. -

A patient is receiving warfarin (Coumadin). What MNT education should this patient receive? a. Decrease intake of vitamin K foods. b. Maintain consistent vitamin K intake. c. Increase intake of vitamin D and calcium-rich foods. d. Maintain consistent vitamin D intake.

c. Moderate nutritional risk (CH 26 Krause, p. 502) A shrimp allergy would be considered a moderate nutritional risk as shrimp are widely available in food products yet their elimination from the diet does not limit a person's food choices or access to vital nutrients. In particular, this person should inquire about recipe ingredients when eating out and should also ask about and be careful about food preparation with equipment that may also be used in preparing shrimp. A low nutritional risk is associated with food allergies focused on a single vegetable or fruit because these can be replaced with others to ensure adequate nutrient intake. A high nutritional risk is associated with foods commonly used in the food supply that may provide vital nutrients that are difficult to replace. Milk, egg, and wheat allergies are examples of high nutritional risk food allergies. - -

A patient who has a food allergy to shrimp would be considered at what level of nutritional risk in allergy management? a. No nutritional risk b. Low nutritional risk c. Moderate nutritional risk d. High nutritional risk

ANS: B Addison disease is primary adrenal insufficiency and insufficient levels of steroid hormones are produced despite adequate levels of ACTH. In Cushing syndrome, too much cortisol remains in the bloodstream over a long period of time. Adrenal fatigue is a syndrome caused by the decreased ability of the adrenal glands to respond to stress and is almost always secondary to something else. Euthyroid sick syndrome is hypothyroidism associated with a severe systemic illness. REF:p. 629 (Krause CH 31) - -

A rare primary adrenal insufficiency in which insufficient levels of steroid hormones are produced despite adequate levels of the hormone ACTH is known as a. adrenal fatigue. b. Addison disease. c. Cushing syndrome. d. euthyroid sick syndrome.

b. ingest liquids or pureed foods. (CH 27 Krause, p. 523) Patients with gastroparesis often have preserved emptying of liquids, so shifting the diet to a more pureed and liquefied form is often useful. Fiber and fat slow gastric emptying and can therefore worsen symptoms. Small, frequent meals are often helpful. - -

A recommendation to a patient with gastroparesis who needs to gain weight would be: a. increase the amount of dietary fiber. b. ingest liquids or pureed foods. c. eat more high-fat foods. d. eat meals and avoid snacks.

ANS: C Euthyroid sick syndrome is hypothyroidism associated with protein-calorie malnutrition, surgical trauma, myocardial infarction, chronic renal failure, diabetic ketoacidosis, anorexia nervosa, cirrhosis, thermal injury, or sepsis. After the underlying cause is treated, the condition is usually resolved. REF:p. 622 (Krause CH 31) - -

A severe systemic illness that causes decreased peripheral conversion of T4 to T3, an increased conversion of T3 to the inactive rT3, and decreased binding of thyroid hormones is known as a. Hashimoto thyroiditis. b. Graves disease. c. euthyroid sick syndrome. d. Addison disease.

d. someone older than 110 years. (CH 20 Krause, pg 369) A supercentenarian is a new group of individuals identified to be older than 110 years of age. This population is now big enough to warrant research. - -

A supercentenarian is a. a woman who is currently 65 years of age who is expected to live to be 100 years old. b. someone born today who can expect to live to be 100 years old or older. c. someone who is 100 years old or older with no disease. d. someone older than 110 years.

c. Weight gain should be controlled to 15 to 25 lb. (CH 15 Krause, pg 263) An overweight BMI is defined as 25 to 29.9. Women with BMIs in this range before pregnancy should promote a weight gain of 15 to 25 lb. Overweight and obese women who are attempting to become pregnant should not promote any weight loss during the pregnancy. -

A woman has a BMI indicating overweight before pregnancy. Which of the following guidelines for weight gain during pregnancy is recommended? a. Weight gain is contraindicated during this pregnancy. b. Guidelines for weight gain for overweight women are not currently established. c. Weight gain should be controlled to 15 to 25 lb. d. It is not necessary to control weight gain during pregnancy.

c. hearing loss. (CH 20 Krause, pg 370) Presbycusis is the most common type of hearing loss. It is usually greater in the high-pitched tonal range. Dysphagia is the inability to swallow. Dysgeusia is the loss of sense of taste or an altered sense of taste. - -

Presbycusis is a. inability to swallow. b. loss of sense of taste. c. hearing loss. d. lack of dentition.

b. language and foods. (CH 14 Krause, pg 230) Surface structure includes such things as food and language. Deep structure includes psychologic and social beliefs and the context of the intervention. -

Educating and counseling with cultural competency should acknowledge surface and deep structure. Surface structure includes things like a. the client's appearance. b. language and foods. c. social beliefs. d. health diagnoses.

Alcohol (Ch 1, Krause) pg 6 The stomach is the site of digestion of small amounts of lipid and protein. Large proteins are broken down into peptides. Otherwise, the stomach breaks down food into smaller particles and passes it into the small intestine, where absorption of all nutrients EXCEPT alcohol takes place. Alcohol is absorbed through the stomach. -

Absorption occurs in the stomach for which of the following nutrients? a. Vitamins b. Carbohydrates c. Minerals d. Alcohol

a. Ingested to affect structure or function of the body (CH 12 Krause, pg 194) A dietary supplement will be taken to either affect structure or function of the body or to supplement the diet. Food is consumed for its taste, aroma, or nutritive value. Only drugs are classified as being able to diagnose, cure, mitigate, treat, or prevent disease. Cosmetics are applied to the body for cleansing, beautifying, or altering appearance. -

According to the FDA, which of the following descriptions applies to a dietary supplement? a. Ingested to affect structure or function of the body b. Consumed for its taste, aroma, or nutritive value c. Used to diagnose, cure, mitigate, treat, or prevent disease d. Applied to the body for cleansing, beautifying, or altering appearance

c. 25 to 35 lb (CH 15 Krause, pg 263) Women with BMIs between 18.5 and 24.9 before conception are considered to be of normal weight and are advised to gain between 25 and 35 lb during the course of the pregnancy. Underweight women who start their pregnancies with a BMI under 18.5 are advised to gain 28 to 40 lb. Women with a prepregnancy BMI of 25 to 29.9 are advised to gain 15 to 25 lb. No standard is established for women who are obese. Women giving birth to twins should gain from 40 to 45 lb during pregnancy. -

According to the National Academy of Sciences, women with normal preconception weight should gain how much during pregnancy? a. 10 to 15 lb b. 20 to 25 lb c. 25 to 35 lb d. 35 to 45 lb

d. mind-body therapies. (CH 12 Krause, pg 192) Art and music therapy, along with meditation and prayer, are classified as mind-body therapies by the NCCAM. Manipulative therapies include massage, yoga, and chiropractic medicine. Energy therapies include magnetic therapy and Reiki. Alternative medical systems include traditional Oriental medicine, ayurveda, and homeopathy. -

According to the National Center for Complementary and Alternative Medicine, art and music therapy are classified as a. manipulative therapies. b. energy therapies. c. alternative medical systems. d. mind-body therapies.

c. 25 g (CH 15 Krause, pg 283) During the first half of pregnancy, the protein requirement is the same as that of a nonpregnant woman; however, during the second half, the requirement increases from an average of 46 g for nonpregnant women to 71 g for pregnant women. This reflects a change of 25 g more per day to promote the protein deposition necessary for both the mother and the fetus. -

According to the RDA, how much additional protein above that of a nonpregnant woman should a pregnant woman consume during the second half of her pregnancy? a. 10 g b. 15 g c. 25 g d. 35 g

c. Water (CH 18 Krause, pg 348) Although athletic activity increases the need for almost all nutrients, the one that has the most likelihood of inadequate intake is water. Dehydration that occurs from inadequate replenishment of fluids during physical activity and accompanying heat illness is the second most common noncardiac cause of death among adolescent athletes. Athletes need more carbohydrate to meet energy demands and more protein to prevent muscle loss. Iron intake may be a problem when working with female adolescents. -

Adequate intake of which nutrient is of greatest concern when working with adolescent athletes? a. Carbohydrate b. Protein c. Water d. Iron

a. lowers glycogen stores and promotes the use of fat for fuel. (CH 21 Krause, pg. 392) Aerobic activity benefits weight management by promoting calorie expenditure. As the duration of aerobic activity continues, glycogen stores are used, and then fat stores start being used as the body's energy source. This results in a decrease in body fat. In combination with resistance training, lean body mass is maintained or increased, causing an increase in resting energy expenditure and continued loss of body fat. Aerobic exercise improves the body's sensitivity to insulin. - -

Aerobic exercise is effective in weight management because it a. lowers glycogen stores and promotes the use of fat for fuel. b. decreases lean body mass in proportion to fat. c. increases resistance to insulin. d. decreases sensitivity to insulin.

c. 50% (CH 22 Kruase, p. 414) The rule of thumb is that most purging activities cause only 50% of the calories taken in through a binge to be eliminated. This means that half of the calories taken in during the binge are retained in the body - -

After a binge episode, how much of the energy intake can be eliminated through purging? a. 100% b. 75% c. 50% d. None

a. Is the GI tract functional? (CH 13 Krause, pg 223) The choice for use of enteral versus parenteral nutrition should be made based on whether the GI tract is functioning and capable of supporting the use of enteral nutrition. If the GI tract works, the preference is to meet the patient's nutritional needs through enteral nutrition. Tube type, formula type, and TPN composition are all decisions that need to be made later. -

After assessment of a patient has determined that oral intake is inadequate, what is the next determination to be made? a. Is the GI tract functional? b. What type of tube should be placed? c. What type of formula should be used? d. What amount and composition of TPN are appropriate?

b. An available substrate for bacterial metabolism (CH 25 Kruase, p. 471) Oral bacteria require fermentable carbohydrate as a substrate for the production of the acids that contribute to tooth decay. Because the oral and nasal cavities expose these bacteria to oxygen, the bacteria do not function anaerobically. Adequate saliva production and alkaline pH reduce the likelihood of dental caries. - -

Along with tooth susceptibility, bacterial presence in the plaque, and sufficient exposure to promote acid production, what other factor is necessary for the development of dental caries? a. An anaerobic environment b. An available substrate for bacterial metabolism c. Adequate saliva d. Alkaline pH

ANS: B T3 is primarily produced by conversion of T4 in the liver and kidneys. The pituitary and nervous system are capable of converting T4 to T3, so they are not reliant on T3 produced in the liver or kidney. Ref. p 620 (Krause CH 31) - -

Although some T3 is produced in the thyroid, approximately 80% to 85% is generated outside the thyroid in which organs? a. Nervous system and adrenal glands b. Liver and kidneys c. Pancreas and gastrointestinal tract d. Hypothalamus and pituitary gland

c. Disordered eating not otherwise specified (CH 18 Krause, pg 340) Because of the strict definitions used by the Diagnostic and Statistical Manual-IV (DSM-IV), many adolescents who demonstrate disordered eating may not fit the criteria associated with anorexia nervosa or bulimia nervosa. The definition for "eating disorder not otherwise specified" can accommodate the variations in growth, the presence of menstrual periods, and cognitive development associated with adolescence. Excessive eating disorder is not defined by the DSM-IV. -

Among adolescents, which type of eating disorder is most likely to be diagnosed? a. Anorexia nervosa b. Bulimia nervosa c. Disordered eating not otherwise specified d. Excessive eating disorder

c. Y-linked (Ch 5, Krause) Y-linked disorders are associated with males, but no specific nutrition-related disorders have been associated with Y-linked inheritance. Nutrition-related autosomal recessive disorders include inborn errors of metabolism. An X-linked dominant nutrition-related disorder is fragile X syndrome, a type of mental disorder susceptible to folic acid deficiency. X-linked recessive nutrition-related disorders include nephrogenic diabetes insipidus and adrenoleukodystrophy. REF: p. 75 -

Among which types of inheritance have no nutrition-related disorders been identified? a. Autosomal recessive b. X-linked dominant c. Y-linked d. X-linked recessive

d. Up to 40 lb (CH 18 Krause, pg 348) Before her pregnancy, this girl's BMI would be interpreted as underweight; therefore, she should attempt to gain from 28 to 40 lb, with a goal toward the upper part of the range. A BMI over 29 would require a weight gain of 15 lb. For a BMI between 26.1 and 29, the goal would be the upper end of the 15- to 25-lb range. For a normal BMI of 19.8 to 26, the goal would be the upper end of the 25- to 35-lb range. -

An adolescent female of gynecologic age of 3 years becomes pregnant. Her prepregnancy BMI was 19.0. According to the IOM, how much weight should she try to gain during her pregnancy? a. 15 lb b. Up to 25 lb c. Up to 35 lb d. Up to 40 lb

c. diets low in sodium may reduce the risk of high blood pressure. (CH 11 Krause, pg 186) A health claim is allowed only on appropriate food products that meet specified standards. The government requires that healthy claims be worded in ways that are not misleading. A claim cannot imply that the food product itself helps prevent disease. Grapefruit helping burn calories is misleading as is the statement that eating fish can improve immunity. Health benefits of fish are specific to levels of omega-3 fat contained in the fish. -

An example of an approved health claim is: a. acai berries help prevent cancer. b. grapefruit can assist in weight loss because it helps burn calories. c. diets low in sodium may reduce the risk of high blood pressure. d. eating fish meals twice a week can improve immunocompetence.

inactive ingredient. (filler such as flavorings and dyes) (Ch 8 Krause) An excipient is an inactive, filler ingredient, such as flavorings, dyes, and preservatives, mixed in with the active ingredient in drugs. Acetylation is a process of inactivating amines, hydrazines, and sulfonamides so as to detoxify a drug. A pressor agent is an organic compound that causes vasoconstriction and an increase in blood pressure. REF: pp. 134-135 -

An excipient is a(n) a. acetylation process. b. active ingredient in a drug. c. pressor agent. d. inactive ingredient.

decreased bioavailability caused by change in gastrointestinal pH. (CH 8 Krause, pg 127) Changes in gastrointestinal pH may occur because of the use of antacids, H2-receptor antagonists, and proton pump inhibitors because these drugs inhibit gastric acid secretion or alter the pH of the stomach. An acidic pH is necessary for the absorption of vitamin B12. However, acidic pH is also necessary for the oxidation of various minerals to improve their bioavailability and absorption. Increasing the gastrointestinal pH may decrease the absorption of these minerals. -

Antacids, H2-receptor antagonists, and proton pump inhibitors affect the absorption of calcium, iron, and magnesium through a. decreased bioavailability caused by change in gastrointestinal pH. b. decreased bioavailability caused by inhibition of small intestine enzymes. c. decreased bioavailability caused by damage to the intestinal mucosa. d. increased bioavailability caused by change in gastrointestinal pH.

is not an independent marker for CVD risk. (Ch 7 Krause, pg 109) The new ACC/AHA guidelines deemphasize use of any markers other than LDL cholesterol and HDL cholesterol. Emerging risk markers for atherosclerotic cardiovascular disease (ACVD) that are not recommended in ATP 4 include differentiating subparticles of LDL by size and grouping by pattern, apolipoprotein B (apoB), and apolipoprotein E (apoE) phenotype. The Cholesterol Expert Panel determined that these markers are not independent markers for risk and do not add to prediction equations. -

Apolipoprotein B (apoB) a. is a subparticle of LDL cholesterol. b. is a subparticle of HDL cholesterol. c. should be measured as a risk factor for CVD according to ACC/AHA guidelines. d. is not an independent marker for CVD risk.

c. by the end of the first counseling session. (CH 14 Krause, pg 230-231) Because the first counseling session is used to establish rapport and the counseling relationship, as well as to obtain information to assess the patient's current eating behaviors, assessment of the patient's stage of change cannot occur before or early in the first session. Ideally, assessment of the stage of change occurs by the end of the first counseling session so that plans and goals in preparation for the next session can be shared with the patient. -

Assessment of a patient's stage of change should be completed a. when the patient is referred for counseling. b. at the beginning of the first counseling session. c. by the end of the first counseling session. d. before the second counseling session.

less than 23 (Ch 7 Krause, pg 113) Differences in race, sex, and age must be considered when evaluating the BMI. BMI values tend to increase with age, yet the relationship between BMI and mortality appears to be U-shaped in adults aged 65 and older. The risk of mortality increased in older adults with a BMI of less than 23. -

BMIs of _______ are associated with increased risk of mortality in those 65 and older. a. less than 23 b. less than 18.5 c. greater than 24.9 d. there is no association between BMI and risk of mortality

ANS: A Soybean, an important source of protein in many developing countries, has goitrogenic properties when iodine intake is limited. The isoflavones, genistein and daidzein, inhibit the activity of TPO and can lower thyroid hormone synthesis. Furthermore, soybean interrupts the enterohepatic cycle of thyroid hormone metabolism. Since the addition of iodine to soy-based formulas in the 1960s, there have been no further reports of hypothyroidism developing in soy formula-fed infants. REF:p. 625 (Krause Ch 31) - -

Before 1960, reports surfaced that soy formula-fed infants were developing hypothyroidism. The addition of what supplement to these formulas ameliorated this problem? a. Iodine b. Selenium c. Iron d. Tyrosine

c. inhibiting osteoclasts. (CH 24 Kruase, p. 465) Alendronate sodium, risedronate sodium, and zoledrosonic acid, which are bisphosphonates, are approved for the prevention of osteoporosis, especially for postmenopausal women. The bisphosphonates act as inhibitors on osteoclasts to reduce bone resorption. - -

Bisphosphonates treat osteoporosis by a. stimulating estrogen production. b. increasing calcium absorption. c. inhibiting osteoclasts. d. inhibiting phosphorus absorption.

c. a disorder of bacterial overgrowth with resulting malabsorption (CH 28 Krause, p. 522) Blind loop syndrome is characterized by a bacterial overgrowth resulting from a stasis or obstruction of the GI tract, radiation enteritis, fistula, or surgical repair. Fat and carbohydrate malabsorption result. A fistula is an abnormal passage between two internal organs. Flatulence results from excessive gas formation in the bowel. Short-bowel syndrome, which results from excessive small bowel resection, promotes the development of malabsorption. - -

Blind loop syndrome is a. an abnormal passage between two internal organs. b. the presence of an excessive amount of gas in the GI tract. c. a disorder of bacterial overgrowth with resulting malabsorption. d. a malabsorptive condition resulting from massive bowel resection.

c. matrix (CH 24 Krause, p. 456) The organic matrix or osteoid phase of bone consists mostly of collagen, which provides both the strength and flexibility of the bone. Osteocalcin and osteopontin are two proteins that occur in the osteoid phase. Cartilage is the connective tissue that occurs at the epiphyseal ends of bone. - -

Bone tissue consists of the mineral phase and the organic _____ phase. a. collagen b. cartilage c. matrix d. osteocalcin

ANS: C Administration of 1 mg of Commiphora mukul/100 g body weight increases iodine uptake by the thyroid, increases TPO activity, and decreases lipid peroxidation, suggesting that increased peripheral generation of T3 might be mediated by this plant's antioxidant effects. REF:p. 628 (Krause CH 31) - -

Botanical preparations have been found in animal studies to influence thyroid activity. Commiphora mukul (guggulsterones from guggul extract) has strong thyroid stimulatory action, demonstrated by a. increasing cortisol production. b. decreasing iodine uptake by the thyroid. c. increasing TPO activity. d. decreasing serum T4 concentrations.

Increase in serum osmolality (CH 6 , Krause) A decrease in extracellular fluid volume results in a hemoconcentration of the blood, resulting in an increase in serum osmolality. The renin-angiotensin system is triggered by the decreased extracellular volume, and as a consequence, angiotensin II stimulates the thirst centers. REF: p. 87 -

By what mechanism is thirst stimulated in the hypothalamus? a. Increase in serum osmolality b. Decrease in serum osmolality c. Increase in both serum osmolality and in extracellular volume d. Decrease in serum osmolality and an increase in extracellular volume

passive diffusion (CH 1, Krause) pg 13 Passive diffusion is limited by the number of channels available for nutrients to randomly pass through. Facilitated diffusion requires the presence of carrier proteins, which may be limited by the health and nutritional status of the person. Active transport requires energy, which also may be limited by the person's health and nutritional status. Osmosis occurs in regard to concentration gradient and only involves the movement of water, not vitamins. -

By which transport mechanism are most vitamins absorbed from the small intestine into the blood? a. Passive diffusion b. Active diffusion c. Facilitative diffusion d. Passive osmosis

c. Vitamin B12 status (CH 27 Krause, pp. 515 & 520) Atrophic gastritis results in achlorhydria and decreased intrinsic factor production. Both stomach acid and intrinsic factor are necessary for the adequate absorption of vitamin B12. If upper GI bleeding is noted, in addition, iron status should be assessed. - -

Evaluation of which of the following should be included in the nutritional assessment of patients with atrophic gastritis? a. Essential fatty acid deficiency b. Vitamin B6 status c. Vitamin B12 status d. Iron deficiency anemia

ANS: A Vitamin D is considered a prohormone with antiproliferative, differentiating, and immunosuppressive activities. Vitamin D also appears to work with other nutritional factors to help regulate immune sensitivity and may protect against development of autoantibodies. REF:p. 628 (Krause CH 31) - -

More than 90% of people with autoimmune thyroid disease have a genetic defect affecting their ability to metabolize what? a. Vitamin D b. Flavonoids c. Selenium d. Tyrosine

d. the most effective glycogen replacement occurs after a training session. (CH 23 Kruase, p. 436) The highest rates of muscle glycogen synthesis occur when large amounts of carbohydrate are consumed immediately and up until 5 hours after activity. Normally, when no carbohydrate is consumed, muscle glycogen synthesis occurs at a rate of 5% each hour after activity. This may be more beneficial for athletes participating in multiple day activities. - -

Carbohydrate intake immediately after a training session is important because a. energy stores are depleted. b. carbohydrate is needed to reverse the feeling of fatigue. c. hypoglycemia is a common end result of prolonged exercise. d. the most effective glycogen replacement occurs after a training session.

ANS: D Chronic adrenal stress results in decreased sensitivity to thyroid hormones. REF:p. 629 (Krause CH 31) - -

Characteristics of chronic adrenal stress do NOT include a. increased thyroid-binding protein activity. b. decreased conversion of T4 to active forms of T3. c. weakened immune barriers of the digestive tract, lungs, and brain. d. increased sensitivity to thyroid hormones.

a. induces an alkaline saliva. (CH 25 Krause, p. 470) Cheese stimulates alkaline saliva, which reduces plaque bacteria. Because of this, combining cheese with a fermentable carbohydrate can reduce the cariogenicity of the meal. As long as the oral pH remains above 5.5, enamel demineralization does not occur. - -

Cheese is considered to be anticariogenic because it a. induces an alkaline saliva. b. induces an acidic saliva. c. induces a neutral saliva. d. decreases saliva production.

b. Human milk has anti-infective factors. (CH 16 Krause, pg 305) Human milk provides secretory immunoglobulin A, lactoferrin, and lysozymes, which all contribute to preventing infection in the infant's gastrointestinal tract. The lactoferrin contributes to improved bioavailability of the iron in breastmilk and makes the iron unavailable to gut flora. Commercial formula has higher iron content because of its reduced bioavailability compared with breastmilk. Human milk is higher in fat content than formula, but the arachidonic acid and docosahexaenoic acid present are more involved in the infant's neurologic development. -

Compared with formula-fed infants, why do breast-fed infants experience a reduced incidence of infections? a. Human milk is higher in fat. b. Human milk has anti-infective factors. c. Commercial infant formula is higher in iron. d. Commercial infant formula is higher in fat.

c. It is higher on a per-kilogram basis than that of an adult. (CH 16 Krause, pg 301) Infancy is a period of rapid growth, with the infant doubling in weight around 4 to 6 months. At no other time postpartum does a human gain in weight and size this quickly. To allow for this growth, the protein needs on a weight basis are greater than those at any other point in the life cycle. -

Compared with later stages of the life cycle, which of the following describes the infant's protein requirement? a. It is lower on a per-kilogram basis than that of the older child. b. It is lower on a per-kilogram basis than that of an adult. c. It is higher on a per-kilogram basis than that of an adult. d. It is the same on a per-kilogram basis as that of an adult.

all of the above. (CH 4, Krase) pg 40 Cytochrome P450 enzymes are essential for the production of cholesterol, steroids, prostacyclin, and thromboxane A2. They are involved in the hydroxylation of endogenous and exogenous toxic molecules in the detoxification transport of toxins. -

Cytochrome P450 enzymes are essential for a. production of cholesterol. b. detoxification transport of toxins for elimination. c. production of prostacyclins. d. all of the above.

a. 10 (CH 11 Krause, pg 277) Because nutrient needs are highly individualized depending on age, sexual development, and reproductive status of females, the DRI framework has 10 age groupings, including age-group categories for children, men, and women 51 to 70 years and those older than 70 years of age. It separates three age-group categories each for pregnancy and lactation; the groups are younger than 18 years, 19 to 30 years of age, and 31 to 50 years of age. -

DRIs have _____ age groupings. a. 10 b. 5 c. 8 d. 16

b. reference points based on a 2000-calorie diet. (CH 11 Krause, pg 185) For most nutrients, the nutrition fact label shows the percentage of the DV. DVs are not recommended intakes for individuals; they are reference points. DVs are listed as percentages, not in milligrams or grams. -

Daily Values (DVs) shown on the nutrition facts label are a. recommended intakes for specific nutrients. b. reference points based on a 2000-calorie diet. c. the same as DRIs. d. listed as grams or milligrams.

d. boiling plant components for 30 to 60 minutes. (CH 12 Krause, pg 195) A decoction is the most concentrated type of water-based beverage, involving boiling the plant parts to extract the most active ingredients. A tea is a beverage made by steeping for a few minutes. Glycerite is an extraction that uses glycerol instead of alcohol to extract active components from an herb. Capsules use either an animal-derived gelatin shell or one made from cellulose. -

Decoction is made by a. steeping fresh or dried herbs in hot water for a few minutes. b. extracting active components of an herb in glycerol. c. enclosing herbal material in a hard shell made from plant-derived cellulose. d. boiling plant components for 30 to 60 minutes.

leading to increased serum levels of free fraction of a drug. (Ch 8 Krause pg 123) Albumin is a primary carrier protein of drugs in the blood, and when albumin levels decrease, fewer binding sites for protein-bound drugs leave a larger free fraction of the protein available to leave the blood and affect target tissues. This results in an increased pharmacologic effect of the drug. Biotransformation systems in the liver and other tissues are not dependent upon albumin. REF: p. 123 -

Decreased serum albumin levels, which are seen in many conditions, including malnutrition and liver disease, affect medications by a. leading to decreased serum levels of free fraction of a drug. b. leading to increased serum levels of free fraction of a drug. c. leading to decreased metabolism of the drug by the liver. d. leading to decreased pharmacologic effects.

c. anorexia nervosa. (CH 22 Kruase, p 419) Complaints of gastric discomfort associated with distention are common in anorexia nervosa because of delayed gastric emptying. The complaints generally occur as the calorie prescription is increased and between-meal snacks become necessary. - -

Delayed gastric emptying and gastric distension are common in a. binge eating disorder. b. bulimia nervosa. c. anorexia nervosa. d. eating disorder not otherwise specified.

c. every 5 years. (CH 11 Krause, pg 184) The U.S. Dietary Guidelines were first published in 1980 and are revised every 5 years. The most recent guidelines were released in 2010. -

Dietary Guidelines for Americans are revised a. every 2 years. b. every 10 years. c. every 5 years. d. as needed.

ANS: D A variety of food antigens could induce antibodies that cross-react with the thyroid gland. A food elimination diet using gluten-free grains and possible elimination of casein, the predominant milk protein, might be considered for patients with hypothyroidism of unexplained origin. REF:p. 628 (Krause CH 31) - -

Dietary intervention is a key therapeutic tool in managing patients with thyroid disease. In the absence of nutritional deficiencies, what is the main goal of nutritional support? a. Maintaining vitamin sufficiency b. Decreasing oxidative stress c. Increasing iodine concentrations d. Reducing antithyroidal antibodies

a. The supplement's efficacy, its application to the cited health problem, and its safety (CH 12 Krause, pg 194) To effectively work with patients using supplements, the practitioner needs to maintain access to a variety of current resources identifying the efficacy of various supplements, their application to particular health problems, and safety issues in regard to the supplement. Sometimes adequate information is available from the manufacturer, but this may not be possible. Also, the Cochrane Database Review may not always be the most up-to-date resource in regard to investigating the efficacy of a supplement. Patients using supplements take them for specified reasons. Counseling should address whether the supplement can provide the health benefit being sought. -

During a nutrition assessment, the patient indicates that she uses a particular dietary supplement. At a minimum, what aspects of the supplement should be researched and reviewed before making a recommendation for continued use? a. The supplement's efficacy, its application to the cited health problem, and its safety b. The source or origin of the supplement, its safety, and alternative treatments for the health problem c. The CDR's evaluation of the supplement, the administration instructions, and the potential side effects d. The form of the supplement, its application to the cited health problem, and its mechanism of action

a. glucose and fatty acids. (CH23 Krause, p. 428) With prolonged physical activity, aerobic production of energy is predominant. The primary substrates for this are glucose and fatty acids. Muscle glycogen are used in short-term, anaerobic activity. Ketones become an energy source when a person has no nutritional intake. Amino acids may be lost to promote gluconeogenesis if an athlete has inadequate carbohydrate intake. - -

During extended physical activity, such as endurance events, the major fuels used for cellular energy are a. glucose and fatty acids. b. glycogen and ketones. c. amino acids and fatty acids. d. glucose and ketones.

a. drop in the RMR as much as 15% in 2 weeks. (CH 21 Krause, pg 385) The resting metabolic rate drops by 15% within 2 weeks of starvation to conserve energy as a protection against future lack of energy and food intake. As starvation continues, ketone production provides the primary energy substrate for most tissues throughout the body. However, the brain, nerve tissues, and red blood cells need glucose as their primary energy source. Without food intake, the only way to provide this glucose is through gluconeogenesis that uses protein from lean body tissue. - -

During starvation, the body's adaptive response is a a. drop in the RMR as much as 15% in 2 weeks. b. rise in the RMR as much as 15% in 2 weeks. c. drop in the RMR as much as 25% in 1 month. d. rise in the RMR as much as 25% in 1 month.

d. Positive behavioral reinforcement of weight gain (CH 22 Krause, p 418) The immediate, short-term goal in treatment of anorexia nervosa is to initiate weight gain. From a psychologic management point of view, this entails the positive encouragement of actions taken by the patient to initiate weight gain. Such actions include praise, reassurance, coaching, and encouragement. Long-term goals of psychologic management are more focused on understanding and changing attitudes and psychologic conflicts that promote disordered eating, as well as improving the patient's interpersonal and social functioning. - -

During the acute stage of anorexia nervosa, what is the focus of psychologic management in treatment? a. Understanding and changing dysfunctional attitudes related to eating b. Addressing psychopathology that reinforces eating-disordered behaviors c. Improving interpersonal and social functioning d. Positive behavioral reinforcement of weight gain

c. Emotions and feelings when eating (CH 22 Krause, p. 422) Because of bingeing, purging, and restrictive eating, the patient with bulimia nervosa often has impaired hunger and satiety cues. By recording emotions and feelings whenever she eats, the patient with bulimia nervosa can learn to recognize these cues. Additionally, patients with bulimia nervosa are more receptive to nutrition counseling than patients with anorexia nervosa, and discussion of the recorded feelings can be used in the counseling to help promote behavioral changes. - -

During the treatment of patients with eating disorders, which of the following is an additional feature that should be monitored in the food records of patients with bulimia nervosa as opposed to anorexia nervosa? a. Eating behaviors b. Exercise c. Emotions and feelings when eating d. Use of alternative sweeteners

a. Blood levels decrease because of decreased synthesis. (Ch 7 Krause page 103) because of decreased synthesis resulting from a downregulation of gene expression and translation. In the case of albumin, other aspects of the reduction include increased catabolism and transport to extravascular spaces. This is different from what happens to albumin during starvation because in that case, plasma albumin levels are maintained by a shift from the extravascular space. -

During trauma, what happens to negative acute-phase respondent levels? a. Blood levels decrease because of decreased synthesis. b. Blood levels increase because of transport into the vascular space. c. Blood levels are not altered because of the catabolism of proteins. d. Blood levels are similar to what they would be during simple starvation.

b. Preoperational stage (CH 17 Krause, pg 318) Classifying foods as "good" or "bad" without explanation is a characteristic of preoperational thinking. Sensorimotor cognitive development occurs in the first 2 years of life as an infant progresses from automatic reflexes to interactions with the environment. Concrete operational cognition involves more cause-and-effect thinking. Formal operational cognition involves hypothetical and abstract thinking. -

During which stage of cognitive development can children identify foods that are "good for you" but probably could not give a reason? a. Sensorimotor stage b. Preoperational stage c. Concrete operations stage d. Formal operations stage

Nutrition intervention (CH 10 Krause, pg 165) Patient-centered goals and objectives identify the desired outcomes of nutrition care, and these are established in the nutrition intervention step after the nutrition problems are identified. During nutrition assessment, data are collected and analyzed so as to identify the nutrition problems in the nutrition diagnosis step. Nutrition monitoring and evaluation watches for the progress and achievement toward the goals and objectives. -

During which step of the nutrition care process should patient-centered goals and objectives be identified? a. Nutrition assessment b. Nutrition diagnosis c. Nutrition intervention d. Nutrition monitoring and evaluation

a. Not-ready-to-change session (CH 14 Krause, pg 232) One of the goals of the not-ready-to-change session is to identify and reduce the client's resistance and barriers to change. The unsure-about-change session may include a summary of perceived barriers to change. The ready-to-change session includes identification of barriers to adherence to change. The first counseling session is used to identify the stage of change of the client. -

During which type of counseling session is one of the goals to identify barriers to change? a. Not-ready-to-change session b. Unsure-about-change session c. Ready-to-change session d. First counseling session

c. end products of metabolism of hormones. (CH 19 Krause, pg 360) Pollutants in water and drugs are exogenous toxins. The end products of hormone metabolism are endogenous toxins. Curcuminoids are antioxidants found in some foods such as turmeric and curry. -

Endogenous toxins include a. pollutants in water. b. drugs. c. end products of metabolism of hormones. d. curcuminoids.

c. failure to use the gastrointestinal tract exacerbates the stress response. (CH 29 Krause, p. 582) The optimal route of nutrition in acute pancreatitis has been the subject of much controversy over the years. Failure to use the GI tract in patients with acute pancreatitis may exacerbate the stress response and disease severity, leading to more complications and prolonged hospitalization; thus, enteral nutrition is preferred for nutrition therapy. The cause of pseudocysts is unclear. - -

Enteral nutrition is preferred over parenteral nutrition when treating patients with severe, acute pancreatitis because a. enteral nutrition is more expensive. b. enteral nutrition can prevent pseudocysts from developing. c. failure to use the gastrointestinal tract exacerbates the stress response. d. it is not the preferred route of nutrition.

Prostaglandins (CH 3, Krause) pg 36-37 Essential fatty acids and polyunsaturated fatty acids are converted into prostaglandins as part of the eicosanoid cascade. Interleukin-6 is a marker of inflammation. Adipokines and cytokines are proinflammatory molecules. Interleukin-6 is a marker of inflammation. -

Essential fatty acids and polyunsaturated fatty acids are converted to what? a. Insulin b. Cytokines and adipokines c. Prostaglandins d. Interleukin-6

c. 1.2 to 2 g/kg (CH 23 Krause, p. 436) The usual protein intake in the United States is above the RDA of 0.8 g/kg. If athletes in strength training continue with the usual protein intake of 1.2 to 2 g/kg (12% to 20% of total calories), they should be taking in enough protein to promote muscle hypertrophy. Higher protein intakes can compromise adequate intake of carbohydrate for energy. Also, high-protein intake can promote diuresis, dehydration, and possible calcium imbalance. - -

How much protein should be consumed daily by athletes in strength training? a. 0.8 to 1 g/kg b. 1 to 1.5 g/kg c. 1.2 to 2 g/kg d. 2 to 3 g/kg

a. French size. (CH 13 Krause, pg 213) Feeding tube internal diameter is measured in French size. -

Feeding tube diameter is measured in a. French size. b. millimeters. c. inches. d. centimeters.

b. 3 years (CH 26 Kruase, p. 487) Most food hypersensitivities develop in the first 2 years of life, and most infants outgrow these sensitivities by 3 years of age. - -

Food allergies that are most likely to resolve with age are those that first manifest during the first _____ of life. a. year b. 3 years c. 10 years d. 15 years

d. all of the above. (CH 17 Krause, pg 320) In 2013 in the United States, 11% of households experienced food insecurity. Federal food assistance programs provided services to about 60% of them. Food insecurity is associated with poor outcomes in children. It increases the risk of iron deficiency anemia in children 3 years and under. -

Food insecure families in the United States a. contribute to poor outcomes in children. b. are associated with inadequate iron intake in children 3 years and under. c. are served by SNAP, WIC, and school lunch. d. all of the above.

d. Vitamin K (CH 16 Krause, pg 304) Most states require that infants receive a vitamin K injection soon after birth to prevent hemorrhagic disease of the newborn. This is more common among breast-fed infants than formula-fed infants because the vitamin K content is not adequate during the first week. Fluoride and iron supplementation is recommended after 6 months of age. Vitamin D supplementation is recommended daily but may be accommodated by providing the infant with adequate sun exposure. -

For a breast-fed infant, which nutrient(s) should be the earliest to be supplemented? a. Fluoride and vitamin D b. Iron and magnesium c. Vitamin C d. Vitamin K

d. 1.5 (CH 13 Krause, pg 214) Patients with cardiopulmonary, renal, or hepatic failure may require fluid restriction. Most standard formulas of 1 to 1.2 kcal/ml concentration contain 80% to 85% water. A concentrated formula with 1.5 to 2 kcal/ml is about 70% water. All patients on enteral nutrition support require water flushes to maintain the integrity of the feeding tube and to ensure adequate fluid. -

For a fluid-restricted patient, an appropriate formula to select for use would be _____ kcal/ml. a. 0.8 b. 1 c. 1.2 d. 1.5

every 3 to 5 days (CH 1, Krause) pg 3 Intestinal mucosal cells have a life span of 3 to 5 days before they are sloughed off and recycled. They are fully functional only for the last 2 to 3 days as they migrate to the distal third of the villi. -

How often do the cells lining the intestinal tract recycle? a. Every 2 to 3 days b. Every 3 to 5 days c. Every 5 to 7 days d. Every 10 to 14 days

b. It can increase muscle glycogen stores. (CH 23 Kruase, p. 434) Carbohydrate feedings consumed 3 to 4 hours before physical activity can restore liver glycogen stores and muscle glycogen stores and provide additional blood sugar for energy at the beginning of the activity. Additional intake of carbohydrate during the activity provides sufficient energy for later stages of the exercise and helps delay fatigue. During exercise, insulin efficiency prevents the development of hyperglycemia and hypotension. - -

For athletes participating in events lasting more than 1 hour, what can a pre-event high-carbohydrate intake do? a. It can cause hyperglycemia. b. It can increase muscle glycogen stores. c. It can cause hypotension. d. It can improve strength during an event.

b. For 6 months (CH 16 Krause, pg 307) Both the AAP and the AND recommend breastfeeding as the sole source of infant nutrition until the infant is 6 months old. Afterward, they both recommend that breastfeeding continue with weaning foods supplemented until the age of 1 year. -

For how long do both the American Academy of Pediatricians and the Academy of Nutrition and Dietetics recommend exclusive breastfeeding of infants? a. For 3 months b. For 6 months c. For the first year d. Up until the age of 2 years

d. Magnesium (CH 15 Krause, pg 262) Magnesium lactate or citrate supplementation is suggested to relieve potential magnesium deficiencies that may occur with pregnancy and lactation. Leg cramps are one sign of this deficiency. Calcium has also been investigated; however, supplementation to treat leg cramps has not been demonstrated. Manganese and potassium have not been investigated in this regard. -

For managing leg cramps in pregnancy, which of the following minerals has the most scientific support for its use? a. Manganese b. Potassium c. Calcium d. Magnesium

2700 (CH 6, Krause) A daily allowance of water from all sources, including beverages and foods, is about 2700 ml/day for women and 3700 ml/day for men. The general recommendation for water intake is approximately 35 ml/kg of usual body weight in adults. REF: p. 89 -

For the average woman, _____ ml/day would meet fluid needs. a. 1500 b. 1700 c. 2700 d. 3500

a. 20%, 50%, and 30% (CH 22 Kruase, pg. 419) Distribution of the calories is necessary as many people with anorexia nervosa try to avoid fat. A dietary fat range of 25% to 30% of calories is recommended to ensure overall caloric intake, which needs to increase for the person with anorexia nervosa to gain weight. Protein should provide 15% to 20% of total calories. This should be protein of high biologic value. Patients with anorexia nervosa often ask for vegetarian diets, but these should be discouraged. Carbohydrates should provide from 50% to 55% of total calories. - -

For the diet therapy in individuals with anorexia nervosa, what percentages of protein, carbohydrate, and fat are recommended, respectively? a. 20%, 50%, and 30% b. 10%, 50%, and 40% c. 30%, 40%, and 30% d. 25%, 40%, and 35%

b. 330 kcal more than the amount for nonpregnant women (CH 15 Krause, pg 252) During the first 6 months of lactation, breastfeeding women need 330 kcal more than nonpregnant women to promote adequate milk production. This is equal to the kilocalorie needs of a pregnant woman during her second trimester. -

For the first 6 months of lactation, what is the recommended energy intake? a. 200 kcal less than the amount for pregnant women b. 330 kcal more than the amount for nonpregnant women c. 550 kcal more than the amount for pregnant women d. The same as the amount for pregnant women in the third trimester

d. nose into the duodenum or jejunum. (CH 13 Krause, pg 211) Nasoduodenal or nasojejunal tubes bypass the stomach, thus reducing the likelihood of instigating nausea or vomiting, not relying on the stomach's motility, and because they are post-pyloric, reducing the risk of aspiration as formula would have to go past two sphincters to work its way back to the trachea. Tubes placed through the mouth are not commonly used because they can interfere with a patient's ability to talk. -

For the patient with delayed gastric emptying, nausea and vomiting, or other indications of risk of aspiration, the tube should be placed through the a. mouth into the stomach. b. nose into the stomach. c. mouth into the duodenum or jejunum. d. nose into the duodenum or jejunum.

d. Back pain (CH 12 Krause, pg 194) CAM was most used for treatment of back pain as reported by the Centers for Disease Control and Prevention. Head cold treatment was the second most common use of CAM. CAM treatment of arthritis is the fifth most commonly reported use, and treatment of headaches the eighth most commonly reported use. -

For which medical diagnosis is CAM most frequently used in the United States? a. Headache b. Arthritis c. Head cold d. Back pain

a. uric acid. (CH 21 Krause, pg. 394) Uric acid is produced as a result of protein breakdown for gluconeogenesis. As the ketones being used as a primary energy substrate impair the elimination of the uric acid, the uric acid can precipitate out into the joints, causing gout. - -

Gout may develop during fasting or after very low-calorie diets. This occurs because elevated ketones are impeding the elimination of a. uric acid. b. creatinine. c. sodium. d. potassium.

b. Arrange for follow-up contact. (CH 14 Krause, pg 233) If a patient is not ready to change, the counselor should acknowledge the client's positive accomplishments and offer the client the opportunity to resume counseling when the patient is ready. Setting goals for the client or expressing disappointment may promote feelings of failure and resentment on the part of the client. Attempting to push, persuade, coax, confront, or tell the client what to do takes the client out of the decision making and does not motivate the client to carry through with dictated actions. -

If a counseling session ends with the client not ready to change, which of the following should the counselor do? a. Establish goals for the client. b. Arrange for follow-up contact. c. Persuade the client to perform some change. d. Express disappointment with the client.

ANS: C In Graves disease, the TRH receptor itself is the primary autoantigen and is responsible for the manifestation of hyperthyroidism. The thyroid gland is under continuous stimulation by circulating autoantibodies against the TRH receptor, and pituitary TSH secretion is suppressed because of the increased production of thyroid hormones. These thyroid-stimulating antibodies cause release of thyroid hormone and Tg and stimulate iodine uptake, protein synthesis, and thyroid gland growth. REF:p. 627 (Krause CH 31) - -

Graves disease is an autoimmune disease in which the thyroid is diffusely enlarged (goiter) and overactive, producing an excessive amount of thyroid hormones. It is the most common cause of hyperthyroidism (overactive thyroid) in the United States. What is the primary target of circulating autoantibodies in this disease? a. Insulin receptors b. Thyroid-stimulating hormone (TSH) receptors c. Thyrotropin-releasing hormone (TRH) receptors d. Cortisol receptors

b. USDHHS (CH 20 Krause, pg 377) In the U.S. Department of Health and Human Services, the Agency on Aging administers a network of local agencies to provide both home-delivered and congregate meals for elderly adults. This is the only food assistance program through USDHHS. All other food assistance programs are provided through the U.S. Department of Agriculture. Assisted-living facilities are residential programs that provide access to supportive health care to older residents. The Centers for Medicare and Medicaid Services provide health care services but no food assistance. - -

Home-delivered meals provided for under the Older Americans Act nutrition program are administered by what agency? a. USDA b. USDHHS c. ALF d. CMS

B Based on amount needed per day (Ch 12 Beerman) - -

How are minerals classified? a. Based on solubility b. Based on amount needed per day c. Based on food source d. Based on their chemical structure

d. By including eggs and dairy foods (CH 18 Krause, pg 341) By including eggs and dairy foods, a vegetarian can be assured that the diet includes adequate vitamin B12, calcium, and vitamin D. Diets that do not include these foods tend to be inadequate in these nutrients as well as zinc and iron. Unless the bones are eaten with the fish, which is not common across all cultures, only adding fish will not provide adequate calcium in the diet. -

How can an adolescent plan a vegetarian diet that is in line with the DRIs and Dietary Guidelines for Americans? a. By focusing on a vegan diet b. By ensuring adequate legumes, nuts, and whole grains c. By including fish d. By including eggs and dairy foods

a. Increasing folic acid intake throughout the childbearing years (CH 15 Krause, pg 249) The Medical Research Council Vitamin Study in the early 1990s was stopped early because the results overwhelmingly supported the reduction of risk of neural tube defects with folic acid supplementation. Niacin needs during pregnancy coincide with the increased energy needs during pregnancy. Sufficient protein during pregnancy allows for the growth of tissues in both the mother and the fetus. Vitamin C may be beneficial in reducing the chance of developing preeclampsia. -

How can the risk of neural tube defects occurring in utero be reduced? a. Increasing folic acid intake throughout the childbearing years b. Ensuring adequate niacin intake during the first 6 weeks of pregnancy c. Providing an adequate protein intake throughout the pregnancy d. Increasing vitamin C during the first trimester

b. They decrease the metabolic rate to lower than normal. (CH 22 Krause, pg 412) For patients who experience a loss of lean body tissue, a concurrent decrease in metabolic rate occurs. For patients with bulimia nervosa, this may be unpredictable; however, during times of dietary restraint between binge episodes, the patient with bulimia nervosa may experience a semistarvation state, which will result in a decrease in RMR - -

How do anorexia nervosa and bulimia nervosa commonly affect the patient's metabolic rate? a. They increase the metabolic rate to higher than normal. b. They decrease the metabolic rate to lower than normal. c. They have no effect on metabolic rate, maintaining it at the normal level. d. They promote an extreme increase in metabolic rate.

d. Intake of almost all nutrients is needed at higher levels during lactation. (CH 15 Krause, pg 283) Although nutrient needs are increased during both pregnancy and lactation above those of women before conception, lactation needs are greater when women must produce breastmilk, the sole source of food for the infant. Women expend 85 kcal for every 100 ml of milk they produce, and the nutritional profile of the milk reflects their nutritional status. Therefore, for the assured health of the infant, breastfeeding mothers have to continue to maintain and replete their nutritional stores. -

How do the dietary recommendations for breastfeeding mothers differ from those for pregnant women? a. Intake of all nutrients is the same as preconception intake for lactating women. b. The intake level during lactation should be severely restricted to promote weight loss. c. Fluids are forced for pregnant women and limited during lactation. d. Intake of almost all nutrients is needed at higher levels during lactation.

It increases the metabolic rate by 7% per degree Fahrenheit above normal. (CH 2, Krause) pg 18 Fever causes an increase in body temperature. For every degree Fahrenheit above the normal 98.6° F, the BMR increases by 7%. -

How does an elevation in body temperature with fever affect the metabolic rate? a. It does not change the metabolic rate. b. It increases the metabolic rate by 7% per degree Fahrenheit above normal. c. It increases the metabolic rate by 14% per degree Fahrenheit above normal. d. It decreases the metabolic rate by 7% per degree Fahrenheit above normal.

Decreases significantly with age and is higher in athletes than nonathletes (CH 6, Krause) At birth, an infant's body weight is about 75% to 85% water compared with a lean adult, who is 60% to 70% by body weight. As muscle mass decreases with age, total body water also decreases. Also, compared with the lean adult, an obese adult may be 45% to 55% water by body weight. This is because the weight is displaced by adipose tissue, which contains very little water. REF: p. 85 ANS: A A decrease in extracellular fluid volume results in a hemoconcentration of the blood, resulting in an increase in serum osmolality. The renin-angiotensin system is triggered by the decreased extracellular volume, and as a consequence, angiotensin II stimulates the thirst centers. -

How does body water, as a percentage of body weight, change based on stage of the life cycle and lifestyle? a. Decreases significantly with age and is higher in athletes than nonathletes b. Decreases significantly with age and is lower in athletes than nonathletes c. Increases significantly with age and is higher in athletes than nonathletes d. Increases significantly with age and is lower in athletes than nonathletes

c. As intensity increases, carbohydrate is the larger fraction of the energy source. (CH 23 Krause, p. 432) High-intensity activities rely on anaerobic metabolism of carbohydrates. Moderate-intensity activities rely mostly on aerobic metabolism of carbohydrates and some utilization of fatty acids. Low-intensity activities also rely on aerobic metabolism of carbohydrate and fatty acids; however, as the duration of activity increases, more fatty acid utilization occurs. This is part of the reason that the Dietary Guidelines recommend 60 to 90 minutes of activity for people wishing to lose weight. - -

How does intensity of the exercise affect the use of fuel sources? a. As intensity decreases, carbohydrate is the larger fraction of the energy source. b. As intensity increases, fatty acids are the larger fraction of the energy source. c. As intensity increases, carbohydrate is the larger fraction of the energy source. d. As intensity decreases, fatty acids are the larger fraction of the energy source.

c. At least 2 days/week for 6 months (CH 22 Krause, pg. 412) The DSM-5 definition of binge eating disorder specifies that binge episodes must occur for 2 days/week for 6 months. In regard to bulimia nervosa, the definition specifies binge episodes and inappropriate compensatory behaviors at least twice a week for 3 months. - -

How frequently should binge episodes occur for a binge eating disorder to be diagnosed? a. At least 5 days/week for 3 months b. At least 2 days/week for 3 months c. At least 2 days/week for 6 months d. At least 6 days/week for 2 months

d. Average of three blood pressure readings that exceed the 95th percentile (CH 18 Krause, pg 346) Blood pressure levels are different between adults and adolescents; therefore, a different set of standards is used for adolescents. Percentile charts are available that report blood pressures based on age, gender, and height. These standards should be used to determine hypertension in an adolescent as opposed to applying adult standards. -

How is hypertension diagnosed in adolescents? a. Blood pressure of 130/85 mm Hg b. Blood pressure of 140/90 mm Hg c. Blood pressure of 160/100 mm Hg d. Average of three blood pressure readings that exceed the 95th percentile

b. It is increased in cholestasis. (CH 29 Krause, p. 561) Serum alkaline phosphatase is an enzyme that is widely distributed in the liver, bone, placenta, intestine, kidneys, and leukocytes. Increased levels suggest cholestasis but can also be increased with bone disorders, pregnancy, normal growth, and some malignancies. Prothrombin time is used to assess bleeding time. - -

How is serum alkaline phosphatase used to diagnose liver disease? a. It is decreased in cholestasis. b. It is increased in cholestasis. c. It indicates prolonged bleeding time. d. It precedes jaundice.

b. By replacing animal protein in the diet (CH 19 Krause, pg 362-363) The isoflavones in soy were originally thought to have a direct effect on the lipoprotein profile associated with serum cholesterol; however, recent evidence has not substantiated any effect on HDL cholesterol or triglycerides. The American Heart Association recommends the use of soy as an alternative to animal protein; therefore, the benefit is seen through a reduction in saturated fat intake. Isoflavones do not have antioxidant properties. Although isoflavones are associated with vasomotor symptoms (hot flashes) in menopausal women, effects on blood pressure have not been noted. -

How is soy beneficial in promoting cardiovascular health? a. By reducing saturated fat intake b. By replacing animal protein in the diet c. By stimulating HDL production d. By promoting vasodilation and decrease in blood pressure

3 to 8 hours (Ch 1, Krause) pg 8 Travel of contents through the small intestine takes 3 to 8 hours. A liquid meal empties from the stomach within 1 to 2 hours of eating. A solid meal takes 2 to 3 hours. Total transport from mouth to anus takes 18 to 72 hours on average. -

How long does it take for small intestine contents to reach the ileocecal valve? a. 18 to 72 hours b. 3 to 8 hours c. 1 to 2 hours d. 2 to 3 hours

b. 30 to 40 kcal/kg/day (CH 22 Kruase, pp. 417-18 Because of the low weight of patients with anorexia nervosa, 30 to 40 kcal/kg/day is sufficient to start weight gain. To continue to promote weight gain, increasing energy intake by 100 to 200 calories every 2 to 3 days is tolerated by patients with anorexia nervosa. To eventually reach healthy weight gain, some patients with anorexia nervosa may need to be advanced to 70 to 100 kcal/kg/day in their diet therapy. - -

How many calories should be provided in the diet of a patient with anorexia nervosa to initiate weight gain? a. 25 to 30 kcal/kg/day b. 30 to 40 kcal/kg/day c. 40 to 50 kcal/kg/day d. 70 to 100 kcal/kg/day

b. 120 kcal (CH 16 Krause, pg 305) Breastmilk and standard infant formula provide 20 kcal/fl oz (6 · 20 = 120 kcal). -

How many kilocalories are provided by 6 fl oz of breastmilk or standard infant formula? a. 60 kcal b. 120 kcal c. 180 kcal d. 300 kcal

a. The equivalent to a half cup of milk (CH 28 Krause, p, 531) Most people with lactase deficiency can consume 6 g of lactose or the equivalent of a half cup of milk without experiencing symptoms. Many can ultimately adapt to consuming one whole cup of milk when it is introduced gradually and increased incrementally over the course of several weeks. - -

How much lactose can most lactase-deficient people consume without major symptoms? a. The equivalent to a half cup of milk b. The equivalent to one cup of milk c. 12 to 15 g of lactose d. Lactase-deficient people cannot consume any lactose without experiencing symptoms

b. Stage 2 (CH 20 Krause, pg 372-373) A Stage 2 pressure ulcer involves a partial-thickness skin loss. Stage 1 pressure ulcers involve intact skin, but the skin may appear red, may feel firm or boggy, and may promote an itching or painful sensation. Stages 3 and 4 pressure ulcers are full-thickness wounds. Stage 3 pressure ulcers involve the subcutaneous tissue, and Stage 4 pressure ulcers go through the fascia and may affect muscle or bone. - -

How would a pressure ulcer be classified if it involves a partial-thickness skin loss involving the epidermis and dermis, presenting as an abrasion or shallow crater? a. Stage 1 b. Stage 2 c. Stage 3 d. Stage 4

Increased ventilation of carbon dioxide (CH 6, Krause) When excessive metabolic production of acids occurs, the body compensates by stimulating increased expiration of carbon dioxide to reduce acid concentrations in the body. Whereas increased kidney excretion of bicarbonate would be the compensation mechanism for dealing with respiratory alkalosis, decreased excretion of bicarbonate would compensate for respiratory acidosis. In metabolic alkalosis, the lungs would compensate by reducing expiration of carbon dioxide. REF: p. 95 -

How would the body compensate for metabolic acidosis? a. Increased kidney excretion of bicarbonate b. Increased ventilation of carbon dioxide c. Decreased kidney excretion of bicarbonate d. Decreased ventilation of carbon dioxide

Phosphorus (Ch 8 Krause pg 130) Loop diuretics also promote the urinary excretion of sodium, magnesium, chloride, and calcium. With long-term use, magnesium and calcium supplementation may need to be prescribed to maintain serum blood levels. Phosphorus is not affected by loop diuretics, but care should be taken in maintaining normal levels of phosphorus because altered levels will affect calcium metabolism from bone. -

Hypertensive patients who are taking loop diuretics for reduction of blood pressure may experience hypokalemia. Which other electrolyte does NOT need to be monitored for a potential secondary nutrient deficiency? a. Magnesium b. Sodium c. Phosphorus d. Calcium

By increasing medium-chain fatty acids in the diet (Ch 1, Krause) pg 7 Medium-chain fatty acids of 8 to 12 carbons can be absorbed directly by mucosal cells without the presence of bile. The long-chain fatty acids require micelle formation for absorption. Short-chain fatty acids result from bacterial fermentation of malabsorbed carbohydrates and fibers. As bile is produced from cholesterol, dietary restriction of cholesterol is negligible in regard to improvements in fat absorption. -

If a patient experiences malabsorption of fat resulting from an impaired ability to produce adequate bile salts for micelle formation, how may fat absorption be improved? a. By increasing short-chain fatty acids in the diet b. By increasing medium-chain fatty acids in the diet c. By increasing long-chain fatty acids in the diet d. By restricting dietary intake of cholesterol

d. A macronutrient distribution individualized based on the patient's metabolic profile (CH 30 Krause, p. 595) Before 1994, the American Diabetes Association (ADA) attempted to define optimal percentages of macronutrients for control of metabolic complications of diabetes. However, since then, the ADA's focus has been on individualizing nutrition care to accommodate the patient's lifestyle, culture, and socioeconomic status. When planning for patient diets, the RD should maintain the acceptable macronutrient distribution ranges (AMDRs) established by the DRIs. - -

If a patient with type 2 DM receives a nutrition prescription for a 2000-kcal diet, which of the following should be used? a. 50% carbohydrate, 20% protein, 30% fat b. 40% carbohydrate, 30% protein, 30% fat c. 20% carbohydrate, 40% protein, 40% fat d. A macronutrient distribution individualized based on the patient's metabolic profile

c. Not used as food because of potential cross-reactivity (CH 26 Kruase,, pp 495-7) Goat's milk should not be used during the first year of life because of a potential cross-reactivity with beta-lactoglobulin in cow's milk. Also, goat's milk is low in folic acid and has a high renal load. Goat's milk needs to be supplemented with iron; folacin; and vitamins A, C, and D. It needs to be diluted to three-quarter strength, and carbohydrate has to be added to reduce the renal solute load. - -

If an infant is allergic to cow's milk, how should goat's milk be used? a. As a food source for the first 6 months of life b. As a food source for the first year of life c. Not used as food because of potential cross-reactivity d. Not used as food because it is too low in fat

b. Cancer (CH 20 Krause, pg 376-377) The addition of medical nutrition therapy to Medicare benefits began in 2002 with coverage for nutrition services in the treatment of diabetes and kidney disease. In 2005, expansion of Medicare benefits included the provision of MNT in preventive services for new Medicare beneficiaries. In 2006, CMS began working with long-term care improvement programs to address certain chronic conditions in an effort to reduce hospitalizations. Cancer has not been identified as one of the conditions covered. - -

In Medicare benefits, for which of the following are MNT services NOT included? a. Preventive services b. Cancer c. Diabetes d. Preend-stage kidney disease

b. bradycardia. (CH 22 Krause, pg 410) Cardiovascular complications may include bradycardia, orthostatic hypotension, cardiac arrhythmias, and pericardial effusion. - -

In conjunction with anorexia nervosa, cardiovascular complications may include a. amenorrhea. b. bradycardia. c. edema. d. ventricular enlargement.

d. Body image (CH 18 Krause, pg 340) During the early adolescence period, adolescents are concerned with their body size, shape, and image. Seeking increased independence and trusting in adults decreases during middle adolescence. Future orientation is characteristic of late adolescence. -

In early adolescence, which of the following is most likely to be a concern to the adolescent? a. Independence b. The future c. Trusting adults d. Body image

c. 75% (CH 13 Krause, pg 223) When a patient is meeting 75% of nutrition needs through either enteral or parenteral nutrition, the parenteral nutrition may be discontinued. This process may take up to 2 to 3 days as the patient is weaned from parenteral nutrition. Parenteral nutrition administration is not stopped at once because this could contribute to fluid and electrolyte imbalances or rebound hypoglycemia. -

In general, a patient's parenteral feeding can be discontinued when enteral nutrition meets _____ of the patient's need. a. 25% b. 50% c. 75% d. 100%

a. Bolus feeding (CH 13 Krause, pg 215-216) Because the provision of the enteral nutrition will generally be by the family or caregivers in the home, the best method for administering enteral nutrition is the easiest, bolus feeding. This method does not require special equipment, such as a hanging pole, or equipment that may require maintenance, such as pumps. The family can be trained to start with half of one can of formula fed four to six times a day and can work their way up to the patient's total needs as tolerated. -

In home care, what is the recommended administration method for enteral nutrition? a. Bolus feeding b. Gravity drip c. Cyclic pump d. Continuous pump

d. Decreased synthesis of albumin by the liver and the presence of edema and ascites (CH 29 Krause, p. 562) Liver disease affects albumin in several ways. Primarily, as the liver produces albumin, liver disease causes a decrease in its production. As liver disease progresses to cirrhosis, alterations in the structure of the liver affect blood flow to the liver. This leads to portal hypertension. A consequence of both the increased blood pressure and the decreased albumin production is a forcing of fluid into the interstitial and third spaces. The reduced fluid in the blood may falsely concentrate the serum albumin level. - -

In liver disease, which factor(s) affect the interpretation of serum albumin values? a. Increased synthesis of albumin by the liver b. Increased synthesis of albumin by the liver and increased nitrogen retention c. Decreased synthesis of albumin by the liver and increased urinary nitrogen excretion d. Decreased synthesis of albumin by the liver and the presence of edema and ascites

c. Inhalant exposure to antigen (CH 26 Krause, pp. 480-2) Children and adults who develop food allergies after 2 years of age are more likely to develop allergies by inhalant exposure as opposed to consumption. During fetal development and infancy, antigen exposure can occur through maternal diet, as has been seen in the development of peanut and tree nut allergies. However, these types of food allergies will normally manifest before 2 years of age. - -

In people who develop food allergies after 2 years of age, what is the most common means of initial food allergen exposure? a. Maternal consumption during fetal development b. Antigen exposure in breastmilk c. Inhalant exposure to antigen d. Ingestion of food antigen

d. Strict control of blood glucose reduces long-term complications of diabetes (CH 30 Krause, p. 593) In the DCCT, subjects with type 1 diabetes mellitus were provided with either intensive treatment involving multiple insulin injections daily or conventional treatment with only one or two insulin injections daily. The subjects who monitored their blood glucose and used intensive insulin therapy experienced a 50% to 75% reduction in the risk progression of retinopathy, nephropathy, and neuropathy. The trial did include prescribed meal plans but did not evaluate the effect of macronutrient content or number of meals on glucose control. - -

In the Diabetes Control and Complications Trial (DCCT), which of the following was demonstrated? a. Strict control of protein intake, particularly animal protein, improves glucose control. b. Minimizing the number of meals and snacks per day decreases hyperglycemic episodes. c. Strict control of carbohydrate intake, particularly simple sugars, improves glucose control. d. Strict control of blood glucose reduces long-term complications of diabetes.

d. High-fiber diet (CH 28 Krause, p.544) The medical nutrition therapy for patients with inflammatory bowel disease is highly variable and individualized depending on acuteness and exacerbation of the disease. Because of problems with flare-ups and diarrhea, low-residue and lactose-free diets may be instituted. Commonly, parenteral nutrition may be used to promote nutritional repletion or to maintain nutritional status when oral and enteral intakes are not tolerated. A high-fiber diet is least likely to be used because the presence of fiber could aggravate the affected regions of the bowel. However, investigation into the use of prebiotic foods and fibers may help control the intestinal flora associated with IBD. - -

In the MNT for inflammatory bowel disease, which of the following is LEAST likely to be used? a. Low-residue diet b. Parenteral nutrition c. Lactose-free or reduced diet d. High-fiber diet

Bicarbonate and carbonic acid (Ch 6, Krause) Bicarbonate and carbonic acid buffer the production of hydrogen ions and carbon dioxide that results from cellular metabolism, and this occurs primarily in the extracellular space. The phosphate buffering system and protein buffering are activities that more commonly occur in the intracellular space. REF: p. 93 | p. 95 -

In the extracellular space, what is the primary buffer system? a. Phosphate b. Bicarbonate and carbonic acid c. Hydrogen d. Protein

micelles (CH 1, Krause) pg 13 Fats must be emulsified into micelles so that they may cross the unstirred water layer that borders the brush-border membranes. These micelles leave monoglycerides and fatty acids at the brush border, where they are reabsorbed and reassembled as triglycerides. The triglycerides are packaged with cholesterol, fat-soluble vitamins, and phospholipids into chylomicrons, which pass into the lymphatic circulation. When these reach the liver, the chylomicron components are repackaged into low-density lipoproteins. -

In what form is dietary fat absorbed from the lumen of the intestine? a. Chylomicron b. Micelle c. Triglyceride d. Lipoprotein

c. High-protein plant or marine foods (CH 26 Kruase, p. 484) The most common foods associated with food allergies are milk, egg, peanut, soy, wheat, fish, and tree nuts. Meats and poultry are not commonly associated with food allergies. - -

In what types of foods are the most common food allergens found? a. High-simple-carbohydrate foods b. High-protein animal foods c. High-protein plant or marine foods d. High-saturated-fat foods

d. Binge eating disorder (CH 22 Krause, pg. 410) In binge eating disorder, a patient will demonstrate binge eating behaviors but no means of compensating for the excessive intake of food. These people tend to be overweight. The person with binge eating disorder feels incapable of controlling eating, which results in feelings of depression, disgust, and guilt. In the other eating disorders, binge eating is commonly followed by purging or some other compensatory mechanism, such as fasting or excessive exercise. - -

In which eating disorder might a person binge often but not try to compensate for this behavior? a. Anorexia nervosa b. Bulimia nervosa c. Eating disorder not otherwise specified d. Binge eating disorder

c. Eating disorder not otherwise specified (CH 22 Krause, pgs. 408-409) Eating disorder not otherwise specified is the category for disorders of eating that do not meet the criteria for other specific eating disorders. Although people with anorexia nervosa or bulimia nervosa may perform binge eating activities, these are generally associated with purging after the food is swallowed. A person with binge eating disorder will have recurrent episodes of binge eating but will not follow them with some inappropriate compensatory behavior. - -

In which eating disorder might a person chew and spit out, but not swallow, large amounts of food? a. Anorexia nervosa b. Bulimia nervosa c. Eating disorder not otherwise specified d. Binge eating disorder

b. Bulimia nervosa (CH 22 Kruase pgs. 408 & 412) People with bulimia nervosa of the nonpurging type engage in compensatory activities such as exercise or fasting to make up for the intake of a binge episode. People with eating disorder not otherwise specified may use inappropriate compensatory activities less frequently than people with bulimia nervosa or may do so after eating small amounts of food. - -

In which eating disorder might a person exercise excessively after an episode of binge eating? a. Anorexia nervosa b. Bulimia nervosa c. Eating disorder not otherwise specified d. Binge eating disorder

a. Anorexia nervosa (CH 22 Krause, pg 410) Although a person with anorexia nervosa may have a low BMI, this person will have a body image distortion, causing the person to feel fat even when wasted in appearance. People with bulimia nervosa tend to be within normal weight standards but are also concerned with body image. People with eating disorder not otherwise specified may vary in their weight and perception of self and body image. People with binge eating disorder tend to be overweight. - -

In which eating disorder might a person have an intense fear of becoming overweight even though she is underweight? a. Anorexia nervosa b. Bulimia nervosa c. Eating disorder not otherwise specified d. Binge eating disorder

nutrition diagnosis. (CH 10 Krause, pg 166) PES statement refers to "problem" or diagnosis, "etiology," and "signs or symptoms." Inadequate energy intake is an example of a nutrition problem or diagnosis. -

Inadequate energy intake" is an example of a. "S" of a PES statement. b. nutrition diagnosis. c. nutrition symptom. d. "E" of a PES statement.

c. Protein (CH 20 Krause, pg 372) Pressure ulcers require protein for wound healing. The amount of protein recommended depends on the stage of the pressure ulcer. Protein deficiency contributes to the development of the pressure ulcer. A multivitamin and mineral supplement is recommended, but there are no specific recommendations for increased vitamin B12 or potassium. -

Increased intake of which nutrient may be required in a patient who has pressure ulcers? a. Water b. Vitamin B12 c. Protein d. Potassium

c. hepatic encephalopathy. (CH 29 Krause, p. 569) A plasma amino acid imbalance exists in end-stage liver disease in which the branched-chain amino acids are decreased and aromatic amino acids are increased. It has been hypothesized that this imbalance is responsible for hepatic encephalopathy because high levels of aromatic amino acids may limit cerebral uptake of branched-chain amino acids. It is known as the false neurotransmitter theory. - -

Increased serum aromatic amino acids and decreased serum branched-chain amino acids is associated with a. sclerosing cholangitis. b. acute pancreatitis. c. hepatic encephalopathy. d. cholecystitis.

a. Provide only breastmilk to the infant for the first 6 months. (CH 26 Krause, p. 504) If they receive breastmilk exclusively for the first 6 months of life and their exposure to known food allergens is limited, infants have a greater probability of completing the first year of life without developing a food allergy. Continued avoidance of common food allergens for the first 2 to 3 years of age will reduce the chances of developing any food allergies. - -

Infants with a genetic predisposition to atopic disease have an increased probability of developing food allergies. Which of the following may help reduce the likelihood of the infant developing food allergies? a. Provide only breastmilk to the infant for the first 6 months. b. Provide only breastmilk and cereal for the first 6 months. c. Provide cereal between breastmilk feedings. d. Provide diluted cereal and formula for the first 6 months.

ANS: B In autoimmune Hashimoto thyroiditis, supplementing with iodine may exacerbate the condition. Because iodine stimulates production of TPO, this in turn increases the levels of TPO antibodies (TPO Abs) dramatically, indicating an autoimmune flare-up. Some people develop symptoms of an overactive thyroid, but others have no symptoms despite tests showing an elevated level of TPO Abs. Therefore, one must be cautious regarding the use of iodine. Furthermore, although iodine deficiency is the most common cause of hypothyroidism for most of the world's population, in the United States and other westernized countries, Hashimoto thyroiditis accounts for the majority of cases. (CH 31 Krause, REF: p. 622) - -

Iodine deficiency is often a culprit in thyroid disorders. However, in which of these thyroid conditions may supplementing with iodine exacerbate the condition? a. Adrenal fatigue b. Hashimoto thyroiditis c. Polycystic ovary syndrome (PCOS) d. Graves disease

a. anorexia nervosa (CH 22 Krause, pg. 410) Lanugo is soft, downy hair growth associated with anorexia nervosa. People with anorexia nervosa also appear cachectic, and their prepubescent body habitus causes them to appear younger. - -

Lanugo is soft, downy hair growth associated with a. anorexia nervosa. b. binge eating disorder. c. bulimia nervosa. d. eating disorder not otherwise specified.

ANS: D Because unhealthy aging is associated with a progressively increasing prevalence of organ-specific and nonorgan-specific autoantibodies, the absence of these antibodies may represent a significantly reduced risk for cardiovascular disease and other chronic age-related disorders. REF:p. 624 (Krause CH 31) - -

Maintaining thyroid hormone function throughout the aging process appears to be an important hallmark of healthy aging. What characteristic is an indicator of thyroid health in centenarians? a. Decreased free T4 and rT3 levels b. An increased libido c. Constant cortisol production d. The absence of circulating thyroid autoantibodies

a. low metabolic state associated with anorexia nervosa. (CH 22 Kruase, p. 411) Low T3 or active triiodothyronine is a characteristic of a reduced state of metabolism in patients with anorexia nervosa. Thyroid hormone production remains normal; however, peripheral deiodization of thyroxine favors the formation of reduced triiodothyronine. The reduced form of triiodothyronine is less metabolically active than the regular form. This alteration in metabolism resolves as weight gain occurs. - -

Low T3 syndrome is a a. low metabolic state associated with anorexia nervosa. b. reduction in thyroxine associated with anorexia nervosa. c. decrease in triiodothyronine associated with bulimia nervosa. d. decrease in reduced triiodothyronine associated with anorexia nervosa.

ANS: C Hashimoto thyroiditis is an autoimmune disorder in which the immune system attacks and destroys the thyroid gland. It is the most common form of hypothyroidism. The TPO Abs test is the most important because TPO is the enzyme responsible for the production of thyroid hormones and the most frequent target of attack in Hashimoto thyroiditis. (Krause CH. 31) - -

Low energy, cold hands and feet, fatigue, hypercholesterolemia, muscle pain, depression, and a positive test result for thyroid peroxidase antibodies could all be indicative of what condition? a. Diabetes mellitus b. Graves disease c. Hashimoto thyroiditis d. Polycystic ovary syndrome (PCOS)

D- not convincing (Beerman, CH 12) - -

Magnesium deficiency has been investigated to determine whether it increases the risk for cardiovascular disease and predisposes people to type 2 diabetes. To date, how convincing is the evidence from clinical trials? a. Strongly convincing b. Moderately convincing c. Marginally convincing d. Not convincing

A-ATP (Beerman CH 12) - -

Magnesium is vital for energy metabolism as it stabilizes: a. ATP. b. DNA. c. NaCl. d. RNA.

c. the frequency of suckling. (CH 15 Krause, pg 285) Although the diet of the mother does affect the milk composition, the frequency of suckling has the biggest effect on milk production. -

Milk production is most affected by a. calories consumed by the mother. b. mother's hydration status. c. the frequency of suckling. d. protein consumed by the mother.

a. USDA and Department of Health and Human Services (DHHS). (CH 11 Krause, pg 174) The MyPyramid offers a method for determining appropriate patterns for daily food choices and was developed by the USDA and DHHS. The Ministry of Health publishes Canada's food guide. -

MyPyramid was developed by a. USDA and Department of Health and Human Services (DHHS). b. the National Cancer Institute. c. the Food and Drug Administration. d. the Ministry of Health.

d. those living below the poverty line. (CH 11 Krause, pg 177) Nutrient intakes are most likely to be low in persons living below the poverty level. Intakes of nutrients reported to be low in the general population are even lower in the poverty group. -

Nutrient intakes are most likely to be low in a. populations living in the South. b. preschool children. c. those older than 65 years. d. those living below the poverty line.

b. may not be accurate because of body composition changes in elderly adults. (CH 20 Krause, pg 375) The MNA includes six questions and the BMI or calf circumference. However, BMI may not yield accurate results because fat mass increases and lean tissue decreases with aging. A measure such as the mid-arm muscle circumference more accurately reflects body composition. - -

Nutrition screening using the BMI as an indicator a. is the best indicator for those 85 years of age and older. b. may not be accurate because of body composition changes in elderly adults. c. is better than using mid-arm muscle circumference. d. is not part of the Mini Nutritional Assessment (MNA).

b. Cancer (ch 5, Krause) PPAR transcription factors function in lipid and lipoprotein metabolism, glucose homeostasis, adipocyte proliferation, and foam cell formation. Omega-3 and omega-6 fatty acids serve as ligands for PPARs, and as a result, potential beneficial effects could result in reduced expression of type 2 diabetes mellitus, atherosclerosis, and obesity. Research connecting nutrigenomics to cancer presently is investigating relationships to detoxification factors. REF: p. 78 -

Nutritional components of food affect genetic expression by acting as ligands for peroxisome proliferator-activated receptors (PPAR). Which disease may NOT be positively affected by this nutrigenomic relationship? a. Type 2 diabetes mellitus b. Cancer c. Obesity d. Atherosclerosis

c. Semi-volatile organic compounds (CH 21 Krause, pg 384) Semi-volatile organic compounds (SVOCs) accumulate in adipose tissues from exposure to toxins, chemicals, and pesticides. When adipose tissue is mobilized during weight loss, SVOCs are released. Obese women should lose weight before becoming pregnant because the impact of SVOCs on the developing fetal brain is not yet known. Ghrelin is a hormone produced primarily by the stomach and acts on the hypothalamus to stimulate hunger and feeding. Insulin acts in the central nervous system and the periphery nervous system to regulate food intake and is involved in the synthesis and storage of fat. Adipose tissue actively secretes a wide range of pro- and anti-inflammatory cytokines, but these have not been shown to be detrimental to pregnancy. - -

Obese women should lose weight before becoming pregnant because of the possible impact of what compound released when adipose tissue is mobilized? a. Ghrelin b. Insulin c. Semi-volatile organic compounds d. Pro- and antiinflammatory cytokines

d. culture is mutable and multiple. (CH 14 Krause, pg 229-230) Knowing about a culture does not eliminate racism, and part of cultural competency is to avoid any stereotyping, including how a specific culture wants to communicate. Culture is mutable and multiple, and any understanding of particular cultural context is always incompletely true, always partial, and always somewhat out of date. -

One of the five tenets of cultural competency is a. you must know a culture's communication style to be an effective counselor. b. cultural competency eliminates racism. c. culture is defined as race, religion, and ethnicity. d. culture is mutable and multiple.

Folate and vitamin D (Ch 8 Krause, pg 129) Phenytoin and phenobarbital increase the metabolism of vitamin D, folic acid, and vitamin K. As a result, these nutrients are commonly supplemented to patients receiving these drugs. -

Patients receiving anticonvulsant medications, such as phenytoin and phenobarbital, may need to be supplemented with which nutrients? a. Ascorbic acid and vitamin A b. Ascorbic acid and vitamin D c. Folate and vitamin D d. Iron and vitamin A

milk (Ch 8 Kraue pg 136) Tetracycline will chelate with calcium from dairy foods and supplements, making both the drug and the calcium unavailable for absorption. Calcium supplements should not be taken during the duration of this antibiotic's prescription. Dairy products should be taken 2 to 6 hours separate from tetracycline administration. -

Patients receiving the antibiotic tetracycline should be taught to avoid taking it with what? a. Fruit juices b. Meat c. Milk d. Sucrose-containing desserts

HCl (Ch 1, Krause) pg 7 Pepsinogen is secreted in the stomach and converted to its active form by the acid environment of the stomach. Enterokinase is secreted by the brush border of the small intestine in response to the presence of chyme. Trypsinogen is secreted by the pancreas and activated by enterokinase. Various peptidases are secreted by the either brush border or the pancreas. -

Pepsinogen is converted to pepsin when it comes in contact with a. enterokinase. b. trypsinogen. c. hydrochloric acid. d. peptidases.

Autosomal recessive (ch 5, Krause) PKU, tyrosinemia, maple syrup urine disease, and other metabolic disorders known as inborn errors of metabolism result from autosomal recessive disorders. Examples of nutrition-related conditions that are autosomal dominant disorders include Albright hereditary osteodystrophy and familial hypercholesterolemia. One nutrition-related X-linked dominant disorder is fragile X syndrome. Diabetes insipidus and adrenoleukodystrophy are examples of X-linked recessive disorders with nutritional implications. REF: p. 75 -

Phenylketonuria is what type of genetic disorder? a. Autosomal dominant b. Autosomal recessive c. X-linked dominant d. X-linked recessive

ANS: A Biochemical and endocrine abnormalities in women with PCOS include elevated levels of androgens (dehydroepiandrosterone, testosterone, and androstenedione), hyperinsulinemia (which results from insulin resistance), impaired glucose tolerance, and hyperlipidemia. Hyperandrogenism is responsible for many of the symptoms of PCOS, such as reproductive and menstrual abnormalities, hirsutism, and acne. REF:p. 626 (Krause CH 31) - -

Polycystic ovary syndrome (PCOS) is characterized by which of the following biochemical and endocrine abnormalities? a. Elevated testosterone, insulin resistance, and impaired glucose tolerance b. Thyrotoxicosis, iodine deficiency, and elevated estrogen c. Impaired glucose tolerance, increased T4 conversion, and hypertension d. Low blood calcium, thyroid receptor hypersensitivity, and elevated cortisol

a. Jewish, Muslim, and Seventh Day Adventist (CH 11 Krause, pg 188) Pork is prohibited in the Jewish, Muslim, and Seventh Day Adventist religions. It is avoided by most in the Buddhist and the Hindu religions. -

Pork is prohibited in which of the following religions? a. Jewish, Muslim, and Seventh Day Adventist b. Catholic, Muslim, and Buddhist c. Mormon, Jewish, and Muslim d. Mormon, Hindu, and Seventh Day Adventist

d. they can create a bond with the caretaker. (CH 13 Krause, pg 214) Generally homemade (real food) tube feedings are chosen because they offer the benefit of whole foods and can create a bond with the caretakers who are often the family. They can be less expensive if there is no reimbursement available. Only commercially prepared products are paid for through insurance and Medicare. They should not be used in immunocompromised patients and must be used with a large bore feeding tube to prevent clogging. -

Possible advantages of using homemade tube feedings include a. they are better reimbursed by insurance. b. they may help an immunocompromised patient. c. they can be used with any French size feeding tube. d. they can create a bond with the caretaker.

that is caused by Clostridium difficile infection. (CH 8 Krause pg 132) The desired effects of medications often are accompanied by effects that are considered undesirable. Side effects are often an extension of the desired effects, such as bacterial overgrowth, as a result of use of an antibiotic. Overgrowth of C. difficile causes pseudomembranous colitis. Suppression of natural oral bacteria may lead to oral yeast overgrowth, or candidiasis. -

Pseudomembranous colitis is a serious side effect of antibiotic therapy a. that results from a food-drug interaction. b. that is caused by Clostridium difficile infection. c. that can be avoided with good nutrition. d. related to hypertension.

all of the above (CH 6 Krause) ANS: D Refeeding syndrome occurs when a starved patient is fed and there is rapid utilization of phosphorus for phosphorylation of glucose. ATP production also requires magnesium. As the body begins making ATP, serum magnesium and phosphorus drop. If not treated this can be a life-threatening condition -

Refeeding syndrome can result in a. low serum phosphorus. b. low serum magnesium. c. death. d. all of the above.

a. affirming. (CH 14 Krause, pg 231) Techniques used for addressing resistance to change include reflecting, double-sided reflection, shifting focus, agreeing with a twist, emphasizing personal choice, and reframing. Affirming is a technique used when counseling clients not ready to change. Affirmation involves acknowledgment that the counselor understands what the client is doing or thinking is normal. -

Resistance to change can be overcome by all the following techniques EXCEPT a. affirming. b. double-sided reflection. c. agreeing with a twist. d. reflecting.

it is an active transporter of sodium and glucose. (CH 1, Krause) pg 12 The SGLT-1 is an active transporter of sodium and glucose and is key to the small intestine's ability to absorb 7 L of fluid a day. In addition to transporting sodium and glucose, SGLT-1 functions as a molecular water pump. For each molecule of glucose absorbed by SGLT-1, 2 molecules of sodium and 210 molecules of water are also absorbed. -

SGLT-1 is key to the small intestine's ability to absorb water because a. it is an active transporter of sodium and glucose. b. it blocks the transport of sodium. c. it stimulates the body's thirst mechanism. d. it changes the pH of the small intestine.

ANS: D Changes in 5'-deiodination occur in a number of situations, such as stress, poor nutrition, illness, selenium deficiency, and drug therapy. Toxic metals such as cadmium, mercury, and lead have been associated with impaired hepatic 5'-deiodination in animal models. Free radicals are also involved in inhibition of 5'-deiodinase activity. REF:p. 622 (CH 31 Krause) - -

Selenium deficiency, inadequate protein, excess carbohydrates, chronic illness, and stress (high cortisol levels) can all impact what thyroid metabolic process? a. Thyroid-stimulating hormone (TSH) production b. Organification of iodide c. Thyroid-binding globulin (TBG) T4 transport d. 5'-Deiodinase conversion of T4 to T3

a. the government, which sets individual standards. (CH 11 Krause, pg 184) Serving sizes of products are set by the government based on reference amounts commonly consumed. -

Serving size on nutrition labels is determined by a. the government, which sets individual standards. b. the manufacturer of the product. c. the types of stores that sell the product. d. the package size of the product.

ANS: B Cyanogenic plant foods (cauliflower, broccoli, cabbage, Brussels sprouts, mustard seed, turnip, radish, bamboo shoot, and cassava) exert antithyroid activity through inhibition of TPO. The hydrolysis of some glucosinolates found in cruciferous vegetables (e.g., progoitrin) may yield goitrin, a compound known to interfere with thyroid hormone synthesis. REF: p. 625 (Krause CH. 31) - -

Some plant foods (cauliflower, broccoli, cabbage) exert antithyroid activity through what mechanism? a. Binding iodine species b. Inhibiting thyroid peroxidase c. Increasing thyroglobulin antibodies d. Stimulating cortisol

d. general education on diet and exercise. (CH 18 Krause, pg 334) Overweight adolescents start out at Stage 1 and need only general information without a lot of restrictions. Stage 2 adds the component of food monitoring. A very structured meal plan and multidisciplinary medical team is indicated in the severely overweight at Stage 4. Vegan diets can be dangerous because of their very restrictive nature. -

Stage 1 of obesity in adolescence should be treated with: a. structured meal plans and a multidisciplinary health team. b. a diet and recording of food intake and activity. c. a vegan diet and exercise. d. general education on diet and exercise.

d. all of the above. (CH 20 Krause, pg 377) The 2010 health care reform legislation changed Medicare to include an annual wellness visit and a personalized prevention assessment. Prevention services include referrals to education and preventive counseling or community-based interventions to address risk factors. It also expands medical nutrition therapy reimbursement for registered dietitians to cover therapy considered reasonable and necessary for the prevention of illness and disability. - -

The 2010 health care reform legislation expands Medicare to include a. an annual wellness visit. b. education and counseling. c. expanded coverage for reimbursement for dietitians. d. all of the above.

24 hours of admission. (CH 10 Krause, pg 163) Although The Joint Commission does not specify how patients should be identified for nutrition risk, all patients need to be screened for nutrition risk within 24 hours of admission, regardless of whether the patient is admitted during the week or on the weekend. Many hospitals accommodate this by including questions regarding nutrition risk in the initial nursing assessment performed on the patient. If the patient meets the hospital's nutrition risk criteria, this is communicated to the clinical nutrition staff so that further nutrition assessment can be performed. -

The Joint Commission requires the identification of nutrition risk of hospital patients to occur within a. 12 hours of admission. b. 24 hours of admission. c. 48 hours of admission. d. 24 hours of admission, except on the weekend.

d. IBS patients (CH 28 Krause, p 541) Studies have shown that patients with IBS may not be able to tolerate a specific type of carbohydrate. Foods that contain highly fermentable carbohydrates in the presence of gut bacteria can exacerbate IBS symptoms. There is emerging research that a diet low in FODMAPs (Fermentable Oligo-, Di-, and Monosaccharides and Polyols) is an effective treatment. - -

The FODMAPs diet is showing success for which group of patients? a. Patients with diverticulosis b. Celiac disease patients c. Crohn disease patients d. IBS patients

d. all of the above. (CH 11 Krause, pg 182) The Healthy Food in Health Care Pledge was started by the Health Care Without Harm organization to encourage health care food service departments to change their practices. It includes eight areas of commitment. It encourages local sourcing of food, using products without harmful chemicals, and preventing food waste whenever possible. Other areas of focus include sustainable foods, patient education, and community engagement. -

The Healthy Food in Health Care Pledge a. encourages vendors to supply foods without harmful chemicals and antibiotics. b. encourages hospital food service to minimize food waste and support the use of food packaging that is ecologically protective. c. promotes locally sourced foods when possible. d. all of the above.

c. Body mass index, weight loss, acute disease effect on intake (ch 4, Krause) The MUST screening tool determines a level of risk for patients based on the body mass index (BMI), recent unintentional weight loss, and an expectation of the disease affecting intake for 5 days or greater. Based on these factors, points are assigned to identify whether a patient is at low, medium, or high risk. The risk level indicates what type of care interventions are used with the patient. The Nutrition Screening Initiative uses BMI, eating habits, and functional status as part of its Level I screen. BMI, blood values for albumin and cholesterol, and identification of clinical signs of nutrient deficiency are included in the NSI's Level II screen. REF: p. 55 -

The Malnutrition Universal Screening Tool (MUST) relies on what factors for the determination of nutritional risk in adults? a. Body mass index, eating habits, functional status b. Body mass index, blood albumin and cholesterol levels, clinical signs c. Body mass index, weight loss, acute disease effect on intake d. Body mass index, medical diagnosis, blood albumin level

d. Eating breakfast (CH 21 Krause, pg. 401) The NWCR consists of more than 5000 individuals who have been successful in long-term weight loss maintenance. It found that there was very little similarity in how individuals lost weight, but there were some common behaviors they all have for keeping weight off: (1) eating a relatively low-fat diet, (2) weighing themselves regularly, (3) engaging in high levels of physical activity, and (4) eating breakfast almost every day. - -

The NWCR identified which behavior that resulted in successful long-term weight loss? a. Use of very low-calorie diets b. Use of pharmaceuticals c. Attending support groups d. Eating breakfast

c. is reform legislation aimed at improving skilled nursing care. (CH 20 Krause, pg 378-379) In 1987, Congress passed OBRA to improve quality of care in skilled nursing facilities by strengthening standards. It does not apply to assisted living facilities or congregate meal sites. Although the legislation requires that nutritional assessments be done periodically, it does not directly pay for dietitian services. - -

The Omnibus Reconciliation Act (OBRA) a. is aimed at assisted living facilities. b. pays for dietitian consults for those older than 65 years of age. c. is reform legislation aimed at improving skilled nursing care. d. is reform legislation aimed at congregate meal sites.

c. 1300 mg; 3000 mg (CH 18 Krause, pg 336) Calcium intake is likely to be low in adolescents. The RDA for all adolescents is 1300 mg with an upper limit of 3000 mg. Calcium intake declines with age in adolescents, especially in females. Research suggests that the high soft drink consumption at this age contributes to decreased calcium intake by displacing milk intake. -

The RDA for calcium for all adolescents is _________ with an upper level intake of _______. a. 800 mg; 1000 mg b. 3000 mg; 5000 mg c. 1300 mg; 3000 mg d. There is no RDA for calcium for adolescents

d. 2.5 (CH 24 Kruase, p 461) Compared with the mean BMD of 20- to 29-year-old adults, the World Health Organization defines osteoporosis as a BMD greater than 2.5 standard deviations below the standard. - -

The WHO's definition of osteoporosis is a BMD greater than _____ standard deviations below the mean for healthy 20- to 29-year-old adults. a. 1 b. 1.5 c. 2 d. 2.5

b. 1 to 1.2 (CH 13 Krause, pg 214) Most standard enteral formulas for use with the general patient population provide 1 to 1.2 kcal/ml. When a fluid restriction is required, formulas that are concentrated to 1.5 to 2 kcal/ml are appropriate. -

The calorie concentration provided by most general-purpose formulas is _____ kcal/ml. a. 0.5 b. 1 to 1.2 c. 1.5 d. 2

folate (Ch 8 Krause pg 129) Methotrexate binds with the enzyme dihydrofolate reductase, inhibiting its activity of converting folate to its active form. This could lead to the development of megaloblastic anemia. To prevent this from happening, leucovorin, or folinic acid, is administered with methotrexate and other folate antagonists -

The cancer chemotherapeutic agent methotrexate acts as an antagonist to a. thiamin. b. vitamin B12. c. folate. d. vitamin B6.

c. 5.5 (CH 25 Kruase, p. 470) When oral pH falls below 5.5, oral bacteria can begin the demineralization process that leads to tooth decay. - -

The caries process begins when the pH drops below what level? a. 7 b. 6.5 c. 5.5 d. 5

a. a detailed clinical history. (CH 26 Krause, p. 491) Diagnosis of adverse food reactions requires identification of the suspected food, proof that the food causes adverse reactions, and verification of an immune- or nonimmune-mediated response. The first step in this process is a detailed clinical history - -

The first diagnostic tool to use in diagnosing a food allergy is a. a detailed clinical history. b. biochemical tests. c. immunologic tests. d. an oral food challenge.

a. Thrombophlebitis (CH 13 Krause, pg 222) Thrombophlebitis is a complication of peripheral PN. Sinusitis, vocal cord paralysis, and pulmonary injury can all be complications of nasoenteric feeding tubes. -

The following is NOT a potential complication of nasoenteric feeding tubes. a. Thrombophlebitis b. Sinusitis c. Vocal cord paralysis d. Pulmonary injury

c. gastrointestinal (CH 13 Krause, pg 209) Enteral nutrition stimulates the gastrointestinal system and its immunity to maintain the integrity of the mucosal lining. This has been substantiated in numerous animal models but not always in animal models. Indirect evidence has suggested that enteral nutrition preserves mucosal integrity during critical illness in humans. However, in regard to possible bacterial translocation and the development of sepsis, these outcomes have been seen in patients with a very short bowel receiving parenteral nutrition. -

The functions of the _____ system are better maintained with enteral feedings than with parenteral feedings. a. cardiovascular b. renal c. gastrointestinal d. pulmonary

d. all of the above. (CH 27 Krause, p. 513) The gastrointestinal effects of smoking include the reduction of lower esophageal and pyloric sphincter pressure, increased reflux, inhibition of pancreatic bicarbonate secretion, accelerated gastric emptying of liquids, and lower duodenal pH. - -

The gastrointestinal effects of smoking include a. reduction of LES pressure. b. lower duodenal pH. c. inhibition of pancreatic bicarbonate secretion. d. all of the above.

constipation. (Ch 8 Krause, pg 132) Narcotic medications affect peristalsis in the gastrointestinal tract, causing constipation to occur. Antineoplastic drugs commonly promote changes in taste. Nonsteroidal antiinflammatory drugs promote gastrointestinal irritation that results in bleeding. A wide variety of medications, including antiinfective agents and antineoplastic drugs, cause anorexia as a side effect. -

The gastrointestinal side effect commonly seen when narcotic drugs such as codeine and morphine are administered is a. a change in taste. b. constipation. c. gastrointestinal bleeding. d. anorexia.

c. at 6 months of age. (CH 21 Krause, pg. 384) During normal growth, the greatest percentage of body fat (approximately 25%) is set at 6 months of age. At age 6 years in lean children, adiposity rebound occurs with an increase in body fat. - -

The greatest percentage of body fat is set a. at birth. b. at 6 years of age. c. at 6 months of age. d. before birth by genetics.

a. Sucrose, glucose, and fructose (CH 25 Krause, p. 470) Sucrose, glucose, and fructose are fermentable by oral bacteria that promote tooth decay. Some nonnutritive sweeteners, such as aspartame and saccharine, are cariostatic. Others, such as xylitol and other sugar alcohols, are anticariogenic. - -

The ingestion of which of the following is most likely to promote bacterial activity that promotes dental caries? a. Sucrose, glucose, and fructose b. Xylitol, mannitol, and lactose c. Aspartame, saccharin, and cyclamate d. Xylitol, saccharin, and galactose

d. heart disease. (CH 19 Krause, pg 357) In the United States, the leading causes of death and debilitation among adults are (1) heart disease, (2) cancer, (3) cerebrovascular accident, (4) chronic lower respiratory disease, and (5) accidents. -

The leading cause of death and debilitation among adults in the United States is a. accidents. b. cancer. c. diabetes. d. heart disease.

c. 800 to 900 (CH 13 Krause, pg 218) Because the cephalic and brachial veins are common sites used for peripheral parenteral nutrition, highly concentrated parenteral solutions with more than 800 to 900 mOsm/kg are not tolerated. The provision of fat is possible because it is isotonic, but protein and carbohydrate may have to be limited. Also, the addition of electrolytes, multivitamins, and trace elements increase osmolality, making it difficult to provide complete nutrition in a limited volume. -

The maximum osmolality of parenteral solution that the cephalic or brachial vein may tolerate for infusion is _____ mOsm/kg. a. 300 to 500 b. 500 to 800 c. 800 to 900 d. 900 to 1200

c. Glucose intolerance, hyperlipidemia, and hypertension (CH 21 Krause, Pg 388) The metabolic syndrome emphasizes central body adiposity that is strongly related to cardiovascular disease, hypertension, and type 2 diabetes mellitus. In the assessment of the metabolic syndrome, a patient demonstrates three of the following five factors: large waist circumference, elevated serum triglycerides, depressed HDL, elevated blood pressure, and high blood pressure. - -

The metabolic syndrome is associated with which of the following groups of disorders? a. Glucose intolerance, gout, and hypertension b. Gout, hypertension, and hyperlipidemia c. Glucose intolerance, hyperlipidemia, and hypertension d. Hypoglycemia, hyperlipidemia, and hypertension

d. Clostridium difficile. (CH 28 Krause, p. 529) Clostridium difficile is the most common cause of antibiotic-induced diarrhea. Escherichia coli has also been implicated. Lactobacillus and Saccharomyces boulardii are used as a supplement to recolonize the colon with beneficial bacteria. - -

The most common cause of antibiotic-induced diarrhea is a. Lactobacillus spp. b. Saccharomyces boulardii. c. Escherichia coli. d. Clostridium difficile.

b. cow's milk allergy. (CH 26 Kruase, p. 487) Allergy to cow's milk protein is the most common food allergy among infants, with a prevalence of 2.5% in the first 3 years of life. During this same period, limiting exposure to peanuts and tree nuts will reduce the chances of the infant developing allergies to these foods. - -

The most common food allergy found among infants is a. cereal allergy. b. cow's milk allergy. c. fruit allergy. d. vegetable protein allergy.

c. natural product use. (CH 12 Krause, pg 193) Use of "natural products" is the most commonly reported CAM practice, particularly among women, people with higher education, and people who had been hospitalized within the past 12 months. Prayer is a commonly used CAM practice among blacks. Non-Hispanic whites use meditation and herbals more frequently than other ethnic groups. -

The most commonly reported CAM practice in the United States is a. prayer. b. meditation. c. natural product use. d. herbal supplement use.

d. general multivitamin-mineral supplement (CH 12 Krause, pg 195) General multivitamin-mineral supplements are used by 36% of women and 31% of men in the United States. Vitamin E and vitamin C supplements are each used by about 12% of the population, and calcium supplements are used by about 10% of the population. Calcium-based antacids are used by almost 25% of the population. -

The most commonly used dietary supplement in the United States is a _____ supplement. a. calcium b. vitamin C c. vitamin E d. general multivitamin-mineral supplement

a. to position the patient at a 45-degree angle for feeding. (CH 13 Krause, pg 217) The role of enteral feeding in the development of aspiration is controversial. Many experts believe that the primary source of aspiration into the airway is saliva and throat contents, not formula. Aspiration can usually be prevented by positioning the patient with the head and shoulders above the chest whenever feeding occurs. -

The most important technique for prevention of aspiration in an enterally fed patient is a. to position the patient at a 45-degree angle for feeding. b. to use a calorically dense enteral formula. c. to use a large bore feeding tube. d. to administer feedings at a slow rate.

a. the counseling relationship. (CH 14 Krause, pg 230) The counseling relationship needs to be developed during the first counseling session. The environment and the way the counselor communicates, both physically and verbally, can help establish rapport and trust on the part of the client. Without this, the client may not feel the necessity to continue with counseling. -

The most important thing to establish in the first counseling session is a. the counseling relationship. b. likes and dislikes in the counseling environment. c. fees for service. d. reimbursement potential by insurance providers.

c. 350; 460 (CH 15 Krause, pg 251) The DRIs for energy needs for pregnancy add 340 to 360 kcal/day during the second trimester and another 112 kcal/day during the third trimester. These increases accommodate not only the growth of both the mother and fetus during the pregnancy but also an increase in the mother's metabolic rate. -

The recommended energy intake in the second and third trimesters is the sum of the energy requirement for a nonpregnant woman and a daily addition of about _____ kcal in the second and _____ kcal in the third trimesters, respectively. a. 100; 160 b. 250; 350 c. 350; 460 d. 400; 550

5 to 10 g protein and 500 to 600 kcal. (CH 10 Krause, pg 170) The clear liquid diet consists of foods such as tea, broth, carbonated beverages, clear juices without pulp, and gelatin. As a result, this diet is very nutritionally incomplete and usually only provides 500 to 600 kcal and 5 to 10 g protein. Ideally, a patient should be transitioned to a more nutritionally complete diet as soon as the patient can tolerate more solid foods. -

The nutritional value of the average clear liquid diet is a. no protein and 500 to 700 kcal. b. 5 to 10 g protein and 500 to 600 kcal. c. 20 to 30 g protein and 100 to 1200 kcal. d. 40 to 50 g protein and 500 to 700 kcal.

b. 0.7 to 1.2 (CH 25 Kruase, p. 472) Standards for fluoridation of a water supply provide between 0.7 and 1.2 ppm. At this level, the incidence of caries development is reduced, and there is low likelihood of teeth staining or mottling occurring. Infants and children should receive fluoride supplementation if the water supply provides 0.6 ppm of fluoride or less. - -

The optimal level of water fluoridation to protect against dental caries is _____ ppm. a. 0.1 to 0.6 b. 0.7 to 1.2 c. 1.2 to 1.4 d. 1.4 to 2.0

d. 250 (CH 13 Krause, pg 218) The grams of dextrose in 1 L or 1000 ml of 5% dextrose are equal to 1000 ml ⋅ 5% = 50 g. For the osmolarity of a dextrose solution, multiply the grams of dextrose per liter by 5 (50 g ⋅ 5 = 250 mOsm/L). -

The osmolarity of a 1-L bottle of 5% dextrose solution is _____ mOsm/L. a. 50 b. 500 c. 200 d. 250

b. parenteral feeding initiated after 7 days NPO. (CH 13 Krause, pg 222) Although all patients who have received no nutrition for an extended period of time are at risk of developing refeeding syndrome, those who have been replenished by parenteral nutrition are at particular risk because of the direct infusion of carbohydrate, electrolytes, and fluids into the blood system. Digestion and absorption mediate the development of refeeding syndrome. With parenteral nutrition, electrolyte and fluid balances should be monitored to prevent potential imbalances and complications. -

The patient most likely to experience refeeding syndrome is a patient who has a. enteral feeding initiated after 7 days NPO. b. parenteral feeding initiated after 7 days NPO. c. enteral feeding after 24 hours NPO. d. parenteral feeding after 24 hours NPO.

c. have higher testing metabolic rate and lean body mass than women. (CH 21 Krause, pg. 392) Men lose weight faster than women of similar size because of their higher proportion of lean body mass and the resulting increase in resting metabolic rate. This results in higher calorie expenditure in men, leading to weight loss occurring more quickly. The amount of fat men and women have is affected by a variety of genetic and lifestyle factors, so men do not necessarily have less fat than women. The use and success of additional therapies beyond diet depend on individual choice and commitment to weight loss. - -

The rate of weight loss for men on energy-deficient diets is more rapid than for women because men a. generally have less fat to lose. b. are more successful at weight reduction programs. c. have higher testing metabolic rate and lean body mass than women. d. are more likely to exercise while trying to lose weight.

b. is 14 g/kcal. (CH 17 Krause, pg 327) Dietary Recommended Intakes (DRIs) for fiber in children is the same as that for adults—14 g/kcal. Surveys indicate that current intake in the United States is much lower than this recommendation. -

The recommendation for fiber in preschool and school age children a. is 5 g/day. b. is 14 g/kcal. c. is 14 g/day. d. is not part of the DRIs.

1 ml/kcal for adults and 1.5 ml/kcal for infants. (Ch 6, Krause) ANS: D Fluid intake of 1 ml/kcal for adults and 1.5 ml/kcal for infants provides about 35 ml/kg in adults and 150 ml/kg in infants. These levels help to maintain fluid balance within humans -

The recommended fluid intake based on caloric intake is a. 2 ml/kcal for adults and 3 ml/kcal for infants. b. 0.5 ml/kcal for adults and infants. c. 2 ml/kcal for adults and 1 ml/kcal for infants. d. 1 ml/kcal for adults and 1.5 ml/kcal for infants.

Carbohydrate (CH 2, Krause) pg 20-21 The RQ compares the carbon dioxide produced with the oxygen consumed when energy substrates are metabolized. The RQ for carbohydrate is 1. The RQs for protein, fat, and a mixed diet are, respectively, 0.82, 0.7, and 0.85. -

The respiratory quotient (RQ) is highest after consumption of a diet that is primarily composed of what? a. Carbohydrate b. Protein c. Fat d. Mixed macronutrients

d. all of the above. (CH 19 Krause, pg 359) The adult years offer opportunities for prevention as well as intervention. It is critical for the dietetic professional to consider how to assess clients and group needs and be able to address the total picture of optimal health. -

The role of nutrition and dietetic professionals in the adult years involves a. addressing the role of nutrition in the leading causes of death and debility. b. incorporating basic education on food access, selection, and preparation. c. setting mutually acceptable, achievable goals for optimal performance and health. d. all of the above.

HCl (CH 1, Krause) pg 5 Parasympathetic innervation that causes release of hydrochloric acid helps prepare the stomach for the potential of receiving food. After food chyme is passed into the small intestine from the stomach, secretin and cholecystokinin are secreted to stimulate pancreatic secretion of water and bicarbonate. They also signal gallbladder contractions and colonic motility, all resulting in reductions in stomach emptying and duodenal motility. Motilin is secreted from the duodenal mucosa during fasting to stimulate gastric emptying and intestinal motility. -

The sight or smell of food produces vagal stimulation of the parietal cells of the gastric mucosa, resulting in the increased production of what? a. Motilin b. Hydrochloric acid c. Cholecystokinin d. Secretin

ANS: B A variety of food antigens could induce antibodies that cross-react with the thyroid gland. Eliminating gluten-containing grains has been suggested for treatment of hypothyroidism of unknown origin. Selenium has been shown to enhance thyroid function; mushrooms, barley, and seafood are good sources of selenium. REF:p. 622 (Krause CH 31) - -

Thyroid health has been shown to be impacted by the elimination of which foods? a. Seafood b. Gluten c. Mushrooms d. Barley

c. food intolerance (CH 26 Kruase, p. 487) Food intolerance does not involve an immunologic reaction. Food intolerances may include toxic, pharmacologic, metabolic, or idiosyncratic reactions. A food allergy involves an immune reaction. Food sensitivity is another term for referring to a food allergy. - -

The term that refers to an adverse reaction to food that is caused by a nonimmunologic mechanism is a. food allergy. b. allergic reaction. c. food intolerance. d. food sensitivity.

d. weight maintenance. (CH 22 Krause, pgs. 420-421) The actions pursued by the person with bulimia nervosa are aimed at weight loss; however, the immediate goals for treatment should focus on interrupting the binge-purge cycle, restoring normal eating behaviors, and stabilizing body weight. Exercise may be used later to help with weight loss, but this is secondary to helping the patient with bulimia nervosa understand and avoid patterns of binge eating and purging. - -

The treatment goal for a patient with bulimia nervosa who is mildly overweight should be a. weight loss to ideal weight. b. daily exercise to help decrease weight. c. a structured exercise routine to help decrease weight. d. weight maintenance.

a. fiber. (CH 11 Krause, pg 185) The RDI for biotin is 0.3 mg, for iron is 18 mg, and for selenium is 70 mcg. There is no RDI for fiber. -

There is an RDI for all of the following nutrients except a. fiber. b. biotin. c. iron. d. selenium.

a. Between the 5th and 85th percentiles (CH 18 Krause, pg 341) When BMI (weight-for-height) is plotted, a BMI that falls below the 5th percentile is interpreted as underweight, and a BMI at or above the 85th percentile is interpreted as at risk for overweight. -

To determine if weight is appropriate for height at a given age for a given gender, where should the data plotted on the NCHS growth chart appear? a. Between the 5th and 85th percentiles b. Between the 10th and 50th percentiles c. Between the 75th and 95th percentiles d. Between the 5th and 25th percentiles

d. All of the above (CH 25 Krause, p. 473) Prolonged bottle feeding, especially at night, of juice, milk, formula, or other sweetened beverages is associated with early childhood caries. - -

To limit the chances of developing early childhood caries, which of the following should never be provided in a bottle as part of prolonged night feeding? a. Formula b. Milk c. Juice d. All of the above

c. Begin giving fluoridated water when the baby's teeth erupt. (CH 16 Krause, pg 302) Because human milk has low fluoride content regardless of the mother's fluoride intake, after 6 months of age, when an infant's teeth begin to appear, the baby should start receiving fluoridated water on a daily basis. Fluoride supplementation is not recommended before 6 months of age, partially to prevent the likelihood of fluorosis. Fluoride provision through food depends on the water supply used in the preparation of the food. Because communities where fluoridation has not been instituted still exist, adequate fluoride provision through foods alone cannot be guaranteed. -

To meet the fluoride needs of a breast-fed infant, what should parents be instructed to do? a. Begin giving fluoride supplements immediately after birth. b. Supplement the mother's diet with fluoride. c. Begin giving fluoridated water when the baby's teeth erupt. d. When the infant starts eating solid foods, the foods will provide enough fluoride.

b. reduce the risk of adverse toxic effects. (CH 11 Krause, pg 174) ULs were established for many nutrients to reduce the risk of adverse or toxic effects from consumption of nutrients in concentrated forms. RDAs are aimed at 97% to 98% of the healthy population's nutrient requirements. Estimated safe and adequate daily dietary intakes were established for nutrients known to be essential for life, but recommended intakes cannot be established because of insufficient data. -

Tolerable upper intakes (UIs) of nutrients were established to a. define nutrient intakes that may promote health. b. reduce the risk of adverse toxic effects. c. define the level of nutrient that will meet the requirements of 97% to 98% of the healthy population. d. provide data on estimated safe and adequate daily dietary intakes.

d. action and maintenance. (CH 14 Krause, pg 228) Change matched to action and maintenance is the focus of traditional nutrition counseling. However, many people with a dietary behavioral problem are in a preaction stage and therefore not ready for change. This may be one of the reasons that traditional nutrition counseling has not always demonstrated long-term success. -

Traditional nutritional counseling focuses on change matched to a. action and relapse. b. empathy. c. precontemplation and preparation. d. action and maintenance.

b. decreases gastric acid production. (CH 27 Krause, p. 512) Cimetidine and ranitidine are two H2-receptor blocking agents commonly used in the treatment of patients with gastrointestinal disorders. In the management of mild cases of esophageal reflux that results in esophagitis, these medications reduce gastric acid production. Sucralfate is a mucoprotective agent used in the treatment of patients with ulcer disease. Lifestyle and dietary management are used to promote LES pressure. Prokinetic agents are used to stimulate gastric emptying. - -

Treating esophagitis with cimetidine, a histamine H2-receptor blocking agent, is effective because it a. provides a viscous protective barrier. b. decreases gastric acid production. c. increases LES pressure. d. promotes gastric emptying.

a. At 6 months if breastfed in a fluoridated community (CH 25 Kruase, p. 472) Because fluoride is low in breastmilk, the infant will require additional fluoride around the time of tooth eruption, around 6 months of age. If an infant is only being breastfed without supplementation with drinking water between breastfeeding episodes, then the child should receive fluoride supplementation from a doctor. - -

Under which conditions should an infant receive fluoride supplementation? a. At 6 months if breastfed in a fluoridated community b. At 6 months if breastfed and given water in a fluoridated community c. At 3 months if breastfed in a fluoridated community d. At 3 months if breastfed and given water in a fluoridated community

b. check for dehydration or fluid loading. (CH 22 Krause, p. 422.) scale, wearing only a gown, and before eating and after voiding. The urine is checked for urine specific gravity to determine its density. Concentrated urine reflects dehydration. A patient with an eating disorder may consume water to simulate weight gain, and a diluted urine specific gravity will reflect this. Other measures of the urine may be used to determine muscle wasting and ketosis. - -

Urine specific gravity should be checked when patients being treated for eating disorders are weighed to a. monitor for muscle wasting. b. check for dehydration or fluid loading. c. check for presence of ketones. d. identify use of medications to suppress appetite or promote purging.

b. Infusions (CH 12 Krause, pg 195) More concentrated than teas, infusions steep fresh or dried herbs in water for about 15 minutes. Extracts involve dissolving the active ingredients in herbs in an organic solvent. A tincture is a type of extract in which the herbs are placed in alcohol to draw out the active components. Tablets are a pill form of taking herbs. -

Which botanical formulations involve water-based extraction and brewing? a. Extracts b. Infusions c. Tablets d. Tinctures

d. They should eat two or three servings of low-mercury fish per week. (CH 15 Krause, pg 271) There are specific recommendations for DHA intake during pregnancy. The main food source is fatty, cold-water fish, but many of these sources have high levels of mercury. It is recommended that pregnant women eat two or three servings per week of low-mercury fish such as sardines. Vegetable sources of omega-3 fat are not as efficient sources as fish. -

What advice should be given to a pregnant woman about eating fish? a. They should eat only vegetable sources of omega-3 fat. b. There is no specific recommendations about fish. c. They should eat two or three servings of fish per week. d. They should eat two or three servings of low-mercury fish per week.

d. Both DHHS and USDA (CH 9 Krause, pg 143) Both the DHHS and the USDA play a role in the completion of NHANES. DHHS is responsible for the sample design and data of the survey, and the USDA is responsible for dietary data collection methodology, maintenance of the database used to code and process data, and data review and processing. -

What agency(ies) is(are) responsible for NHANES? a. DHHS b. FDA c. USDA d. Both DHHS and USDA

Chelation (Ch 1, Krause) pg 15 Chelation refers to the binding of a cation mineral to a ligand, not a whole protein. Cotransporters carry two different minerals at a time, such as the case with sodium and phosphorus. An overlap of mineral transport mechanisms may lead to competitive absorption between minerals in the presence of other minerals, such as the case with iron or zinc supplementation, leading to a decrease in copper absorption. -

What are primarily absorbed by the large intestine? a. Water and fats b. Carbohydrates c. Proteins d. Water and electrolytes

water and electrolytes (Ch 1, Krause) pg 11 Water and electrolytes are usually the only absorbable remnants of dietary intake that reach the large intestine. Fats, carbohydrates, and proteins from the diet are absorbed throughout the small intestine. -

What are primarily absorbed by the large intestine? a. Water and fats b. Carbohydrates c. Proteins d. Water and electrolytes

d. IgE (CH26 Krause, pp. 480-482) Immunoglobulin E antibodies are the ones involved in classic allergic reactions. IgG is the predominant antibody produced during the secondary immune response. IgG has been actively investigated as being involved with food allergies; however, its clinical usefulness has yet to be substantiated. IgM is the predominant antibody produced by new B cells and is secreted during the first immune response. T-cells are not antibodies but are involved with the cell-mediated immune response. - -

What are the most common antibodies involved in food allergies? a. IgG b. IgM c. T-cells d. IgE

c. These are inappropriate for infants during the first year. (CH 16 Krause, pg 306) All forms of cow's milk are not recommended for provision to infants younger than the age of 1 year. Cow's milk may cause gastrointestinal blood loss, may provide an excessive renal solute load, and is an inadequate source of iron and linoleic acid. -

What are the recommended guidelines regarding the use of low-fat and nonfat milk for infants? a. These are appropriate for overweight infants during the first year. b. These are appropriate for any infant during the first year. c. These are inappropriate for infants during the first year. d. These are inappropriate for infants during the first 3 months of life.

a. Hypertension, proteinuria, and edema (CH 15 Krause, pg 268) Pregnancy-induced hypertension is evident when a pregnant woman has a blood pressure greater than 140/90 mm Hg, protein spilling into the urine, and subsequent edema. The edema results from the increased blood pressure and protein loss. Weight loss is not common; however, impairment of uterine blood flow can result in reduced placental size and fetal intrauterine growth restriction. -

What are the signs and symptoms of pregnancy-induced hypertension? a. Hypertension, proteinuria, and edema b. Hypotension, hyperalbuminemia, and excessive urine output c. Abdominal cramping and weight loss d. Weight loss, edema, and hypertension

b. Rapid digestion and absorption of sugars and elevation of insulin levels (CH 27 Krause, p. 522) Alimentary hypoglycemia is a consequence of the dumping syndrome that results from rapid delivery and absorption of carbohydrate into the blood system, producing an exaggerated increase in insulin and then a decrease in blood glucose levels. The gut peptides glucose insulinotropic polypeptide and glucagon-like polypeptide-1 play some role in this. Alimentary hypoglycemia is aggravated by intake of simple carbohydrates. The pancreas is functioning normally as evidenced by the secretion of insulin in response to the carbohydrate load. - -

What causes the hypoglycemia that occurs after meals in patients who have had a gastrectomy? a. Poor dietary intake b. Rapid digestion and absorption of sugars and elevation of insulin levels c. Pancreatic insufficiency d. Elevated levels of secretin and pancreozymin

c. Irritable bowel syndrome (CH 28 Krause, p.546) Symptoms of irritable bowel syndrome include abdominal discomfort and altered intestinal motility, including feelings of bloating, incomplete evacuation, presence of mucus in the stool, straining and increased urgency to defecate, and increased GI distress. Inflammatory bowel disease presents with diarrhea, fever, weight loss, anemia, malnutrition, and food intolerances. Constipation is defined by the frequency and quantity of stool, and it may be experienced by patients with IBS. Diverticulosis is often experienced without any symptomatology. - -

What condition is associated with the feeling of incomplete evacuation of the bowel? a. Inflammatory bowel disease b. Constipation c. Irritable bowel syndrome d. Diverticulosis

a. Somogyi effect (CH 30 Krause, p. 613) The Somogyi effect may be caused by excessive exogenous insulin administration. In response to the hypoglycemia that results, the counterregulatory hormones are secreted to promote gluconeogenesis. This results in an increase in blood glucose levels. Cushing syndrome is a rare condition involving the excessive secretion of cortisol. The dawn phenomenon involves an increase in morning blood glucose because of an increased need for insulin. The hyperglycemic hyperosmolar state is a complication in older adults with diabetes. In this condition, the patient has a very high blood glucose level but no ketones. - -

What condition occurs when rebound hyperglycemia follows an episode of hypoglycemia? a. Somogyi effect b. Cushing syndrome c. Dawn phenomenon d. Hyperglycemic hyperosmolar state

a. Cognitive behavioral therapy (CH 14 Krause, pg 228-229) Cognitive behavioral therapy focuses on changing maladaptive thoughts and beliefs through cognitive restructuring. This technique is particularly effective in working with patients with body image issues. The transtheoretical model is not a counseling activity itself but rather a model for setting up counseling related to the stage a person is in regarding a needed behavioral change. Reflective listening is a technique used with patients who are not ready for change and involves the counselor communicating understanding of the patient's feelings through a statement. Shifting focus is a means used for dealing with patient resistance that involves the counselor changing the focus of a patient from a perceived barrier to change to one more appropriate. -

What counseling activity is based on the idea that thoughts and beliefs that affect behavior can be changed to promote behavior change? a. Cognitive behavioral therapy b. Transtheoretical model c. Reflective listening d. Shifting focus

b. Using the client's previously expressed ideas to show a discrepancy (CH 14 Krause, pg 234) Double-sided reflection is a technique used to overcome resistance behaviors. The intent behind this is to give the client the opportunity to think about and examine what is said so as to open the discussion more, allowing for the opportunity to facilitate change. Reframing involves offering a different perspective to what the client has said, not necessarily comparing current statements with past statements by the client. -

What does double-sided reflection involve doing? a. Monitoring a counseling session from behind a two-way mirror b. Using the client's previously expressed ideas to show a discrepancy c. Asking questions multiple times d. Reframing

Oxygen consumption and carbon dioxide excretion (CH 2, Krause) pg 20 Indirect calorimetry measures gas exchange that results from metabolism. The oxygen consumption and carbon dioxide excretion can be used to estimate a resting metabolic rate. Direct calorimetry measures heat production, either from humans in a controlled environment, or from food, by incinerating the food and measuring the amount of heat released. -

What does indirect calorimetry measure? a. The amount of heat produced by the body at rest b. The energy potential of foods consumed c. Oxygen consumption and carbon dioxide excretion d. The resting metabolic rate

b. Sodium intake should be above 2 to 3 g/day. (CH 15 Krause, pg 262) Pregnancy does not place any additional demands in regard to sodium restriction. Excessive sodium restriction runs the risk of promoting water intoxication, renal and adrenal necrosis, and neonatal hyponatremia. -

What recommendation about sodium should be given for pregnant women? a. Aggressive restriction is warranted. b. Sodium intake should be above 2 to 3 g/day. c. Sodium intake should not exceed 1 g/day. d. Sodium intake should not exceed 2 g/day.

a. Limit fat calories to 30% of total caloric intake. (CH 17 Krause, pg 327) The AAP recommends that children older than 2 years of age adopt a lower-fat diet so that no more than 30% of calories come from fat by the age of 5 years. This recommendation comes from the knowledge that obesity and cardiovascular disease development begins during childhood. -

What does the American Academy of Pediatrics recommend in regard to calories from fat in the diet of a young child? a. Limit fat calories to 30% of total caloric intake. b. Limit fat calories to 20% of total caloric intake. c. Fat calories should only be limited in extreme circumstances. d. Fat calories should be limited for every child older than 1 year of age.

d. Bone resorption both precedes and exceeds bone formation. (CH 24 Krause, p. 458) The uncoupling of bone remodeling explains why after peak bone mass is achieved, throughout the rest of the life cycle, bone mass declines. Normal bone remodeling involves first bone resorption and then bone formation. However, when uncoupling occurs, more resorption than formation occurs. This results in bone loss. - -

What does uncoupling of bone remodeling mean? a. Bone formation precedes bone resorption. b. Bone formation both exceeds and precedes bone resorption. c. Bone resorption precedes bone formation. d. Bone resorption both precedes and exceeds bone formation.

Decreased absorption of bile salts (CH 1, Krause) pg 11 The use of prebiotic carbohydrates favors the growth of friendly bacteria such as lactobacilli and bifidobacteria. These bacteria ferment the prebiotic carbohydrates, promoting increased short-chain fatty acid production. These types of carbohydrates have not been demonstrated to have a bile-sequestering effect. Impairments in absorption of cation minerals tend to be in relation to phytates and oxalates that are present in plant foods. -

What effect may be achieved by eating a diet high in prebiotic carbohydrates? a. Decreased SCFA production in the bowel b. Increased growth of Lactobacillus spp. c. Decreased absorption of bile salts d. Increased absorption of cation minerals

ANS: A Ready.gov is an Internet education tool developed jointly by the USDA and DHS to teach the public how to prepare for national emergencies. PrepNet is a Food Safety and Inspection Service program that coordinates food safety activities to protect the food supply. CFSAN focuses on issues related to seafood HACCP, safety of additives, biotechnology, supplements, and labeling. FEMA provides support services after a disaster occurs but not prevention or emergency preparation training. REF: p. 153 CH 9 Krause - -

What government program educates the public about how to prepare for a national emergency? a. Ready.gov b. Center for Food Safety and Applied Nutrition c. PrepNet (Food Threat Preparedness Network) d. FEMA

d. RMR increases as lean body mass is increased with exercise. (CH 21 Krause, pg. 396) Resistance training increases lean body mass, increasing the RMR and the ability to use more energy. This promotes weight loss through the loss of body fat in place of lean body tissue. - -

What happens to the resting metabolic rate (RMR) as a result of exercise as part of a weight management program? a. RMR increases as adipose tissue is increased with exercise. b. RMR decreases as adipose tissue is decreased with exercise. c. RMR decreases as lean body mass is decreased with exercise. d. RMR increases as lean body mass is increased with exercise.

b. Segmental versus continuous distribution of the disease throughout the GI tract (CH 28 Krause, p. 542) Whereas Crohn disease appears as segments of diseased bowel separated by segments of healthy bowel, in ulcerative colitis, the disease is continuous throughout the bowel. Both diseases occur more frequently between the ages of 15 and 30 years. Diarrhea, food intolerances, and dehydration are experienced in both diseases. The medical management for both diseases involves treatment with corticosteroids and manipulation of the diet during flare-up episodes. - -

What is a characteristic difference between Crohn disease and ulcerative colitis? a. Age at disease onset b. Segmental versus continuous distribution of the disease throughout the GI tract c. Clinical features of diarrhea, food intolerance, and dehydration d. Medical management with steroids and diet

b. 2 Tbsp (CH 17 Krause, pg 322) The general recommendation for serving sizes for children is two tablespoons of food per year of age. This may be adjusted according to the child's appetite. Milk and juice may be served in 1/2-cup volumes or less. Cooked cereals may be 1/4 to 1/2 cup in volume, and dry cereal may be 1/2 to 1 cup in volume. -

What is an appropriate food serving size for a 2-year-old child? a. 2 tsp b. 2 Tbsp c. 1/2 cup d. 1 cup

small intestines, kidneys (Beerman, CH 12) - -

What regulates blood levels of magnesium? a. Bone, hormones b. Small intestine, kidneys c. Large intestine, liver d. Stomach, chylomicrons

standards of care (Ch 10 Krause, pg 168) Standards of care are the basis for providing consistent quality of care to patients within a particular health care institution. They act as the overall guide to the care. A critical pathway more specifically pinpoints when particular activities of care will occur during the patient's time in the health care facility. Evidence-based practice refers to providing care based on the best, most successful practices in health care that have been demonstrated through research, consensus, or other evidence. Discharge planning prepares for taking care of the patient when the patient leaves the health care facility. -

What is another name for the practice guidelines that define the appropriate and consistent care for a patient with a specific diagnosis or medical problem? a. Standards of care b. Critical pathway c. Evidence-based practice d. Discharge planning

b. Loss of sense of taste (CH 20 Krause, pg 370) Some change in the sense of taste is associated with aging; however, a total loss of taste sensation more often is associated with the use of medications, diseases of the kidney or liver, diabetes, hypertension, head injury, neurologic conditions, and zinc or niacin deficiencies. Hyposmia is a diminished sense of smell and can contribute to a reduced taste sensation. Xerostomia is dry mouth. A reduction in hydrochloric acid production in the stomach can result in nutrient malabsorption. - -

What is dysgeusia? a. Diminished sense of smell b. Loss of sense of taste c. Dry mouth from decreased salivation d. Result of hypochlorhydria

b. No routine supplementation is needed except fluoride in nonfluoridated areas. (CH 17 Krause, pg 318) The American Academy of Pediatrics (AAP) supports the use of diet as the means for providing adequate intake of vitamins and minerals to healthy children. Fluoride is the only mineral advocated for supplementation in children. However, some children may be at risk for inadequate nutrition, and for these exceptions, the AAP is supportive of supplementation. These include children from deprived families, children with anorexia or those following fad diets, children with chronic disease, and children following diets to treat obesity. -

What is the American Academy of Pediatrics' recommendation on the use of supplements for children? a. Routine multiple vitamin and mineral supplements should be given. b. No routine supplementation is needed except fluoride in nonfluoridated areas. c. Maximal doses of vitamins and minerals should be given because it is uncertain that children will eat enough of the nutrients in food. d. Supplements should not be restricted because children are growing so rapidly that overdosing is unlikely.

c. Provide moist, soft, nonspicy food. (CH 25 Krause, p. 476) Sjögren syndrome results in salivary gland dysfunction, which could cause xerostomia. Providing moist, soft, nonspicy foods and increasing fluid consumption make it easier for the patient to eat without experiencing pain. Pureed foods are more commonly used in situations in which the patient has either chewing or swallowing difficulties. Oral liquid supplements may be necessary when oral intake is severely impaired, such as when stomatitis or ulceration of the mouth occurs. - -

What is the appropriate dietary management for a patient with Sjögren syndrome? a. Provide foods that are pureed in form. b. Provide oral liquid supplements. c. Provide moist, soft, nonspicy food. d. Begin fluid restriction.

b. 1 Tbsp for each year of age (CH 16 Krause, pg 311) A general guide for serving infants and young children is 1 Tbsp of each food for each year of age. Foods should be served on plates appropriate to the child's size. The child will indicate hunger if the served amount does not provide satiety. Larger portions and forced feeding may lead to either over- or underfeeding of the child. -

What is the appropriate serving size for foods offered to a small child? a. 1 tsp for each year of age b. 1 Tbsp for each year of age c. 1/2 cup of each food served d. 1 cup total food per meal

Indirect calorimetry (CH 2, Krause) pg 20 Indirect calorimetry is commonly used in hospital settings. The piece of equipment is known as a metabolic cart or monitor. Other methods of measuring energy expenditure include doubly labeled water and direct calorimetry; however, these are not practical for clinical practice. Bomb calorimetry measures the energy available from food. -

What is the clinical method for measuring human energy expenditure? a. Bomb calorimetry b. Indirect calorimetry c. Doubly labeled water d. Direct calorimetry

c. Muscle glycogen (CH 23 Krause, p. 428) The breakdown of glycogen stored in the muscle makes glucose available for anaerobic metabolism at the beginning of physical activity. As muscle glycogen stores are used, blood glucose is taken up by the muscle for continued energy utilization. To maintain a supply of glucose for energy, liver glycogen will be broken down for release of glucose. Pyruvate and lactate can be recycled back to the liver for gluconeogenesis. - -

What is the first source of glucose for the exercising muscle? a. Blood glucose b. Liver glycogen c. Muscle glycogen d. Muscle protein

Stimulation of the pancreas to secrete water and bicarbonate (Ch 1, Krause) pg 6 Secretin is the hormone that works in opposition to gastrin. Whereas gastrin stimulates stomach digestion activities, secretin decreases gastric output and promotes pancreatic secretions to neutralize the acidity of chyme. Cholecystokinin is also secreted when chyme enters the duodenum, and it is responsible for stimulating the gallbladder. -

What is the function of secretin? a. Stimulation of gastric secretions and increased motility b. Stimulation of gallbladder contraction and the release of bile c. Stimulation of the pancreas to secrete water and bicarbonate d. Stimulation of the parietal cells to secrete gastrin

a. Gastric ulcers can develop with low acid output, and duodenal ulcers develop with high acid secretion (CH 27 Kruase, p. 518) Because gastric ulcers occur most frequently along the lesser curvature of the stomach, they may develop even when acid secretion is decreased. Duodenal ulcers occur as a result of constant exposure to high acid secretion. H. pylori is involved with the development of both types of ulcers. Because the activity of both proton pump inhibitors and H2-receptor antagonists is to decrease acid secretion, either may be used in the treatment of both gastric and duodenal ulcers. - -

What is the major difference between gastric and duodenal ulcers? a. Gastric ulcers can develop with low acid output, and duodenal ulcers develop with high acid secretion. b. Gastric ulcers develop with high acid secretion, and duodenal ulcers develop with low acid output. c. H. pylori is the primary cause of gastric ulcers. d. Gastric ulcers are treated with proton pump inhibitors, and duodenal ulcers are treated with H2-receptor antagonists.

d. Based on calories per kilogram (CH 17 Krause, pg 316) Although the EER equations for children older than 3 years of age include age, weight, and height as factors, for children between 13 and 35 months of age, the EER equation only includes the weight factor. The growth rate is highly variable throughout childhood, so it should not be used as a base factor for determination of energy needs. Children vary in weight, height, and body size at any age, which is the basis for the development of growth charts. Because of this variation, age should not be used as the sole base factor for determining energy needs. Calorie needs can be determined on a per centimeter of height basis, but this is independent of the child's age. -

What is the most accurate means for determining the energy requirements (EER) of a child? a. Based on growth rate b. Based on age and height c. Based on age d. Based on calories per kilogram

b. Avoid alcohol completely throughout the entire pregnancy. (CH 15 Krause, pg 272) The American College of Obstetricians and Gynecologists and the March of Dimes both recommend no alcohol through the entire pregnancy. Fetal alcohol syndrome results from fetal exposure to alcohol. This could be attributable to alcohol's effects on cell differentiation, dietary deficiencies associated with alcohol use, and alterations in metabolism. -

What is the most appropriate recommendation to make when counseling a newly pregnant patient about alcohol consumption? a. Avoid alcohol for the first trimester; then no more than 1 oz of alcohol per day. b. Avoid alcohol completely throughout the entire pregnancy. c. Limit consumption to 1 oz of alcohol per day. d. Limit consumption to 2 oz of alcohol per day.

b. Weight bearing (CH 24 Krause, p. 460) Weight-bearing activities place stress on the skeleton and stimulate osteoblast activity during the developmental periods of bone accretion. Swimming, regular walking, and isometrics do not place as much stress on the skeleton and are not associated with improvements in BMD. - -

What is the most beneficial type of exercise to include in an exercise program for a patient at risk for osteoporosis? a. Swimming b. Weight bearing c. Nonweight bearing d. Isometric

a. Lactose intolerance (CH 28 Krause, p. 539) Lactose intolerance or maldigestion is the most common carbohydrate intolerance. Some carbohydrate intolerances occur in newborns as rare congenital defects. Lactose tolerance is considered to be a genetic mutation that affects peoples in Northern Europe and some ethnic groups in India, Africa, and Mongolia. However, 90% of the world's population has a low tolerance to milk. - -

What is the most common carbohydrate intolerance that affects people of all ages? a. Lactose intolerance b. Galactose intolerance c. Sucrose intolerance d. Maltose intolerance

b. Succussion (CH 12 Krause, pg 192) Succussion is the shaking of a diluted remedy that leads to the potentization or increased power of a remedy. Moxibustion applies heat to points along the meridia of the chi to help bring it back into balance. Pharmacognosy is a type of botanical medicine. -

What is the name of the process in which a homeopathic remedy is shaken more than 100 times? a. Moxibustion b. Succussion c. Pharmacognosy d. Potentization

National Health and Nutrition Examination Survey (CH 9 Krause pg 142) NHANES was the original survey that includes assessment of health status of Americans. The Nationwide Food Consumption Survey, the Continuing Survey of Food Intake of Individuals, and the Diet and Health Knowledge Survey all focused on only the dietary and nutrient intake of Americans. The DHKS also included some examination of nutrition knowledge and attitudes. -

What is the name of the survey that currently collects information on nutrient intake and health status in the United States? a. Diet and Health Knowledge Survey b. National Health and Nutrition Examination Survey c. Nationwide Food Consumption Survey d. Continuing Survey of Food Intake of Individuals

Decreased partial pressure of carbon dioxide (CH 6, Krause) When excessive metabolic production of acids occurs, the body compensates by stimulating increased expiration of carbon dioxide to reduce acid concentrations in the body. Whereas increased kidney excretion of bicarbonate would be the compensation mechanism for dealing with respiratory alkalosis, decreased excretion of bicarbonate would compensate for respiratory acidosis. In metabolic alkalosis, the lungs would compensate by reducing expiration of carbon dioxide. REF: p. 95 -

What is the primary disturbance when respiratory alkalosis occurs? a. Increased bicarbonate b. Increased partial pressure of carbon dioxide c. Decreased bicarbonate d. Decreased partial pressure of carbon dioxide

normal tissue metabolism (CH 6, Krause) ANS: A Hydrogen ion and acid formation results from normal metabolism. All foods, not just acidic ones, may result in the production of organic acids. Carbon dioxide is a common volatile acid that results from oxidation of carbohydrates, fats, and proteins. Bicarbonate is considered to be a base, and control of its concentration, along with carbon dioxide concentrations, is a primary focus of acid-base regulation. -

What is the primary means by which hydrogen ions are generated in the body? a. Normal tissue metabolism b. Ingestion of highly acidic foods c. Oxidation-reduction reactions d. Reabsorption of bicarbonate

b. 4 to 6 months (CH 16 Krause, pg 303) The iron stores in infants are adequate enough to provide for growth up to double their birth weight, which, in most cases, comes around 4 to 6 months. Although the iron in breastmilk is highly bioavailable, continuing to only breastfeed an infant past 4 to 6 months of age will lead to depletion of the infant's iron stores by 6 to 9 months. To prevent this depletion, additional sources of iron are needed in the infant's diet. -

When should the addition of iron to the diet of both formula-fed and breast-fed infants begin? a. 6 weeks b. 4 to 6 months c. 6 to 9 months d. 1 year

a. Fatty acids (CH 23 Krause, p. 428) Low- to moderate-intensity activities use fatty acids as a primary source of energy. High-intensity activities rely on glycogen and glucose as energy sources because these can be broken down faster than fat. Amino acids may be used for gluconeogenesis when other substrates for this process are limited. Glycerol may be used as an energy source by entering partway through the glycolytic pathway. - -

What is the primary source of energy for exercise of low to moderate intensity? a. Fatty acids b. Glycogen c. Amino acids d. Glycerol

c. Four or more times daily for type 1 DM and one to four times for type 2 DM (CH 30 Krause, p. 603) Patients with type 1 DM should perform SMBG four times or more daily, including before each meal and at bedtime. Patients with type 2 DM should perform SMBG one to four times daily to help in achieving glucose goals. Whenever they add to or modify their therapies, patients with either DM should test more often. - -

What is the recommendation for self-monitoring of blood glucose? a. Every morning and every night before bed b. When there is a change in activity level or diet c. Four or more times daily for type 1 DM and one to four times for type 2 DM d. At least eight times a day for type 1 DM

a. It is rarely needed. (CH 16 Krause, pg 304) Commercially prepared infant formulas are fortified with all necessary vitamins; therefore, infants being fed formulas rarely need supplementation. The Food and Drug Administration regulates the manufacture of infant formulas so that the nutrient levels provided are consistent with the Infant Formula Act of 1985. -

What is the recommendation for vitamin supplementation of infants fed commercially prepared formula? a. It is rarely needed. b. Supplement only vitamins. c. Only water-soluble vitamins should be supplemented. d. Only fat-soluble vitamins should be supplemented.

Reduced penetrance (ch 5, Krause) Reduced penetrance refers to when a person has the genotype that does not express as the phenotype. This concept is usually applied to the genetic inheritance of disease, such as when the pedigree of a disease is tracked through a family history. Dominance refers to the phenotypic expression of a single allele, whether it is associated with an autosome or a sex-linked chromosome. Linkage refers to how genes are sequenced with others in the DNA strand. REF: p. 72 -

What is the term that describes a gene that is present but not expressing a particular phenotype? a. Pedigree b. Reduced penetrance c. Dominance d. Linkage

b. Subluxation (CH 12 Krause, pg 192) Subluxations are musculoskeletal problems that chiropractors focus on manipulating to promote healing and wellness. Moxibustion is the use of heat in the attempt to restore chi. Meridians are the channels that carry the chi in traditional Oriental medicine. Pharmacognosy is the science of natural substances and is one of the areas included in naturopathy. -

What is the term that refers to the dislocation of part of the body, which is thought to interfere with normal nerve function? a. Moxibustion b. Subluxation c. Meridian d. Pharmacognosy

a. Diarrhea (CH 17 Krause, pg 322) The high osmolality of apple and pear juices can contribute to carbohydrate malabsorption and diarrhea and therefore should not be used in situations in which fluid replacement is needed. Additionally, excessive fruit juice consumption has been associated with growth failure in children. The juice can replace other food sources of energy and protein, contributing to the child's not feeling hungry but not providing the nutrients necessary for adequate growth. Constipation may result from a poor fiber intake. -

What may a child be at risk for if she drinks large amounts of apple or pear juice throughout the day and limited amounts of water or milk? a. Diarrhea b. Hypervitaminosis A c. Overhydration d. Constipation

c. Oxytocin (CH 15 Krause, pg 286) Oxytocin stimulates the myoepithelial cells of the mammary gland to contract, causing milk to move toward the nipple for feeding. Progesterone promotes the development of the mammary glands during pregnancy. Prolactin promotes milk production. Colostrum is the first milk that a woman produces around term. -

Which hormone promotes letdown? a. Colostrum b. Progesterone c. Oxytocin d. Prolactin

c. An increased ratio of fat mass to lean mass as an adult. (CH 17 Krause, pg 324-325) Adiposity rebound is associated with the gain of body fat starting between 4 and 6 years of age, and the earlier this occurs, the more likely it is that the person will be heavier as an adult. This will be reflected in a greater BMI (or weight-to-height ratio) as an adult and an increase in fat mass in proportion to overall body weight. The degree of fatness during adolescence is also a contributing factor to the development of adult obesity. -

What may be a consequence of a child experiencing an early adiposity rebound? a. An increased ratio of lean mass to fat mass as an adolescent. b. An increased ratio of lean mass to fat mass as an adult. c. An increased ratio of fat mass to lean mass as an adult. d. An increased height-to-weight ratio as an adult.

a. Fasting or refraining from food (CH 18 Krause, pg 340) Seventeen percent of female and 7% of male high school students reported fasting or refraining from food as means of weight control. A total of 7% of females and 4% of males reported using diet pills. A total of 53% of females and 28% of males reported using exercise as a means of weight control. -

What method of weight control is most used by adolescents? a. Fasting or refraining from food b. Eating fewer calories and less fat c. Use of diet pills, laxatives, or diuretics d. Exercise

c. Diet, exercise, and lifestyle modification all combined(CH 21 Krause, pg. 392) Weight reduction programs that integrate diet with physical activity and lifestyle modifications are the most likely to succeed. Whereas the promotion of weight loss by decreasing calorie intake is a primary component, physical activity has been demonstrated as being necessary for helping to maintain the weight loss. Behavior modification helps the person seeking weight loss learn about lifestyle and environmental factors that contribute to the promotion of weight gain. Inclusion of the understanding of what beyond food and activity contributes to obesity can help the patient make lifestyle changes to support the weight loss. - -

What methods of weight reduction have the highest rates of success? a. Diet combined with exercise b. Diet combined with lifestyle modification c. Diet, exercise, and lifestyle modification all combined d. Exercise combined with lifestyle modification

a. May reduce the prevalence of allergy during the first year of life (CH 26 Kruase, p. 482) Avoiding exposing infants to potential allergenic foods during infancy only decreases the prevalence of food allergies during the first year of life and delays the onset of atopic dermatitis. Breastfeeding is the preferred means of feeding infants for the first 6 months. However, as solid foods are introduced into the infant's diet, they should be presented one at a time, and known common food allergens should be avoided to reduce the risk of developing food allergies. Wise selection in combination with adequate breastmilk or formula should prevent malnutrition. Commercial formulas may be modified to remove potential allergens. Although as the child grows older, the likelihood of developing food allergies later in life may decrease, research has not demonstrated a long-term benefit to avoiding food allergens. - -

What might happen if allergenic foods are limited or avoided during infancy? a. May reduce the prevalence of allergy during the first year of life b. May lead to malnutrition c. May result in eliminating all commercial infant formulas from the infant's diet d. May prevent allergy after the first year of life

b. Functioning beta-cells in the pancreas (CH 30 Krause, p. 599) Oral glucose-lowering medications may be used in the treatment of patients with type 2 DM; therefore, the patient has to have beta-cell function for the production of insulin. The mechanisms of action for the oral glucose-lowering medications include the stimulation of pancreatic insulin secretion. Other mechanisms of action include reducing insulin resistance at muscle and adipose tissue and decreasing hepatic glucose output. - -

What must a patient demonstrate to be a candidate for use of oral glucose-lowering medications? a. Functioning alpha-cells in the pancreas b. Functioning beta-cells in the pancreas c. Functioning gastrointestinal mucosa d. Resistance to insulin at all times

d. Vitamin D (CH 15 Krause, pg 258) Because of reports of clinical rickets, the AAP recommends an additional 5 mcg of vitamin D daily for infants, starting at 2 months of age. The vitamin A status of the breastfeeding mother may affect the infant, but the DRIs have an increased value for vitamin A intake of lactating women. The DRIs for zinc are also increased for lactation. The calcium content of breastmilk is not related to the calcium intake of women, so the DRIs for calcium do not increase during lactation. -

What nutrient does the American Academy of Pediatricians (AAP) recommend to be supplemented at 2 months of age for breast-fed infants, although the DRI for the nutrient in lactating women is at the same level as that in nonpregnant women? a. Vitamin A b. Calcium c. Zinc d. Vitamin D

b. Excessive milk consumption (CH 17 Krause, pg 326) Milk anemia results from excessive intake of milk to the exclusion of other foods, and this could occur in a 1-year-old child because young children prefer milk to meat. Excessive fruit consumption is more associated with toddler ages, 2 to 3 years. At 1 year of age, a child will participate in feeding herself finger foods, but a caregiver should still assist in the feeding process. By 1 year of age, most children can eat the same foods as adults, and a well-selected diet can provide adequate iron. -

What reason would most likely explain why a 1-year-old child is experiencing iron deficiency? a. Excessive fruit juice consumption b. Excessive milk consumption c. Child is feeding herself, particularly finger foods d. Child is eating table foods

b. They are cariostatic. (CH 25 Kruase, p 470) Nuts do not contain a significant amount of fermentable carbohydrates and therefore are cariostatic. Other cariostatic foods include protein foods such as seafood, meat, eggs, and poultry; fats such as oils, butter, margarine, and seeds; and most vegetables. - -

What role do nuts play in regard to the development of dental caries? a. They are cariogenic. b. They are cariostatic. c. They are anticariogenic. d. They are cariogenic when eaten with fruit.

c. Eat small, frequent meals low in fat. (CH 15 Krause, pg 270) Eating small, frequent dry meals of either carbohydrate or protein seems to reduce nausea in pregnant women. Although taking liquids between meals has been historically advised, no research has validated the suggestion. Meals high in fat tend to stay in the stomach longer, more likely promoting nausea and vomiting. The best recommendation is to eat whatever does not promote nausea and avoid odors that trigger nausea in the individual. -

What should a pregnant woman do to relieve nausea and vomiting during the early months of her pregnancy? a. Drink liquids with meals and have small, frequent feedings. b. Eat high-fat foods and include liquids with meals. c. Eat small, frequent meals low in fat. d. Eat three regular meals per day.

b. Glucose and electrolytes in water (CH 28 Krause, p. 532) Oral rehydration therapy solutions for infants and children should contain 2% glucose and sodium and potassium. Additional vitamins and minerals are not included because they would increase the osmolality of the solutions and promote potential intolerance. Chemically defined and lactose-free formulas are usually used for more complete nutrition repletion as opposed to simple rehydration. - -

What should be in the solution used for fluid replacement for acute diarrhea in infants and small children? a. Vitamins, minerals, and electrolytes b. Glucose and electrolytes in water c. Chemically defined formula d. Lactose-free formula

b. Replacement of lost fluids and electrolytes (CH 28 Krause, p. 528) To resolve or prevent fluid and electrolyte imbalances, the first step in treating diarrhea is replacing the lost fluids and electrolytes through use of electrolyte solutions, soups, broths, vegetable juices, or other isotonic liquids. Later, starchy carbohydrates and low-fat foods may be introduced to limit the secretion of fluids into the GI tract and to prevent rapid GI transit. This could include modest amounts of pectin and fiber (<20 g). - -

What should be included in the initial treatment of diarrhea? a. Low-fat, low-fiber diet b. Replacement of lost fluids and electrolytes c. Increase of high-pectin foods d. High-fiber diet to increase stool bulk and restore normal bowel motility

c. Promote weight maintenance and increased physical activity. (CH 21 Krause, pg 402) As children and adolescents are still in growth phases, restriction of energy and nutrient intake and promotion of weight loss may impair appropriate growth and physical maturation. Unless the child is experiencing other medical conditions aggravated by excessive weight, the recommended approaches to weight control are providing adequate calories to maintain weight and increasing the child's physical activity, allowing the child to grow into his weight. If a child or adolescent has exceeded his optimal adult weight, then a slow, 10- to 12-lb/year weight loss is recommended. - -

What should be the treatment goal in regard to a child or adolescent who is identified as overweight? a. Promote weight loss of 1 to 2 lb/week. b. Promote weight loss to reduce BMI below the 85th percentile. c. Promote weight maintenance and increased physical activity. d. Promote weight loss of 10 to 12 lb/year.

c. Zinc and potassium status (CH 28 Krause, p. 545) Although maldigestion, malabsorption, and drug-nutrient interactions may contribute to the need for overall vitamin and mineral supplementation in patients with Crohn disease, diarrhea experienced by the patient promotes the reduction of potassium, selenium, and zinc stores in particular. - -

What should patients with Crohn disease be monitored for when experiencing diarrhea? a. Calcium and phosphorus status b. Magnesium and iron status c. Zinc and potassium status d. Iron and selenium status

b. Grapes and hot dogs (CH 16 Krause, pg 310) Foods with skins or rinds, such as grapes and hot dogs, and foods that stick to the roof of the mouth, such as peanut butter, should not be offered to infants and young children. Foods should be well cooked, mashed, or finely chopped to limit the hazard of choking. -

What should the caregiver avoid feeding an infant to reduce the risk of choking? a. Cheese and pudding b. Grapes and hot dogs c. Graham crackers and cheese d. Pasta, such as macaroni

b. Decrease insulin dosage dependence on duration and intensity of exercise. (CH 30 Krause, p. 598) Decreasing the amount of insulin injected is necessary to prevent hypoglycemia that can occur during exercise. Prolonged or intensive exercise may require a modest decrease of 1 to 2 U of rapid- and short-acting insulin before and possibly after exercise. As an alternative, the person with type 1 DM may ingest carbohydrate before or after exercise to prevent hypoglycemia. Exercising at the time of insulin peak action and providing extra insulin can increase the likelihood of hypoglycemia. - -

What should the person with type 1 DM do when planning to exercise? a. Strictly adhere to dietary restrictions. b. Decrease insulin dosage dependence on duration and intensity of exercise. c. Plan to exercise when the insulin is peaking. d. Take an extra injection of insulin.

a. Nausea, abdominal cramping, and diarrhea (CH 27 Krause, pp. 521-522) Signs and symptoms in the early stage of dumping syndrome include abdominal fullness and nausea within 10 to 20 minutes after eating, flushing, rapid heartbeat, faintness, and sweating. In the intermediate stage of dumping syndrome, 20 to 60 minutes after eating, symptoms include abdominal bloating, flatulence, cramping pains, and diarrhea. In the late stage, 1 to 3 hours after eating, patients experience reactive hypoglycemia, which is exhibited through sweating and feelings of anxiety, weakness, or hunger. - -

What symptoms of dumping syndrome follow the total or subtotal removal of the stomach? a. Nausea, abdominal cramping, and diarrhea b. Elevated blood pressure, headache, and substernal pain c. Heartburn, vomiting blood, and hypoglycemia d. Decreased saliva production, steatorrhea, and bloating

c. Osmotic diarrhea (CH 28 Krause, p. 528) When a person with lactase deficiency consumes a lactose load, the presence of lactose in the bowel promotes an osmotic effect, drawing fluid into the bowel. Exudative diarrheas result from damage to the intestinal mucosa. Malabsorptive diarrheas occur when there is a reduction to the absorptive surface area of the bowel or rapid transit of bowel contents. Secretory diarrheas result in the loss of fluid and water into the bowel, but this is caused by bacterial toxins, viruses, or increased intestinal hormone secretions. - -

What type of diarrhea may be experienced by a patient with lactase deficiency? a. Exudative diarrhea b. Malabsorptive diarrhea c. Osmotic diarrhea d. Secretory diarrhea

a. Nasogastric (CH 13 Krause, pg 210) A nasogastric tube is inserted through the nares and runs down the throat and esophagus to end in the stomach. A nasojejunal tube is inserted in the same manner, but its terminus is in the jejunum. A PEG tube is placed through the abdominal wall, so it feeds directly into the stomach. A PEJ is similar except that it feeds into the jejunum. -

What type of enteric tube enters the body at the nose and terminates at the stomach? a. Nasogastric b. Nasojejunal c. Percutaneous endoscopic gastrostomy d. Percutaneous endoscopic jejunostomy

d. Peppermint (CH 27 Krause, p. 513) Peppermint is known to decrease the lower esophageal sphincter pressure, contributing to the development of esophageal reflux. High-acid foods may cause pain in the esophagus when it is already inflamed. Milk, yogurt, and high-protein foods may temporarily buffer gastric secretions, but they also stimulate the secretion of gastrin, pepsin, and acid - -

What type(s) of foods should be avoided in a patient with GERD because it(they) decrease(s) LES pressure? a. High-acid foods b. Low-fat milk and yogurt c. High-protein foods d. Peppermint

c. She plays with or throws her utensils. (CH 16 Krause, pg 307-309) By age 7 to 9 months, an infant has the developmental coordination to pick up and handle utensils through a refined pincer grasp. This allows her to be more active in participating in her feeding, as well as in signaling when she is done. Falling asleep when sated is common in infants up to 3 months of age. From 4 to 6 months, fussing, crying, or losing attentiveness toward the food and paying more attention to the surrounding environment are signals of being done eating. A 10- to 12-month-old infant will hand her mother her cup or bottle as a signal of being done. -

What would be a behavior demonstrated by a 7- to 9-month-old to indicate that she is sated and finished eating? a. She falls asleep. b. She pays more attention to her surroundings. c. She plays with or throws her utensils. d. She hands her cup or bottle to her mother.

c. The adolescent is at risk for becoming overweight. (CH 18 Krause, pg 341) As in children, the plotting of an adolescent's BMI above the 85th percentile but below the 95th percentile is interpreted as being at risk for becoming overweight. A BMI above the 95th percentile is interpreted as being overweight. For an interpretation of obesity, the BMI would have to be significantly above the 95th percentile. -

What would be the correct interpretation of an adolescent who has a BMI above the 85th percentile? a. The adolescent is overweight. b. The adolescent is obese. c. The adolescent is at risk for becoming overweight. d. The adolescent is overfat.

c. reframing. (CH 14 Krause, pg 234) Reframing is a technique where the counselor makes a statement that provides a different perspective to the patient's view. Self-motivational statements are created by the client when she recognizes that a problem or concern exists and that she can do something about it. Problem recognition, concern, intention to change, and optimism are types of questions that the counselor can ask that will result in the client developing self-motivational statements. -

When a counselor elicits self-motivational statements, this includes all of the following EXCEPT a. optimism. b. problem recognition. c. reframing. d. intention to change.

c. 4 to 6 months (CH 16 Krause, pg 309-310) Introduction of semisolid foods depends on both the infant's developmental readiness and his or her nutrient needs. To maintain iron nutriture, additional sources of iron need to be introduced into the diet by 4 to 6 months of age. However, to be able to feed an infant foods other than formula, the infant needs to be able to demonstrate head and neck control, voluntary movements of the tongue, chewing movements, and the ability to sit unsupported. -

When should the introduction of semisolid and strained foods to the infant's diet begin? a. 6 to 8 weeks b. 2 to 4 months c. 4 to 6 months d. 6 to 8 months

a. Antral and pyloric dysfunction, poor peristalsis, and diminished gastric acid secretion (CH 27 Kruase, p 521) The vagus nerve controls the motility of the stomach, the tone of the pylorus, and the secretion of gastrin and stomach acid. Depending on the type of vagotomy, the antrum and pylorus will exhibit decreased motility and tone, resulting in delayed stomach emptying and decreased gastric acid secretion. The vagotomy is performed below the point of innervation to the lower esophageal sphincter (LES) and therefore will not affect swallowing or LES tone. Some impairments in digestion may result due to decreased acid secretion; however, these can be accommodated for by diet manipulation and do not require lifelong need for parenteral nutrition. - -

When a gastrectomy is performed with an accompanying vagotomy, what physiologic changes occur? a. Antral and pyloric dysfunction, poor peristalsis, and diminished gastric acid secretion b. Antral and pyloric dysfunction, poor peristalsis, and high gastric acid secretion c. Impaired digestion leading to the need for parenteral nutrition almost all the time d. Dysphagia because of a relaxed LES

a. Recent changes in the patient's weight (CH 25 Krause, p. 474) As dentures replace missing teeth, they need to be checked for appropriate fit. The fit of the dentures could be affected by weight changes. If a patient has lost weight recently, the dentures may fit loosely, and this could result in continuing decreased intake. Dentures do need to be cared for, so the patient or caregiver should perform regular cleaning. Fluid intake contributes to maintaining the dentures; however, adequacy of fluid intake is not associated with denture use. Dentures are made of materials that are not susceptible to cariogenicity. - -

When a patient is identified as wearing full dentures, what else should be included in the nutrition assessment of the patient? a. Recent changes in the patient's weight b. The patient's regular daily fluid intake c. When the patient last had her own teeth d. The patient's daily intake of cariogenic foods

c. Vitamin B12 (CH 27 Krause, p 512) An H2 blocker is used to decrease stomach acid production. This can result in the development of achlorhydria and the loss of intrinsic factor. Patients should be monitored for serum vitamin B12 level to ensure that a deficiency in this vitamin does not develop. - -

When an H2 blocker is used in the treatment of peptic ulcer disease, the patient may be at risk for deficiency of which nutrient? a. Vitamin A b. Vitamin B6 c. Vitamin B12 d. Vitamin D

b. From 4 months' gestation through the preteen years (CH 25 Krause, p. 468) Tooth formation begins at 2 to 3 months' gestation, and tooth mineralization begins at 4 months' gestation. Tooth mineralization continues into the preteen years as the baby teeth are replaced by the adult teeth. - -

When and for how long does tooth mineralization occur? a. From 2 months' gestation through 5 years of age b. From 4 months' gestation through the preteen years c. From 2 months' gestation through the preteen years d. From 4 months' gestation through 5 years of age

a. Right after school (CH 18 Krause, pg 338-339) Teens more frequently visit fast-food restaurants on the way home from school. This may have to do with the location of the fast-food restaurants in relation to their schools because many are located within a short walking or driving distance. Weekday dinnertime is the next most frequently reported visit time. On the weekends, adolescents visit fast-food restaurants more during the lunchtime than during the dinnertime hours. -

When are adolescents most likely to visit a fast-food restaurant? a. Right after school b. During dinnertime through the week c. During dinnertime on the weekend d. During lunchtime on the weekend

d. use a certified translator. (CH 14 Krause, pg 230) Unofficial translators are not usually a good choice because of the lack of understanding of nutrition and health. Using professional translators is also not without limitations but is the best option. -

When counseling a client who speaks a different language, the most effective way to communicate is to a. use a family member or close friend who speaks the language and can translate. b. use a visual aid that has been developed using international symbols. c. find a counselor who speaks the same language and can do the counseling. d. use a certified translator.

a. Over the 18- to 24-month "growth spurt" period (CH 18 Krause, pg 333) Although linear growth occurs throughout the 4 to 7 years of puberty, the greatest increase is during the growth spurt. Growth slows down but continues after sexual maturation. -

When does the greatest increase in height during puberty occur? a. Over the 18- to 24-month "growth spurt" period b. Continually over the entire time c. During the first year of puberty d. Primarily toward the end of puberty

b. By approximately 30 years of age (CH 24 Krause, p. 457) Peak bone mass occurs by about 30 years of age. Through adolescence, the opportunity exists to increase bone mineral density by promoting calcium intake, maintaining weight-bearing activity, and limiting use of alcohol and cigarettes. Around the age of 40 years, BMD gradually declines. After menopause, the rate of loss averages from 1% to 2% per year. - -

When is the peak bone mass of a female typically achieved? a. By the end of adolescence b. By approximately 30 years of age c. Within the decade preceding menopause d. After the onset of menopause

b. Growth channel (CH 16 Krause, pg 300) As a growth pattern develops on a growth chart, the curve produced is known as the growth channel and is reported as a percentile based on the chart. Growth velocity is a measure of the rate of growth during a specified period of time and usually starts high in infancy and then decreases as the person gets older. Catch-up growth is when a smaller infant starts to reach his or her genetic growth potential by increasing growth rate during the first year of life, and lag-down growth is when a larger infant's growth rate decreases to its genetic potential. -

When plotting an infant's weight or length on a percentile growth chart, a curve is produced that can be used to follow the infant's growth. What is this curve called? a. Growth velocity b. Growth channel c. Catch-up growth d. Lag-down growth

It can provide 1.1 kcal/ml of lipid.(CH 8 Krause pg 136) Propofol provides fat calories to a patient, and this must be considered when determining total calories and other sources of fat to administer to a patient under sedation. Laxatives such as FiberCon and Fiber-Lax contain extra calcium, which may be nutritionally significant. Various drugs use lactose or wheat starch as excipients, and these must be considered when dealing with patients with lactose intolerance and celiac disease. -

When the sedative propofol (Diprivan) is administered to a patient, what nutritional consequence must be considered? a. It can provide 1.1 kcal/ml of lipid. b. It can provide extra calcium. c. It contains lactose and may promote lactose intolerance. d. It contains wheat starch and may affect people with celiac disease.

Low serum sodium because water is retained (Chapter 6, Krause) Vasopressin, also known as antidiuretic hormone, stimulates renal reabsorption of water. In syndrome of inappropriate diuretic hormone, excessive secretion of vasopressin retains water and results in hyponatremia and low urine output. Aldosterone is the hormone that stimulates renal sodium retention, and when present, the kidneys excrete potassium in exchange for the sodium, which then attracts the retention of water. REF: p. 88 -

When vasopressin is excessively secreted in the blood, which of the following effects occurs? a. Low serum potassium because water is retained b. Low serum sodium because water is retained c. High serum potassium because water is excreted d. High serum sodium because water is excreted

b. Yoga (CH 12 Krause, pg 192) Yoga is considered to be a manipulative therapy because it involves manipulation of the body. Homeopathy is classified by NCCAM as an alternative medical system. Cognitive behavior therapy is a type of mind-body therapy. Magnetic therapy is a type of medicine that is based on energy therapy. -

Which CAM would be considered a manipulative therapy? a. Homeopathy b. Yoga c. Cognitive behavior therapy d. Magnetic therapy

Metabolic alkalosis (CH 6, Krause) Metabolic alkalosis results from a loss of acid, such as HCl through gastric suctioning, and loss of extracellular fluid, such as would occur in diuretic use or vomiting. Respiratory alkalosis occurs when excessive respiration occurs, such as heavy breathing after exercise or in anxiety. Respiratory acidosis happens when there is damage to the lungs or respiration that does not allow for the exchange of carbon dioxide with oxygen. Metabolic acidosis results from medical conditions that lead to a buildup of metabolic acids in the body. REF: p. 96 -

Which acid-base imbalance can result from diuretics use, vomiting, and loss of chloride? a. Respiratory alkalosis b. Respiratory acidosis c. Metabolic alkalosis d. Metabolic acidosis

a. 0 to 2 years of age (CH 26 Krause, p. 479) Infants younger than 2 years of age are more likely to develop food allergies than are older children and adults. The incidence of food allergies is estimated to be 4% to 7% in children and from 1% to 2% in adults. - -

Which age group is most likely to develop food hypersensitivity? a. 0 to 2 years of age b. 3 to 12 years of age c. Adolescents d. Elderly

b. Proteomics (ch5, Krause) Proteomics focuses on studying the proteins produced by genes. Metabolomics focuses on the study of cellular metabolites that may be useful in the monitoring of disease progression. Nutrigenomics studies the influence of nutrient and other food factors on the expression of genes. Genomics is the all-encompassing study of genes and proteins and their relation to disease, including the influences of environmental factors. REF: p. 66 -

Which area of study focuses on the identification and understanding of the function of the end products of the genes within the human genome? a. Nutrigenomics b. Proteomics c. Metabolomics d. Genomics

c. Chewing sugar-free gum (CH 25 Krause, p. 470) Chewing sugar-free gum after a meal can help reduce its cariogenicity by promoting salivary stimulation, which clears fermentable carbohydrates from the surface of the teeth. Eating dessert, pretzels, or fruit at the end of a meal can have a cariogenic effect by contributing to fermentable sugars being available for plaque formation. - -

Which can reduce the cariogenicity of a meal when it follows the meal? a. Eating dessert b. Eating a snack of pretzels c. Chewing sugar-free gum d. Eating fruit

a. Loss of lean body mass, decreased metabolic rate, and decreased function (CH 20 Krause, pg 369-370) As lean body mass decreases, the loss of muscle mass results in decreased strength, function, and metabolic rate. Sarcopenia occurs more rapidly in people with limited physical activity; therefore, elderly adults who are also obese tend to be less active and retain excessive adipose tissue. - -

Which changes are associated with the normal process of aging? a. Loss of lean body mass, decreased metabolic rate, and decreased function b. Loss of lean body mass, increased metabolic rate, and increased physical activity c. Increased lean body mass, increased metabolic rate, and increased physical activity d. Increased muscle strength and function

a. Those who do not eat meat or seafood. (CH 17 Krause, pg 317) Primary sources of zinc in the diet are foods of animal origin. Because of the costs of meat and seafood, children from families with low incomes are more at risk for developing zinc deficiency. Milk and vegetables are not primary sources of zinc. -

Which children are at risk for having a zinc intake lower than the recommended level? a. Those who do not eat meat or seafood. b. Those who do not receive zinc supplementation. c. Those who drink too much milk. d. Those who do not like many vegetables.

a. Severe acute pancreatitis (CH 13 Krause, pg 210) Severe acute pancreatitis may result in inadequate digestive and absorptive capacity in the gastrointestinal tract. In this situation, providing enteral feeding may not be able to provide adequate nutrition. Pancreatitis may be associated with severe abdominal pain associated with feeding or with ileus. Enteral nutrition can be provided to patients with HIV/AIDS, patients who have failure to thrive, and patients with oral or esophageal trauma because the gastrointestinal tract is still accessible in these conditions. -

Which condition would most likely require parenteral nutrition as the medical nutrition therapy? a. Severe acute pancreatitis b. HIV/AIDS c. Failure to thrive d. Oral or esophageal trauma

d. CAP-FEIA (CH 26 Krause, p. 491) CAP-FEIA is a quantitative assessment of allergen-specific IgE antibodies; higher levels of antibodies are often, but not always, predictors of clinical symptoms. It has been approved for only six foods: egg, milk, peanut, fish, wheat, and soy. - -

Which diagnostic methods for determining food allergy by immunologic function are the most accurate? a. Skin-prick test, ELISA, and RAST b. Cytotoxic testing and ELISA c. Sublingual testing, RAST, and skin-prick test d. CAP-FEIA

b. Diverticulitis (CH 28 Krause, p. 548) Diverticular disease is one of the most common medical conditions in industrialized countries and incidence increases with age. It is more common in people over 50 years. UC, Crohn disease, and tropical sprue generally occur earlier in life. - -

Which disease is associated with advanced age? a. Crohn disease b. Diverticulitis c. Ulcerative colitis d. Tropical sprue

a. Increased volume of the brain cells (Ch 6, Krause) intracellular fluid volume increases. The increase in the volume of brain cells results in headaches, nausea, blindness, vomiting, and convulsions. Hypertension, decreased circulating blood volume, and changes in urinary output tend to be associated with or the cause of hypovolemia. REF: p. 88 -

Which effect is of greatest concern in water intoxication? a. Increased volume of the brain cells b. Hypertension c. Decreased circulating blood volume d. Increased urinary output

b. Lipoprotein lipase (CH 21 Krause, pg 385) Lipoprotein lipase hydrolyzes triglycerides from VLDL into fatty acids and glycerol. The glycerol travels back to the liver while the fatty acids are incorporated into adipocytes. Hormone-sensitive lipase hydrolyzes triglycerides packaged within the adipocytes so that fatty acids and glycerol can be released back into circulation. CCK's action at the brain level is to inhibit food intake. Insulin is the hormone that promotes lipid storage. - -

Which enzyme removes dietary triglyceride from the blood? a. Hormone-sensitive lipase b. Lipoprotein lipase c. Cholecystokinin d. Insulin

d. Providing small, frequent meals (CH 17 Krause, pg 326) Because of children's small stomach size and variability in hunger sensation, providing small, frequent meals with age-appropriate, nutrient-dense foods promotes a child's intake of foods. Lack of appetite and inappropriate feeding practices contribute to failure to thrive. Parents may inappropriately restrict foods or amounts of food out of fear their children will develop obesity or other chronic diseases. Inadequate fiber intake is associated with constipation development, which can impair a child's appetite. Excessive fruit juice intake replaces calories from other nutrient-dense foods and limits intake of nutrients necessary for growth, such as protein. -

Which factor has NOT been associated with failure to thrive? a. Food restriction caused by parental concerns about obesity b. Inadequate fiber intake c. Excessive fruit juice intake d. Providing small, frequent meals

d. All of the above (CH 27 Krause, p. 519) The etiology of gastric cancer is multifactorial, but more than 80% have been attributed to H. pylori infection. In addition, diet, lifestyle, and genetic factors contribute. A Western diet, high in processed meats, fat, and salt, is associated with an increased risk of gastric cancer. Populations that eat a lot of pickled foods, cured foods, and other high-salt foods have a higher incidence. Smoking and obesity have also been linked to gastric cancer - -

Which factor has been associated with developing gastric cancer? a. Smoking b. Obesity c. A diet high in salt and fat d. All of the above

c. Put the infant to bed without a bottle. (CH 16 Krause, pg 310) Giving an infant a bottle at bedtime with either milk or fruit juice allows the teeth to be bathed in the simple carbohydrate that promotes dental caries development. To promote dental health, infants should be fed and burped and then put to bed without a bottle. Also, infants and young children should be limited to 4 to 6 fl oz of juice per day. -

Which feeding practice will avoid the development of tooth decay in infants? a. Give the infant a bottle with fruit juice at bedtime. b. Give the infant a bottle with milk at bedtime. c. Put the infant to bed without a bottle. d. Give the infant a bottle at bedtime only when he or she appears fussy.

c. Banana (CH 26 Krause, p. 486) Bananas, avocados, chestnuts, and kiwi are the most frequent food-latex cross-reactive allergies. Milk allergies have not been seen to cross-react with latex. Wheat and fish have been associated with latex cross-reactions but not as frequently as various fruits - -

Which food has one of the most frequent cross-reactivity allergies between latex and food? a. Milk b. Wheat c. Banana d. Fish

a. Nuts (CH 26 Kruase, p. 484) Peanuts and tree nuts are believed to cause the most near-fatal and fatal anaphylactic reactions to foods. About 30,000 anaphylactic reactions and 200 deaths occur per year in the United States. - -

Which food is the most common cause of near-fatal and fatal anaphylactic reactions? a. Nuts b. Shellfish c. Egg d. Wheat

c. Those with food allergies (CH 19 Krause, pg 360) People with food allergies are at particular risk because they are often forced to eliminate whole food groups from their diet. Several segments of the adult population fall into high-risk groups that are unlikely to meet their nutrient needs because of life stage, alcohol or drug dependency, food insecurity, chronic illness, or choosing a restrictive diet. -

Which group is at risk of not eating the RDI of nutrients and may benefit by the use of multivitamin supplements? a. Athletes b. Premenopausal women c. Those with food allergies d. Men older than age 50 years

albumin (Ch 7 Krause pg 104) Albumin has a half-life of about 3 weeks. Prealbumin has a half-life of 2 days. Retinol-binding protein has a half-life of about 12 hours. Transferrin has a half-life of 8 days. -

Which indicator of protein status has the longest half-life? a. Albumin b. Prealbumin c. Retinol-binding protein d. Transferrin

b. Introverted (CH 22 Krause, pgs. 408 & 410) Psychologic features of a person with anorexia nervosa include perfectionism, harm avoidance, overly restrained emotional expression, and limited social spontaneity. People with bulimia nervosa tend to be more extroverted and easygoing and are more likely to engage in risk-taking behaviors. - -

Which is a typical personality characteristic of people with anorexia nervosa? a. Extroverted b. Introverted c. Underachiever d. Easygoing

b. Follow a nutrient-dense, phytochemical-rich food plan. (CH 19 Krause, pg 359) The Dietary Guidelines for Americans focus on eating nutrient-dense foods, but there is the added benefit that the identified foods tend to also be phytochemical rich. Focusing on lipid intake limits awareness of the variety of benefits that arise from the intake of whole grains, fruits, and vegetables. Dietary recommendations for adolescents focus on the promotion of adequate growth and development during adolescence, and in many cases, they exceed the needs of the average adult. A carefully chosen vegan diet supplemented with specific nutrients can be healthy, but vegan diets (and other very restrictive diets) are generally considered a risk factor for inadequate nutrition. -

Which is an appropriate nutritional guideline for the adult years? a. Focus primarily on lipid intake. b. Follow a nutrient-dense, phytochemical-rich food plan. c. Follow a high-protein, low-carbohydrate diet. d. Follow a vegan diet.

a. Albumin (Ch 7 Krause, pg 101-102) The Centers for Medicare and Medicaid Services established that for the BMP, the following tests are reimbursable: glucose, calcium, potassium, carbon dioxide, chloride, blood urea nitrogen, and creatinine. For the CMP, the following are added: albumin, total protein, alkaline phosphate, alanine aminotransferase, aspartate aminotransferase, and bilirubin. WBC would appear on a CBC, which may include a differential identifying the levels of the different types. -

Which laboratory value would be added in a comprehensive metabolic panel (compared with a BMP)? a. Albumin b. Glucose c. Blood urea nitrogen d. White blood cells

A-magnesium (Beerman, CH 12) - -

Which mineral stabilizes enzymes and neutralizes anions? a. Magnesium b. Chloride c. Potassium d. Sodium

anemia (CH 7 Krause pg 105) The fecal occult blood test is routinely ordered for adults older than age 50 years and younger adults with unexplained anemia. The presence of blood may indicate bleeding in the gastrointestinal tract that is causing the anemia. -

Which nutrient-related disorder is likely to result in a stool test being ordered? a. Diabetes b. Anemia c. Night blindness d. Cheilosis

b. Sprinting (CH 23 Krause, p. 427) Activities that involve high intensity and short duration rely on anaerobic metabolism for energy. Sprints or running drills use muscle glycogen anaerobically to provide the energy quickly. Longer duration activities such as jogging, hiking, and dancing cause a switch to aerobic glycolysis of the glucose produced from muscle glycogen breakdown. - -

Which of the following activities relies heavily on the anaerobic pathway in muscle cells? a. Jogging b. Sprinting c. Hiking d. Dancing

a. Wheat, rye, barley, and oats (CH 28 Krause, p. 533) Wheat, rye, barley, and oat products all contain gluten peptide fractions and should be excluded from the diet. Corn, potatoes, rice, soybeans, tapioca, arrowroot, amaranth, quinoa, millet, and buckwheat are sources of grains that may be used in creating products for people with celiac disease. - -

Which of the following are NOT included in the gluten-restricted, gliadin-free diet to treat celiac disease? a. Wheat, rye, barley, and oats b. Corn, rice, soybeans, and tapioca c. Potatoes, hydrolyzed vegetable protein, and arrowroot starch d. Wheat, rice, barley, and corn

c. Growth measurements of height, weight, and weight for height should be determined at regular intervals. (CH 17 Krause, pg 314) Children change their eating and activity patterns, causing changes in growth. Because these changes are not always predictable, by checking growth measures at regular intervals, potential growth problems can be identified and treated early. Depending on the child's eating and activity, a child for whom a growth channel had been established could stray from that channel. -

Which of the following are appropriate general guidelines for measurements to assess the growth of a child? a. Growth measurements should be made at 1, 4, and 10 years. b. After the child's channel on the growth charts has been determined, it is not necessary to reevaluate on a regular basis. c. Growth measurements of height, weight, and weight for height should be determined at regular intervals. d. After the growth pattern is established, it will not change until adolescence.

b. Osteoblasts (CH 24 Kruase, p. 456) Osteoblasts are the cells responsible for the formation of bone tissue. Osteocytes are formed by osteoblasts and are incorporated into the mineralized bone. Osteoclasts are responsible for the resorption or breakdown of bone. Osteocalcin is a matrix protein of bone. - -

Which of the following are bone-forming cells? a. Osteocytes b. Osteoblasts c. Osteoclasts d. Osteocalcins

a. High content of protein, calcium, and phosphorus (CH 25 Krause, p. 470) Milk and dairy products are considered to have low cariogenic potential because they have a high calcium and phosphorus content. Protein in food is considered to provide a cariostatic effect. - -

Which of the following are characteristics of foods having a low cariogenic potential? a. High content of protein, calcium, and phosphorus b. Low content of protein, calcium, and phosphorus c. Low content of phosphorus and protein d. High content of magnesium and fat

d. GMP (CH 12 Krause, pg 197) Good Manufacturing Practices are a joint endeavor between the government and industry to develop high quality manufacturing guidelines for all dietary supplements. The Cochrane Database Review only reviews studies in regard to the efficacy of use of supplements. The Dietary Supplement Health and Education Act of 1994 did not establish standards or guidelines in regard to supplement production, nor did it give the FDA authority to ensure the quality and efficacy of dietary supplements. -

Which of the following assures that a dietary supplement is a quality product that contains what the label states? a. CDR b. DSHEA c. FDA d. GMP

b. Roux-en-Y gastric bypass (CH 21 Krause, pg 398) The roux-en-Y gastric bypass creates both a reduced stomach reservoir to limit food intake and connects the small intestine to an opening near the reservoir to bypass some of the absorptive surface of the small intestine. Gastric banding and vertical banded gastroplasty only create the limited size reservoir for food intake. Liposuction is technically a cosmetic surgery because it is usually limited to removal of subcutaneous tissue. - -

Which of the following bariatric surgeries promotes weight loss through a combination of food restriction and malabsorption? a. Gastric banding b. Roux-en-Y gastric bypass c. Vertical banded gastroplasty d. Liposuction

a. Low serum ferritin levels can have a detrimental effect on exercise performance. (CH 23 Krause, p. 443) As adequate iron is necessary for oxygen transport in both blood and muscle as well as in the production of ATP, low serum ferritin levels of iron can limit aerobic endurance and the capacity for work. Iron is also lost through sweat. Iron supplementation may help improve iron stores, but no evidence is available regarding the impact of iron supplementation on improving performance. - -

Which of the following best characterizes the role of iron in exercise? a. Low serum ferritin levels can have a detrimental effect on exercise performance. b. Exercise performance is affected only in cases of severe iron deficiency anemia. c. Mild iron deficiency has no detrimental effects. d. Iron plays no significant role in muscle activity.

b. Direct serum bilirubin (CH 29 Krause, p. 561) Direct serum or conjugated bilirubin is a measure of the liver's excretory function. Levels increase when liver function and excretion are impaired. Serum alkaline phosphatase is an indicator of cholestasis. Alanine aminotransferase is hepatocyte cytosolic enzyme. Levels of ALT increase when liver cells are damaged. Prothrombin time is a measure of the liver's protein synthesis. - -

Which of the following biochemical measures is a test of the liver's excretion? a. Serum alkaline phosphatase b. Direct serum bilirubin c. Alanine aminotransferase d. Prothrombin time

b. BMD (CH 24 Krause, p. 457) After peak bone mass is achieved by about 30 years of age, bone mineral density is the term used to describe bone throughout the rest of life. Bone mineral content is used to describe the amount of bone accumulated before the end of growth (usually in height). Alkaline phosphatase measured in the blood is a marker of bone formation. - -

Which of the following bone measurements is more appropriate for individuals from late adolescence to the elderly years? a. BMC b. BMD c. PBM d. Alkaline phosphatase

a. Root caries (CH 25 Krause, p. 471) When the gingiva recedes, this exposes the root of the tooth to plaque formation, causing root caries. Lingual caries, which occurs on the tongue side of the anterior teeth, is seen in people with bulimia or anorexia-bulimia. Caries development is associated with decreased salivation and erosion of tooth enamel. - -

Which of the following can commonly occur after a patient develops gingival recession? a. Root caries b. Lingual caries c. Increased salivation d. Increased enamel production

d. Maternal illness (CH 15 Krause, pg 283) Illness in a breastfeeding mother diverts energy stores away from adequate milk production. Smoking is a factor that can interfere with the letdown reflex, impairing milk flow. Failure to thrive because of poor intake is commonly associated with improper breastfeeding technique and poor suck on the part of the infant. An infant who is small for gestational age may develop failure to thrive because of his or her increased energy needs but limited size to handle increased food intake. -

Which of the following can promote failure to thrive in a breast-fed infant as a result of poor milk production? a. Poor latching or suck during the feeding episode b. Small for gestational age infant c. Maternal smoking d. Maternal illness

b. Diminished absorption of vitamin B12 (CH 20 Krause, pg 371) When the gastric mucosa atrophies, there is a reduction in the secretion of intrinsic factor. This is necessary to promote the absorption of vitamin B12 from dietary sources. Reduced acid production also affects the absorption of nonheme iron but not necessarily calcium. The acid is also needed for the denaturation of protein. -

Which of the following can result from atrophic gastritis associated with aging? a. Decreased calcium and nonheme iron absorption b. Diminished absorption of vitamin B12 c. Decreased absorption of carbohydrates d. Increased intestinal transit time

c. Increased renal excretion of calcium (CH 24 Kruase, p. 464) Although an excessive sodium intake may contribute to an increased renal excretion of calcium, aging itself does not result in this physiologic change. Aging is associated with decreased intestinal absorption of calcium through alterations in the gastrointestinal mucosa and in vitamin D biosynthesis and metabolism. In contrast to osteoblast activity, osteoclast activity increases because of the uncoupling of bone remodeling. - -

Which of the following changes related to nutrition and bone metabolism does NOT occur in elderly subjects? a. Decreased intake of protein b. Reduction of skin biosynthesis of vitamin D c. Increased renal excretion of calcium d. Decline in osteoblast function

a. Delayed gastric emptying (CH 27 Krause, p. 509) Nighttime GERD is distinguished from daytime GERD by changes in physiology during sleep. Stomach emptying is delayed, saliva production and swallowing decrease, chyme is exposed to stomach acid for longer periods of time, and other gastrointestinal digestive secretions increase. These changes are partly explained by the supine or recumbent position that a person is in while sleeping. - -

Which of the following contributes to nighttime GERD? a. Delayed gastric emptying b. Increased swallowing c. Decreased digestive secretions d. Increased saliva production

a. Autoantibodies that contribute to the destruction of beta-cells (CH 30 Kruase, p. 587) Immune-mediated diabetes mellitus (DM) is the form of type 1 DM that results from autoimmune destruction of the pancreatic beta-cells. This results in the patient's inability to produce insulin. Idiopathic type 1 DM is the other form, and it is DM that has no known etiology. Insulin resistance and beta-cell failure are what lead to the development of type 2 DM. Diet and sedentary lifestyle can contribute to this development. Gestational diabetes mellitus develops because of an increase in the release of insulin-antagonist hormones during pregnancy. These increased hormone levels contribute to an insulin resistance in the pregnant woman. - -

Which of the following contributes to the development of type 1 DM? a. Autoantibodies that contribute to the destruction of beta-cells b. Insulin resistance and beta-cell failure c. Increase in insulin-antagonist hormone levels d. Diet and sedentary lifestyle

d. All of the above (CH 30 Krause, p. 586) The American Diabetes Association's standard for the diagnosis of diabetes mellitus is based on measurement of the plasma glucose level. Recently, the glycosylated hemoglobin has been added to diagnostic methods. - -

Which of the following criteria is NOT appropriate for the diagnosis of diabetes mellitus? a. Fasting plasma glucose of greater than 126 mg/dl b. HbA1c of greater than 7.0% c. Postload (2 hour) plasma glucose of 200 mg/dl or greater d. All of the above

Vitamin B12 (CH 7 Krause, pg 105) Vitamin B12 and folate deficiencies both may promote the development of macrocytic anemia because they are involved in DNA synthesis, and loss of either results in the impaired production of red blood cells. Homocysteine is a product of methionine metabolism. Levels of homocysteine increase when vitamin B12 and folate are deficient because these are necessary for its conversion back to methionine. Vitamin B6 affects homocysteine by converting it to cysteine. However, neither of these promote anemia. Iron is associated with microcytic anemia. -

Which of the following deficiencies could cause macrocytic anemia? a. Vitamin B12 b. Vitamin B6 c. Homocysteine d. Iron

c. Infants double their birth weight by 4 to 6 months of age. (CH 16 Krause, pg 300) Infants usually double their birth weight by 4 to 6 months of age and triple their weight by 1 year. Infants usually lose about 6% of their birth weight during the first few days postpartum, but this weight is usually regained by 7 to 10 days postpartum. -

Which of the following describes the growth expectations for most infants? a. The birth weight is usually regained by the end of the first month. b. Infants triple their birth weight by 6 months of age. c. Infants double their birth weight by 4 to 6 months of age. d. Infants triple their length by 3 years of age.

b. Decreasing intake of liquids and simple sugars with meals (CH 27 Krause, p. 521) Dumping syndrome is a common problem that occurs after a gastrectomy with a vagotomy. Dietary management for dumping syndrome focuses on limiting hypertonic simple sugars, which can draw fluids into the gastrointestinal system and produce the side effects of the dumping syndrome. Because of the smaller stomach capacity and impaired control of gastric emptying, the intake of fluids with meals could promote rapid gastric emptying. A high-protein, moderate-fat diet including fibrous foods and complex carbohydrates is recommended. Patients may experience lactose intolerance after these surgeries; therefore, milk and dairy portions should be limited in quantity. - -

Which of the following dietary modifications minimize the problems associated with a gastrectomy with a vagotomy? a. Increasing carbohydrate intake, particularly simple sugars, and decreasing fat and protein b. Decreasing intake of liquids and simple sugars with meals c. Reducing the intake of fiber from fruits and vegetables d. Increasing the intake of milk at mealtimes and between meals

d. Excessive potassium (CH 24 Krause, pp.462-465) Bone mass is maintained by adequate vitamin D and calcium intake and adequate phosphorus to maintain the serum calcium-to-phosphate ratio. Too much phosphorus in relation to calcium stimulates parathyroid hormone, resulting in loss of bone. Oral doses of potassium bicarbonate by postmenopausal women neutralize endogenous acids that may contribute to bone resorption. - -

Which of the following dietary variables is NOT related to low bone mass? a. Inadequate calcium b. Excessive phosphorus c. Inadequate vitamin D d. Excessive potassium

d. Zone Diet (CH 21 Krause, pg 395) The Zone Diet and the South Beach Diet are two diets that promote an intake of carbohydrate to less than 40% of total calories. The claim behind this is that this level of carbohydrate intake will result in reduced insulin secretion. Insulin is responsible for fat storage, and if less is secreted, less fat deposition will occur. Typical low-carbohydrate diets tend to have levels of 20% of total calories or less. A protein-sparing modified diet is a type of very low-calorie diet (VLCD) that provides protein but limits calories. The idea behind this is that if protein were provided by the diet, lean body tissue could be preserved while the patient lost weight. VLCDs provide the patient with only 200 to 800 calories/day and should be conducted under the care of a physician. - -

Which of the following diets advocates that carbohydrates should contribute no more than 40% of the total calories? a. Low-carbohydrate diet b. Protein-sparing modified diet c. Very low-calorie diet d. Zone Diet

fat (CH 2, Krause) pg 19 Although dietary fat provides the highest concentration of energy, metabolism of fat is highly efficient, with only 4% of calories wasted. This partly explains the obesogenic aspect of dietary fat. The TEF after intake of carbohydrates and proteins tends to be higher than after fat intake. Following a regular eating schedule results in a higher TEF than irregular eating. The use of spice and mustard increases metabolism more than unspiced -

Which of the following does NOT increase the thermic effect of food (TEF)? a. Carbohydrates b. Fat c. Regular eating schedule d. Spicy foods

d. A patient with BMI of 40 or higher (CH 21 Kruase, pg 398) Bariatric surgery is an acceptable treatment for people with extreme obesity. A person with a BMI greater than 40 or person with a BMI greater than 35 with comorbidities would be an ideal candidate. Combinations of diet therapy, exercise, and lifestyle modification should be attempted before surgical intervention is sought. - -

Which of the following does the NIH identify as being an appropriate candidate for bariatric surgery? a. A person with metabolic syndrome b. A patient with BMI of 25 to 29 with high blood pressure c. A patient with BMI of 30 to 34 d. A patient with BMI of 40 or higher

Metronidazole (Flagyl) (CH 8 Krause pg 131) Metronidazole (Flagyl), cefoperazone, and chlorpropamide cause a reaction similar to that of Antabuse when taken with alcohol. The reaction is characterized by flushing, a rapid heartbeat, palpitations, and elevated blood pressure. -

Which of the following drugs will produced an Antabuse (disulfiram)-like reaction when taken with ethanol? a. Metronidazole (Flagyl) b. Captopril (Capoten) c. Paclitaxel (Taxol) d. Eszopiclone (Lunesta)

d. Erythropoietin (EPO) (CH 23 Krause, p. 450) EPO is a hormone produced by the body that stimulates red blood cell production. This can result in an increase in serum hematocrit and increased oxygen-carrying capacity. Iron supplements may improve iron stores but have not been shown to enhance performance. Beta-HMB and steroids are generally taken as means of increasing muscle mass. Steroids can have an anabolic effect in regard to increasing blood cell production. - -

Which of the following ergogenic aids is used to increase the oxygen-carrying capacity to increase VO2 max and endurance? a. Beta-hydroxy-beta-methylbutyrate (HMB) b. Iron supplements c. Steroids d. Erythropoietin (EPO)

c. A White woman who is underweight, had premature menopause, does not exercise, and smokes cigarettes (CH 24 Krause, p. 459) Risk factors for the development of osteoporosis include being a woman of European or Asian descent, being underweight or sarcopenic, having experienced menopause or oophorectomy, lack of exercise, cigarette smoking, excessive alcohol or caffeine use, and limited intake of calcium and vitamin D. - -

Which of the following female patients is at risk for osteoporosis? a. An African American woman who was normal age at menopause and has a large frame and poor intake of calcium. b. A White woman who is overweight, had multiple pregnancies and late menopause, and consumes alcohol. c. A White woman who is underweight, had premature menopause, does not exercise, and smokes cigarettes. d. An Asian woman who is premenopausal and has a large frame, a high calcium intake, and a sedentary lifestyle.

d. Crackers and pretzels (CH 25 Kruase, p. 473) they are sources of fermentable carbohydrates. Crackers and pretzels provide these types of carbohydrates that can stick to the teeth. Raw carrots and apples have dietary fiber, which tends to be cariostatic. Cheddar cheese is anticariogenic because of its effect on oral pH. Butter is cariostatic. - -

Which of the following foods would be identified to a patient as being potentially cariogenic? a. Raw carrots and apples b. Cheddar cheese c. Butter d. Crackers and pretzels

a. Dried beans, dried fruits, and shellfish (CH 29 Krause, p. 567) Foods high in copper include shellfish, pork, game birds, soy protein, organ meats, avocado, chocolate, dried beans, wheat germ, sweet potatoes, mushrooms, nectarines, dried fruits, and Brewer's yeast. - -

Which of the following foods would be recommended to be avoided by a patient with Wilson disease, in which excretion of copper from the body becomes impaired? a. Dried beans, dried fruits, and shellfish b. Eggs, milk, and cheese c. Sweet potato, rice, and oatmeal d. Butter, cream, oil, and salad dressing

a. Research has not linked a dietary cause to autism. (CH 17 Krause, pg 326-327) Although a number of dietary factors have been investigated, research has not demonstrated these to be the cause of autism or to be helpful in the treatment of autism. -

Which of the following has been demonstrated by research to be TRUE regarding diet and autism spectrum disorder? a. Research has not linked a dietary cause to autism. b. Autism results from a food allergy. c. Gluten in the diet leads to autism. d. Fluoride in the water has been linked to autism.

a. Milk thistle (CH 29 Krause, p. 574) Milk thistle is popular among those with viral hepatitis and alcoholic liver disease. The active component is silymarin. It is proposed to reduce free radical production and lipid peroxidation. Centella asiatica, black cohosh, and Kava have been implicated in liver injury; there is insufficient evidence to support their use. - -

Which of the following herbal supplements is popular for treating liver disease? a. Milk thistle b. Centella asiatica c. Ephedra d. Kava

advance directives (CH 10 Krause, pg 172) Advance directives are legal guidelines established by a patient that designate personal health care intervention preferences or identify a person to make decisions when the patient does not have the capacity to do so. -

Which of the following identifies a patient's health care preferences in regard to end-of-life issues? a. Discharge planning b. Disease management c. Case management d. Advance directives

critical pathway (Ch 10 Krause, pg 168)- A critical pathway is used by health care providers to identify what activities should occur at times throughout a patient's care. This is a key component of case management, which focuses on the achievement of patient-focused goals in the delivery of health care. -

Which of the following identifies the essential elements and activities that should occur in a patient's care? a. Case management b. Critical pathway c. Utilization review d. DRG

c. Decrease in dietary calcium intake (CH 24 Krause, pp. 459-460) Although decreased dietary calcium intake is a risk factor for osteoporosis, it is not specific to either type of osteoporosis. Type I osteoporosis in women results after menopause when estrogen production ceases. The BMD of women with postmenopausal osteoporosis may fall as low as 25% to 40% of that of age-matched women without osteoporosis. Type I osteoporosis is associated with "crush" fractures of the wrists and lumbar vertebrae. - -

Which of the following is NOT a characteristic of hormone-related (type I) osteoporosis? a. Rapid rate of bone loss and therefore of BMD b. Cessation of ovarian estrogen production c. Decrease in dietary calcium intake d. Increased risk of fracture of wrist and vertebrae

serum glucose (Ch 7 Krause pg 103) Although high serum glucose levels in people with diabetes mellitus have a dehydrating effect, by itself, it is not a measure of hydration. Glucose, sodium, and blood urea nitrogen (BUN) are all factors in the calculation of serum osmolality. Serum osmolality, serum sodium, BUN, and urine specific gravity are all laboratory values that may be used to assess hydration status. -

Which of the following is NOT a laboratory measure of hydration status (factors the serum osmolality)? a. Serum sodium b. Blood urea nitrogen c. Serum glucose d. Urine specific gravity

d. Peripheral vascular disease (CH 30 Krause, p. 614) Microvascular diseases are ones that affect smaller blood vessels and nerves. Retinopathy, neuropathy, and nephropathy have been the three primary microvascular diseases that develop from uncontrolled diabetes mellitus. Peripheral vascular disease is classified as a macrovascular disease because it involves larger blood vessels. - -

Which of the following is NOT a microvascular disease associated with hyperglycemic patients? a. Retinopathy b. Neuropathy c. Nephropathy d. Peripheral vascular disease

Hypotension (Ch 8 Krause, pg 130) Because corticosteroids such as prednisone promote sodium and water retention, they can contribute to an increase in blood pressure. Calcium is excreted in the place of sodium, and this coupled with corticosteroid impairment of intestinal calcium absorption and osteoblast activity can lead to bone loss. Corticosteroids promote gluconeogenesis and insulin resistance, both of which can cause hyperglycemia. Weight gain is a common side effect of corticosteroids because of both fluid retention and a stimulation of appetite. -

Which of the following is NOT a possible side effect of use of corticosteroids? a. Bone loss b. Hyperglycemia c. Hypotension d. Obesity

d. Blood vessel damage (CH 30 Krause, p. 613) Blood vessel damage may be either microvascular or macrovascular in nature; however, this occurs because of long-term exposure of the blood vessels to the physiologic changes that commonly occur with diabetes, such as hyperglycemia, hyperlipidemia, and hypertension. Hypoglycemia, hyperglycemia, and ketoacidosis are acute complications of diabetes mellitus because each of these can occur rapidly in relation to alterations in the patient's lifestyle, health, or use of medications. - -

Which of the following is NOT a potential acute complication of type 1 DM? a. Hypoglycemia b. Hyperglycemia c. Ketoacidosis d. Blood vessel damage

d. High blood pressure (CH 24 Krause, pp. 462-3) Increased calcium intake has been associated with lowered blood pressure. In susceptible individuals, excessive calcium supplementation may result in urinary tract stones. Excessive calcium supplementation may cause constipation and can interfere with the absorption of other divalent cations. - -

Which of the following is NOT a risk associated with excessive calcium supplementation? a. Constipation b. Iron deficiency c. Urinary tract stones d. High blood pressure

b. Motivation (CH 14 Krause, pg 228) The transtheoretical model, or stages of change, includes the stages of precontemplation, contemplation, preparation, action, maintenance, and relapse. Motivation may be applied as a technique of interviewing to achieve a willingness to change in the client. -

Which of the following is NOT a stage of change? a. Precontemplation b. Motivation c. Relapse d. Action

c. Reframing (CH 14 Krause, pg 234) The six steps that are important when working with clients struggling with behavior change are expressing empathy, understanding cultural factors, developing discrepancy, avoiding arguments and defensiveness, rolling with resistance, and supporting self-efficacy. Reframing is a technique in which the counselor changes the client's interpretation of data by offering a new perspective. This is used when a client is demonstrating resistance to change. -

Which of the following is NOT an activity that facilitates behavior change? a. Self-efficacy b. Empathy c. Reframing d. Discrepancy

a. Infant feeds from both breasts during a feeding. (CH 15 Krause, pg 289) When an infant feeds from both breasts during a feeding, it is an indication of the volume of milk the mother is producing between feedings. Usual signs for adequate infant growth are the weight and length gain and the wet diaper and stool frequency. -

Which of the following is NOT an indication of adequate infant growth from breastfeeding? a. Infant feeds from both breasts during a feeding. b. Infant has at least six to eight wet diapers per day. c. Infant has frequent stools. d. Infant continues to gain weight and length steadily.

Malnutrition (CH 7 Krause pg 104) Prealbumin levels may appear normal when a patient has uncomplicated or severe malnutrition. Prealbumin levels decrease in response to inflammation, protein-wasting diseases of the intestines and kidney, and in malignancy. As zinc is necessary in the synthesis of prealbumin, zinc deficiency will result in decreased levels -

Which of the following is NOT associated with a decrease in prealbumin levels? a. Malnutrition b. Inflammation c. Protein-wasting disease of the gastrointestinal tract d. Zinc deficiency

c. Being underweight (CH 27 Krause, p. 515) Dyspepsia refers to nonspecific, persistent upper abdominal discomfort or pain. The discomfort may be related to organic causes such as esophageal reflux, gastritis, peptic ulcer, gallbladder disease, or other identifiable pathology. Diet, stress, and other lifestyle factors may contribute to the symptoms. An underlying mechanism may be altered gastric motility. - -

Which of the following is NOT associated with dyspepsia? a. Diet b. Abnormal gastric emptying c. Being underweight d. Upper abdominal discomfort

d. Bringing lunch from home (CH 17 Krause, pg 323) Although lunches brought from home tend to provide fewer nutrients and less variety than those served in school, they do provide school age children with some nutritional value. Children in poverty tend to be more likely to experience iron deficiency, food insecurity, and skipping of meals. These each have been demonstrated to result in poor performance in regard to academic skill development. The School Breakfast Program provides a means to ensure that children from families with lower incomes have the opportunity for a nutritious breakfast before the school day. -

Which of the following is NOT associated with impaired academic performance in children? a. Iron deficiency b. Food insecurity c. Skipping breakfast d. Bringing lunch from home

b. Creatine phosphate (CH 23 Kruase, p. 427) The coenzyme called nicotinic acid dehydrogenase (NAD) converts pyruvic acid to lactic acid. Lactic acid is the end product of anaerobic glycolysis. Adenosine diphosphate is phosphorylated to create ATP. - -

Which of the following is NOT involved in the anaerobic production of ATP through the lactic acid pathway? a. Pyruvic acid b. Creatine phosphate c. Nicotinic acid dehydrogenase d. Adenosine diphosphate

d. Cleanses out metabolic end products by promoting diuresis (CH 23 Krause, p. 439) The diuretic effect of caffeine can have a negative effect on the athlete by promoting excessive fluid losses and potential dehydration. High intake of caffeine before events is banned by the International Olympics Committee because of the benefits of conserving glycogen by promoting fatty acid mobilization, enhancing muscle contractility, and reducing fatigue. - -

Which of the following is NOT one of the benefits of caffeine use as an ergogenic aid? a. Enhances fatty acid mobilization b. Facilitates calcium transport in muscle contractility c. Reduces fatigue by reducing plasma potassium accumulation d. Cleanses out metabolic end products by promoting diuresis

Easier to change and update forms used for documentation (CH 10 Krause, pg 166) Customized screens and drop-down menus are easier to incorporate before the EHR is implemented for use in a health care system than later as changes require access to the programming. Many forms included have extraneous options for selection in documentation so as to limit the need for revisions. When EHR is available, the records are available through most of the health care system's computer access sites, thus eliminating the need for waiting for other health care providers to complete using the record. Because all documentation is typed, records are easier to read. -

Which of the following is NOT one of the benefits of electronic health records? a. Easier accessibility for health care team members b. Improved legibility of documentation c. Easier to change and update forms used for documentation d. Increased efficiency in documentation and providing care

c. Increased glucose absorption and insulin response (CH 27 Krause, p. 522) Pectin and guar gums used in the treatment of patients with dumping syndrome provide the benefits of reducing upper GI transit time, slowing down or decreasing the rate of glucose absorption and the glycemic load, and decreasing the insulin response. A potential caution in regard to using pectin or bulk fiber sources is in regard to the development of intestinal obstructions from using large amounts. - -

Which of the following is NOT one of the benefits of using pectin to manage dumping syndrome? a. Reduced upper gastrointestinal tract transit time b. Reduced glycemic load and decreased insulin response c. Increased glucose absorption and insulin response d. Slow carbohydrate absorption

Food security (CH 9 krause pg 145) Although how they are performed varies depending on the community setting, the three core functions of public health are community assessment, policy development, and public health assurance. Food security may be one of the issues addressed in public health but is not a focus in all public health programs. -

Which of the following is NOT one of the core functions of public health? a. Community assessment b. Food security c. Policy development d. Public health assurance

b. Diabetes (CH 20 Krause, pg 374) The four syndromes known to be predictive of adverse outcomes in older adults that are prevalent in patients with frailty (known as failure to thrive) include impaired physical functioning, malnutrition, depression, and cognitive impairment. - -

Which of the following is NOT one of the four syndromes known to be predictive of adverse outcomes in older adults? a. Depression b. Diabetes c. Cognitive impairment d. Malnutrition

a. CAM is not as expensive as conventional medicine. (CH 12 Krause, pg 194) Unlike conventional medicine, most CAM is not generally covered by health insurance. The cost of this care must be borne by the patient, and it is not necessarily cheaper than conventional care. CAM will be an option for people who believe that medical conditions such as cancer or migraines cannot be treated effectively by conventional medicine. Conventional medicine's treatment of back pain or neck pain may focus on surgical interventions that patients will see as having significant risk involved. CAM has been shown to be effective in the treatment of pain and arthritis. -

Which of the following is NOT one of the reasons that there has been an increased usage of CAM in the United States? a. CAM is not as expensive as conventional medicine. b. CAM is used when conventional medicine has little to offer in effective treatment. c. CAM is used when conventional medicine has significant risks and side effects. d. CAM has a history of efficacy in treatment of some conditions.

c. Healthy Eating Index (HEI) (CH 11 Krause, pg 174) The DRI Model includes four reference points—AI, EAR, RDA, and Upper Intake Level (UL). The HEI measures how well people's diets conform to recommended healthy eating patterns. -

Which of the following is NOT part of the Daily Reference Intake (DRI)? a. Adequate Intake (AI) b. Estimated Average Requirement (EAR) c. Healthy Eating Index (HEI) d. Recommended Dietary Allowance (RDA)

a. Eat foods with prebiotics and probiotics. (CH 11 Krause, pg 183) The Universal Prescription for Health and Nutritional Fitness is a compilation of various guidelines that represents an overall pattern of eating. It makes statements that are very widely accepted to be true and does not include prebiotics or probiotics. -

Which of the following is NOT part of the Universal Prescription for Health and Nutritional Fitness? a. Eat foods with prebiotics and probiotics. b. Drink fluoridated water. c. Eat fewer high cholesterol foods. d. Limit or omit high-sodium foods.

b. Advance to full liquid diet. (CH 27 Krause, p. 512) Nissen fundoplication is a surgical treatment for severe reflux esophagitis. The fundus or top portion of the stomach is wrapped 360 degrees around the lower esophagus. Recommendations after surgery are to start with clear liquid and advance to small meals of soft, most foods. Dry, hard foods such as meat, nuts, rolls and bread, and raw fruits should be avoided for 3 to 6 months. A full liquid diet is no longer recommended. - -

Which of the following is NOT true for diet recommendations after Nissen fundoplication? a. Start with clear liquid diet. b. Advance to full liquid diet. c. Consume multiple, small meals. d. Avoid dry, hard foods.

c. They encourage strict dietary control for diabetes and heart disease (CH 20 Krause, pg 378) The Dining Practice Standards were developed by a multidisciplinary group who wanted to improve the dining experience and nutritional care for elders living in long-term care. They were endorsed by the Centers for Medicare and Medicaid. The standards promote person-centered care, liberalized diets, and food first with less reliance on canned supplements. - -

Which of the following is NOT true of The Dining Practice Standards? a. Are guidelines for long-term care dining b. Recommend against canned nutrition supplements c. They encourage strict dietary control for diabetes and heart disease d. They are endorsed by the Centers for Medicare and Medicaid

It marks a point in the care of the patient that will extend their DRG. (CH 10 Krause, pg 163) A sentinel event has nothing to do with DRGs or reimbursement. It is an unanticipated event that involves death or serious physical or psychologic injury. The outcomes of this event must be documented in the medical record. -

Which of the following is NOT true of a sentinel event? a. It is an unanticipated event that involves serious injury or death. b. It marks a point in the care of the patient that will extend their DRG. c. The outcomes of the event must be documented in the medical record. d. It can be psychologic injury.

All of the above. (CH 7 Krause, page 115-116) Bioelectrical impedance analysis (BIA) is a body composition analysis technique based on the principle that, relative to water, lean tissue has a higher electrical conductivity and lower impedance than fatty tissue because of its electrolyte content. BIA has been found to be a reliable measurement of body composition (fat-free mass and fat mass). The equipment needed is portable, and the method has been shown to be safe and noninvasive. For accurate results the patient should be well hydrated; have not exercised in the previous 4 to 6 hours; and have not consumed alcohol, caffeine, or diuretics in the previous 24 hours. If the person is dehydrated, a higher percentage of body fat than really exists is measured. -

Which of the following is TRUE about bioelectric impedance analysis? a. It measures lean body tissue. b. It is noninvasive and portable. c. It is not accurate in a dehydrated individual. d. All of the above.

a. The patient is of normal or near normal weight. (CH 22 Kruase, pg 412) A person with bulimia nervosa is commonly within the normal weight range, although some may be slightly under- or overweight. A person with bulimia nervosa is very concerned about body shape and size. Binge eating, which involves short periods (usually 2 hours or less) of intake of large amounts of food, and purging behaviors are commonly used by a person with bulimia nervosa. - -

Which of the following is a characteristic of bulimia nervosa? a. The patient is of normal or near normal weight. b. The patient binges but does not purge. c. The patient does not have a body image problem. d. A typical binge lasts all day.

Accumulation of acids from abnormal metabolism (Ch 6, Krause) Metabolic acidosis is simply named for the production of acids through metabolic processes. Diabetic ketoacidosis and lactic acidosis are two conditions that lead to metabolic acidosis. Metabolic alkalosis results from an accumulation of bicarbonate. Respiratory acidosis results from retention of carbon dioxide, but respiratory alkalosis is associated with excessive carbon dioxide respiration. REF: p. 95 -

Which of the following is a characteristic of metabolic acidosis? a. Accumulation of bicarbonate b. Decreased ventilation and retention of carbon dioxide c. Accumulation of acids from abnormal metabolism d. Excessive loss of carbon dioxide from the lungs

b. Loss of height occurs(CH 24 Krause, p. 460) Type II osteoporosis, or age-related osteoporosis, usually occurs after 65 years of age and affects both men and women. Type II osteoporosis affects both trabecular and cortical bone. Fractures of the hip and vertebrae are the most common to occur with this type of osteoporosis. Because of the latter, a loss of height, spinal deformity, and dowager's hump occur. - -

Which of the following is a characteristic of type II, or age-related, osteoporosis? a. Occurs after 50 years of age b. Loss of height occurs c. Only affects trabecular bone d. Rarely occurs in males

a. Self-efficacy (CH 14 Krause, pg 229) Self-efficacy is a goal of counseling as behavior change can only occur if the client believes that he is capable of changing. Self-management is a technique used by a counselor that involves the client's decisions in facilitating change. Self-monitoring refers to any means by which a client records or tracks activity related to behavior change. Self-reflection is when the client is given the opportunity to think about and express thoughts related to behavior change. -

Which of the following is a client's belief in his ability to carry out change? a. Self-efficacy b. Self-management c. Self-monitoring d. Self-reflection

a. Ascites (CH 29 Krause, p 566) Portal hypertension occurs secondary to cirrhosis because of obstruction of blood flow through the liver. With a decreased production of albumin by the liver, the increased pressure within the circulatory system can force fluid into the peritoneal cavity, resulting in ascites development. Cardiac arrhythmias can be affected by changes in the blood pressure, but this is not specific to portal hypertension. Pulmonary complications can result from hepatitis. - -

Which of the following is a common sign of portal hypertension secondary to cirrhosis? a. Ascites b. Cardiac arrhythmias c. Increased gastrointestinal motility and peristalsis d. Pulmonary fibrosis

d. Barrett esophagus (CH 27 Krause, p. 511) Barrett esophagus is partly responsible for the development of adenocarcinoma of the esophagus, although esophageal cancer may develop in the absence of Barrett esophagus. GERD is also a risk factor for esophageal cancer; however, the condition does not necessarily involve the histologic changes seen in Barrett esophagus. An esophageal stricture may result from a variety of conditions that lead to decreased esophageal tone and narrowing of the esophageal lumen. Hiatal hernia involves an outpouching of the stomach through the diaphragm. - -

Which of the following is a condition involving the cells lining the distal esophagus becoming abnormal and premalignant? a. Gastroesophageal reflux disease b. Esophageal stricture c. Hiatal hernia d. Barrett esophagus

Trypsin, chymotrypsin, and carboxypeptidase (Ch 1, Krause) pg 8 Trypsin, chymotrypsin, and carboxypeptidase are three protein digestive enzymes secreted by the pancreas. Insulin is an endogenous hormone secreted by the pancreas. Secretin is a hormone secreted by the small intestine. Lactase and isomaltase (also known as dextrinase) are brush-border enzymes. Pepsin, which is a protease, and gastrin are hormones secreted by the stomach. -

Which of the following is a list of enzymes released from the pancreas? a. Insulin, trypsin, and secretin b. Lactase, isomaltase, and dextrinase c. Protease, pepsin, and gastrin d. Trypsin, chymotrypsin, and carboxypeptidase

ANS: B Ch 7 Krause The percentage of glycosolated hemoglobin (hemoglobin A1C) in the blood is directly related to the average blood glucose levels for the preceding 2 to 3 months. Homocysteine and hs-CRP are two inflammatory markers, higher levels of which are associated with increased risk of cardiovascular disease. Apolipoprotein B is the part of the protein present in low-density lipoprotein, and it is also associated with increased risk of atherogenesis, possibly through its susceptibility to oxidation by ROS and lipid peroxides. REF: p. 109 - -

Which of the following is a measure of glucose control? a. Highly sensitive C-reactive protein b. Hemoglobin A1C c. Homocysteine d. Apolipoprotein B

ANS: B Ferritin is the storage protein that sequesters the iron gathered in the liver, spleen, and marrow. Total-iron binding capacity is a direct measure of all proteins available to bind iron dependent on the number of free iron-binding sites on transferrin. Transferrin is the primary blood transport protein of iron. Hemoglobin is the oxygen-carrying protein in red blood cells and uses iron as a functional component. REF: p. 105 Ch 7 Krause - -

Which of the following is a measure of iron storage? a. TIBC b. Serum ferritin c. Transferrin d. Hemoglobin

Urinary creatinine (Ch 7 Krause pg 108) Urinary excretion of creatinine is related to the skeletal muscle or somatic protein; however, as the value can be affected by the intake of muscle meats, use of this measure is more limited to research. C-reactive protein is used as an indicator of inflammation. Retinol-binding protein may be used as an indicator of protein calorie malnutrition. Urinary methylmalonic acid is a sensitive indicator of vitamin B12 deficiency. -

Which of the following is a measure of somatic protein status? a. C-reactive protein b. Retinol-binding protein c. Urinary methylmalonic acid d. Urinary creatinine

c. Tanner stages (CH 18 Krause, pg 332-333) Tanner stages are used to determine degree of sexual maturation during puberty. "Periods of adolescence" refers to the years in age during which cognitive and emotional development occur. Gynecologic age is a reference to the number of years between menarche and chronologic age. Peak gain velocity refers to linear growth only. -

Which of the following is a method for rating sexual maturation? a. Periods of adolescence b. Gynecologic age c. Tanner stages d. Peak gain velocity

C-reactive protein (CH 7 Krause pg 103) Positive acute-phase respondents include C-reactive protein, alpha-1-antichymotrypsin, alpha-1-antitrypsin, fibrinogen, ferritin, and complement components C3 and C4, just to name a few. Interleukin-1 is one of the cytokines triggered by trauma that reorients hepatic synthesis of plasma proteins. Albumin and transthyretin (prealbumin) are both negative acute-phase proteins. -

Which of the following is a positive acute-phase respondent? a. Interleukin-1 b. Albumin c. Transthyretin d. C-reactive protein

c. Changes in genital size (CH 23 Krause, p. 450) Androgenic effects of steroid use include the development of the secondary sex characteristics of men, such as changes in genital size and function and growth of auxiliary pubic and facial hair. Anabolic effects of steroid use include increases in muscle mass, bone mineral density, and blood cell production; a decrease in body fat; increased size of the heart, liver, and kidneys; changes in the vocal cords; and increased libido. - -

Which of the following is a potential androgenic effect of taking steroids? a. Decreased body fat b. Increased libido c. Changes in genital size d. Vocal cord changes

c. Polyps (CH 28 Krause, p. 549) Colorectal polyps are considered to be precursors of colon cancer. Inflammatory bowel disease is also a risk factor. Diets high in calories, fat, and animal protein and low in fruits, vegetables, and grains are associated with increased risk of colon cancer. - -

Which of the following is a risk factor for developing colon cancer? a. IBS b. Lactase deficiency c. Polyps d. Diverticulitis

a. Brewed tea (CH 25 Krause, p. 472) Most foods, unless prepared with fluoridated water, contain minimal amounts of fluoride, except for brewed tea. - -

Which of the following is a significant food source of fluoride? a. Brewed tea b. Beef c. Coffee d. Spinach

DRG (Ch 10 Krause, pg 169) Utilization management uses established standards to identify tests, procedures, and services in health care that have known benefits toward patient improvement. The utilization manager reviews the case management of a patient to ensure that only approved activities that will help the patient are being performed. Patient-focused care is organized around the concept that the patient is the focus of the health care team, and the patient ultimately makes the decisions in regard to health care. Case management is the overall management organized around the achievement of patient health care goals. DRG is a system of paying for the care based on the diagnosis of the patient. -

Which of the following is a system of paying for the care of a patient based on their diagnosis? a. Utilization management b. Patient-focused care c. Case management d. DRG

Stop the feeding and irrigate the tube with water before and after administering the medication. (CH 8 Krause page 127) Because of the potential for physical incompatibilities that result in the clogging of a feeding tube, medications should not be mixed directly with feeding formula. Protocols should be established in regard to which medications may be administered through a feeding tube. In these protocols, specification should be included in regard to stopping the feeding administration, irrigating the feeding tube to ensure patency, administering the medication, and irrigating the tube again with water. Sudafed and Robitussin are two medications that could lead to clogging of the feeding tube. -

Which of the following is an appropriate procedure for administering oral medications through a feeding tube? a. Stop the feeding and irrigate the tube with water before and after administering the medication. b. Mix the medications with water and add them to the feeding formula. c. Crush the medications, mix them with water, and then add them to feeding formula. d. Sudafed cough syrup and Robitussin expectorant may be added directly to the feeding formula.

c. Increase in LDL levels (CH 19 Krause, pg 358) Estrogen has a protective effect in regard to bone and cardiovascular health. When estrogen circulation decreases, an increase is seen in LDL levels. Estrogen plays a role in bone remodeling, and with loss of estrogen, a decrease in bone mass occurs. Decreased estrogen is associated with a decrease in HDL. Symptoms of menopause include a decrease in energy level. -

Which of the following is an effect of the decreased circulation of estrogen associated with menopause? a. Increase in bone remodeling b. Increase in HDL levels c. Increase in LDL levels d. Increase in energy levels

a. Trabecular bone (CH 24 Kruase, p. 456) Trabecular bone is the spongy bone that is found in the knobby ends of the long bones. Cortical bone, which consists of osteons or Haversian systems, is found in the shaft regions of the long bones - -

Which of the following is associated with the bone tissue at the end of the long bones? a. Trabecular bone b. Cortical bone c. Haversian systems d. Osteons

c. Choledocholithiasis (CH 29 Krause, p. 577) Choledocholithiasis occurs when stones slip from the gallbladder into the bile ducts. Cholangitis or inflammation of the bile ducts can occur as a result of choledocholithiasis. Cholecystitis is an inflammation of the gallbladder, which can also be caused by gallstones. Cirrhosis is the final stage of liver deterioration, which involves the scarring and formation of fibrous tissue that interferes with the liver's normal structure. - -

Which of the following is characterized by the presence of gallstones in the common bile duct? a. Cholangitis b. Cholecystitis c. Choledocholithiasis d. Cirrhosis

a. Vitamin K (CH 24 Kruase, p. 464) Vitamin K is involved in the posttranslational carboxylation process or maturation of osteocalcin. Vitamin D's role in bone formation is in regard to the adequate absorption of calcium from the gastrointestinal tract as well as the stimulation of osteoblast activity. Adequate vitamin A intake is necessary for the promotion of bone growth and maintenance. Vitamin E does not have a direct role in bone formation. - -

Which of the following is necessary for the maturation of osteocalcin? a. Vitamin K b. Vitamin D c. Vitamin E d. Vitamin A

c. Muscle wasting (CH 22 Krause, pg. 410) Loss of lean body mass and muscle wasting are common physical attributes in individuals with anorexia nervosa. Enamel wasting, esophagitis, and Russell sign (scarring of the dorsum of the hand) are signs that a person uses vomiting as a purging behavior. Although some people with anorexia nervosa may be of the binge eating and purging type, these signs are more commonly seen in people with bulimia nervosa. - -

Which of the following is often exhibited in patients with anorexia nervosa? a. Enamel wasting b. Esophagitis c. Muscle wasting d. Russell sign

Primary prevention (CH 9 Krause, pg 139) The traditional public health approach to health care is through health promotion at the primary prevention level. Secondary prevention focuses on risk appraisal and reduction before development of disease. Tertiary prevention focuses on treatment and rehabilitation to prevent the progression of disease or to reduce the debilitating consequences of the disease. -

Which of the following is the focus of the traditional public health approach to health care? a. Primary prevention b. Secondary prevention c. Tertiary prevention d. Patient care

c. Paracentesis (CH 29 Krause, p 571) Paracentesis is a procedure used to treat ascites that involves removing fluid from the abdominal cavity with a needle. Radiologic or surgical placement of a shunt may be used to treat esophageal varices. The Whipple procedure is a surgical pancreaticoduodenectomy performed in the treatment of pancreatic cancer. Shock-wave lithotripsy is a method of breaking apart gallstones through use of sonic waves. - -

Which of the following is used in the medical treatment of ascites? a. Shunt placement b. Whipple procedure c. Paracentesis d. Shock-wave lithotripsy

d. Serum amylase (CH 29 Krause, p. 580) In pancreatitis, the serum amylase level may appear normal. Ranson criteria include an increase in the white blood cell count and the blood glucose level caused by the inflammatory process occurring in pancreatitis, which could impair insulin production and secretion. A decrease in serum calcium is also one of the Ranson criteria caused by possible hypoalbuminemia and third spacing of fluid. - -

Which of the following laboratory measures is NOT included in Ranson criteria to classify severity of pancreatitis? a. WBC b. Blood glucose c. Serum calcium d. Serum amylase

microcytic anemia (CH 7 Krause page 105) Microcytic anemia is mostly associated with iron deficiency. Macrocytic anemia, which would be reflected by a high MCV value, is usually caused by either folate or vitamin B12 deficiency. Reticulocytes are large, nucleated, immature red blood cells that are released in small numbers with mature red blood cells. The presence of these may indicate erythropoiesis in response to blood loss, hemolysis, or iron, folate, or vitamin B12 therapies. -

Which of the following manifestations will occur in iron deficiency anemia? a. Microcytic anemia b. Macrocytic anemia c. High reticulocyte count d. High MCV

b. Pancreatic enzyme replacements (CH 29 Krause, p. 580) Pancreatic enzyme replacement therapy provides dosages of lipase that are consumed at meal times. This allows for the lipase to aid in the digestion of fat in the diet to promote improved absorption, thus reducing steatorrhea. Fat-soluble vitamins are supplemented in water-soluble form to ensure against potential deficiencies. Because pancreatic bicarbonate secretion is impaired, antacids and proton pump inhibitors may be provided to reduce or neutralize stomach acids to restore the optimal pH in the duodenum for digestive enzyme actions. - -

Which of the following may be provided to a patient with chronic pancreatitis to minimize steatorrhea? a. Water-soluble form of fat-soluble vitamins b. Pancreatic enzyme replacements c. Antacids d. Proton pump inhibitors

c. Scheduling feeding time right after play or activity. (CH 17 Krause, pg 322) Preschoolers tend to not eat well when they are tired, so a short rest or quiet activity should be scheduled before feeding time. Young children eat better with reduced size utensils that are easier for them to hold. Young children avoid foods with temperature extremes, and they tend to not want to eat mixed foods or foods that touch each other on the plate. -

Which of the following may cause a preschool child to eat less food at a meal? a. Providing the child with a short-handled spoon or fork. b. Providing food at a warm temperature, not too hot or cold. c. Scheduling feeding time right after play or activity. d. Serving foods so they do not touch one another on the plate.

Genomic imprinting (ch 5, Krause) ANS: A Genomic imprinting refers to the process by which a change in the DNA results in a phenotype influenced by whether a gene variant was inherited from the mother or the father. Mutations or gene variants are changes in the nucleotide sequence. A single nucleotide polymorphism results in a change in the nucleotide sequence, but by only one nucleotide. REF: p. 73 -

Which of the following may promote a change in a DNA molecule affecting gene expression without changing the nucleotide sequence? a. Genomic imprinting b. Mutation c. SNP d. Gene variant

d. Odynophagia (CH 25 Krause, p. 476) Oral candidiasis is a fungal infection that causes a burning sensation in the mouth that can lead to pain during chewing, sucking, or swallowing (odynophagia). Tooth loss results from poor oral care and trauma to the mouth. Dry mouth can be caused by use of medications, cancer treatments, and different medical conditions. Ulcers may form in the mouth as a result of viral infections such as herpes simplex or cytomegalovirus. - -

Which of the following may result from oral candidiasis? a. Tooth loss b. Dry mouth c. Ulcer formation d. Odynophagia

Oxandrolone (Oxandrin) (Ch 8 Krause, pg 134) Commonly used appetite stimulation medications include oxandrolone, megestrol acetate, somatropin, and dronabinol. Dextroamphetamine, sibutramine, and phentermine are appetite suppressant medications. -

Which of the following medications could be given to a patient with cancer or AIDS to stimulate food intake? a. Dextroamphetamine (Dexedrine) b. Sibutramine (Meridia) c. Phentermine (Adipex-P) d. Oxandrolone (Oxandrin)

Digoxin (Lanoxin) (Ch 8 Krause ph 127) The phytates in bran bind with the drug digoxin leading to a loss of therapeutic effect. Various food components impair the absorption of alendronate, iron supplements, and ciprofloxacin. REF: p. 127 -

Which of the following medications should not be taken with foods that contain bran? a. Alendronate (Fosamax) b. Iron supplements c. Ciprofloxacin (Cipro) d. Digoxin (Lanoxin)

c. Double-blind, placebo-controlled food challenge (CH 26 Kruase, p 490) The DBPCFC is considered to be the gold standard for establishing a food and symptom relationship. The DBPCFC provides objective results because both the patient and physician are unable to detect differences between placebo foods and foods with the allergen of interest. The physician is on hand to monitor for allergic reactions. A single-blind food challenge may be useful because the person receiving the challenge does not know what is being offered. An open challenge is used as a follow-up to a DBPCFC that has negative results. An elimination diet may be used for diagnosis but takes extensive time to complete, taking weeks to complete instead of hours. - -

Which of the following methods is effective for diagnosis when symptoms are subjective or when multiple food allergies are suspected? a. Open food challenge b. Single-blind food challenge c. Double-blind, placebo-controlled food challenge d. Elimination diet

b. Acetyl CoA (CH 23 Krause, p. 427) Beta oxidation of fatty acids produces acetyl CoA, which enters the mitochondria and goes through the Krebs cycle to produce ATP. Carbon dioxide is one of the end products of energy metabolism. Myoglobin acts within the muscle as an oxygen acceptor to hold a supply of oxygen readily available for use by the mitochondria. Lactic acid is a product of anaerobic glycolysis. - -

Which of the following molecules is required in mitochondria of muscle cells to use fatty acids as energy resources? a. Carbon dioxide b. Acetyl CoA c. Myoglobin d. Lactic acid

a. Iron (CH 19 Krause, pg 358-359) Unless men are diagnosed with iron deficiency anemia, they should avoid intake of additional iron. Excessive iron intake is problematic because it is an oxidant in the body. Lycopene is an antioxidant found in tomato products, pink grapefruit, and watermelon, and increased intake of lycopene has been reported as reducing the risk of prostate cancer. Anthocyanin is a flavonoid that has antioxidant properties, particularly in regard to reducing oxidation of LDL. Vitamin D intakes have been found to be deficient in much of the population, and supplementation is not considered to be a risk. -

Which of the following nutrients should men avoid taking as supplements during their adult years? a. Iron b. Anthocyanin c. Lycopene d. Vitamin D

Negative acute-phase respondents decrease. (Ch 7, Kraues page 103) Negative acute-phase proteins are those that are negatively affected by trauma, meaning that their levels and production decrease in response to the trauma. These include albumin, transferrin, prealbumin, and retinol-binding protein. Positive acute-phase respondents are those that increase in levels and production in response to trauma. -

Which of the following occurs during acute illness or trauma? a. Negative acute-phase respondents increase. b. Positive acute-phase respondents decrease. c. Negative acute-phase respondents decrease. d. Both positive and negative acute-phase respondents increase.

a. Change in weight (CH 29 Krause, p. 571) Subjective Global Assessment is a reliable and valid technique for assessing a patient's nutritional status without relying on biochemical measures or tests. The patient completes a short history regarding changes in weight, appetite, taste, intake, and gastrointestinal problems. A medical practitioner evaluates the patient physically for muscle wasting, fat stores, and the presence of edema or ascites. Then with knowledge of the patient's medical condition, a nutritional rating is assigned to the patient regarding whether the patient is well nourished, moderately malnourished, or severely malnourished. - -

Which of the following parameters is included in Subjective Global Assessment? a. Change in weight b. Nitrogen balance c. Fluid intake and output d. Liver enzyme levels

a. Salmonella spp. (CH 9 Krause, pg 152) The seven foodborne pathogens identified by the CDC as potential bioterrorist weapons are tularemia, brucellosis, Clostridium botulinum toxin, epsilon toxin of Clostridium perfringens, Salmonella spp., E. coli, and Shigella spp. -

Which of the following pathogens has been identified by the CDC as potentially usable by bioterrorists? a. Salmonella spp. b. Staphylococcus aureus c. Bacillus cereus d. Listeria monocytogenes

b. Gluconeogenesis (CH 23 Krause, p. 432) Gluconeogenesis occurs in the liver, and although glucose produced by this pathway may travel to the muscle, energy generation in the muscle itself is not dependent on gluconeogenesis. During anaerobic conditions, glycogenolysis breaks down muscle glycogen stores to provide glucose for glycolysis. During aerobic conditions, glycolysis can still occur. Beta oxidation of fatty acids becomes a primary energy source for muscle cells as activity continues. - -

Which of the following pathways is NOT used to generate energy in muscle cells during any type of exercise? a. Glycogenolysis b. Gluconeogenesis c. Glycolysis d. Beta oxidation

a. 40-year-old man with a jejunal resection (CH 28 Krause, p. 550) Although most patients with significant bowel resection need parenteral nutrition initially to restore and maintain nutritional status, a patient with a jejunal resection adapts to enteral nutrition and oral diet more quickly after surgery because the ileum can adapt to perform the functions of the jejunum. Risk factors for dependence on long-term parenteral nutrition include elderly age, extensive removal of the ileum, loss of the ileocecal valve, and residual disease in the bowel (e.g., radiation enteritis). - -

Which of the following patients with small bowel resection is the LEAST likely to need long-term parenteral nutrition? a. 40-year-old man with a jejunal resection b. 20-year-old woman who had radiation enteritis before resection c. 60-year-old woman with an almost total ileal resection d. 50-year-old woman with the ileocecal valve removed

d. Increased protein (CH 29 Krause, p. 575) In the acute postliver transplant phase, nutrient needs are increased to promote healing, deter infection, provide energy for recovery, and replenish depleted body stores. Nitrogen requirements are elevated. - -

Which of the following recommendations should be included in the nutrition care plan for a patient in the acute post liver transplant phase? a. Calorie restriction b. Increased sodium c. Decreased protein d. Increased protein

c. 20% of the adult height and 50% of the adult weight (CH 18 Krause, pg 332) Although there is great variability in timing of growth among adolescents, the average adolescent gains 20% of his or her adult height during puberty and 40% to 50% of adult body weight. The gain in weight accompanies the gain in height; however, weight gain tends to continue after linear growth has stopped. -

Which of the following represents the growth experienced by a child during adolescence? a. 50% of the adult height and 20% of the adult weight b. 30% of the adult height and 30% of the adult weight c. 20% of the adult height and 50% of the adult weight d. 10% of the adult height and 20% of the adult weight

Increased urinary calcium excretion (CH 6, Krause) Persistent excessive sodium intake has been associated with the development of hypertension and increased calcium excretion. Hypotension does not occur as result of decreased sodium intake. Muscle cramping is more associated with imbalances in calcium and magnesium intake. Increased urinary output of potassium is a common side effect of the use of loop diuretics for the control of hypertension and fluid volume. REF: p. 91 -

Which of the following results from ingesting a large amount of sodium in a short time? a. Hypotension b. Muscular cramps c. Increased urinary calcium excretion d. Increased urinary output of potassium

b. An initial rapid weight loss from diuresis (CH 21 Krause, 394) More than half of the weight loss associated with fasting is caused by loss of fluid and loss of lean body tissue. Because of this, fasting is seldom prescribed for weight loss. Excessive weight gain after refeeding can occur when weight loss is associated with a decrease in metabolic rate from the body adapting to starvation. - -

Which of the following results from the use of fasting as a means of weight loss? a. Excessive weight gain after refeeding b. An initial rapid weight loss from diuresis c. A decrease in ketone formation d. Carbohydrate craving

b. Short stature (CH 17 Krause, pg 317) Zinc is necessary for growth, and improving zinc intake has been beneficial in treating stunted growth and underweight in children. Children develop increased fat stores by taking in an excess of calories compared with energy needs. Chronic diarrhea may be associated with excess intake of fruit juices. Mental retardation may result from a number of nutrient deficiencies and from lead toxicity. -

Which of the following results from zinc deficiency in a child's diet? a. Increased fat stores b. Short stature c. Chronic diarrhea d. Mental retardation

c. Drink at least 2 L of water each day. (CH 28 Krause, p. 528) A high-fiber diet of up to 25 g/day for women and 38 g/day for men should be eaten with the intake of 2 L or quarts of water each day to promote effectiveness of the diet. Intake of this amount entails the consumption of whole-grain bread and cereal products, vegetables, legumes, fruits, nuts, and seeds. These foods provide additional iron and vitamin C. - -

Which of the following should be included in the advice given to a patient regarding eating a high-fiber diet? a. Increase polyunsaturated fat intake. b. Make sure to get more vitamin C. c. Drink at least 2 L of water each day. d. Make sure to get more iron.

c. Most elderly live in nursing homes. (CH 20 Krause, pg 378) Most of today's elderly adults (95%) live someplace other than a nursing home. Women tend to outlive men, so there are more women alive than men at ages 85 years and older. Because of this, there are more women available for men to marry. In 1900, the life expectancy was into the 40s. Now life expectancy is 77 years. - -

Which of the following statements about elderly adults is FALSE? a. Among the oldest old, there are more women than men. b. More elderly men than women are married. c. Most elderly live in nursing homes. d. The life expectancy of older adults is greater than it was 100 years ago.

b. Upper body strength activities may improve overall BMD. (CH 24 Kruase, p. 460) Use of upper body strength activities has been shown to improve the BMD of the femur. Regular weight-bearing exercise seems to have little influence on BMD of older persons but is beneficial during periods of bone accumulation. Walking and swimming similarly do not provide benefit to bone health in elderly adults. - -

Which of the following statements about exercise and bone is TRUE? a. Regular weight-bearing exercise has little influence on BMD. b. Upper body strength activities may improve overall BMD. c. Walking several miles each day has a large impact on hip BMD. d. Swimming regularly has a large impact on vertebral BMD.

b. H. pylori is a viral infection found in the intestine. (CH 27 Krause, p. 515) Helicobacter pylori is a gram-negative bacterium that has been associated with increased risk of atrophic gastritis, peptic ulcer, and gastric cancer. Treatment of an infection usually involves a combination of antibiotics and acid suppression medications - -

Which of the following statements is NOT associated with Helicobacter pylori? a. H. pylori increases the risk of atrophic gastritis and gastric cancer. b. H. pylori is a viral infection found in the intestine. c. H. pylori is a bacterial infection found in the gastric mucosa. d. H. pylori infection may be treated with antibiotics and acid suppression therapies.

a. MCTs are rapidly hydrolyzed and absorbed in the absence of bile acids. (CH 28 Krause, p. 545) MCTs of 8 and 10 carbon length are short enough to be absorbed directly into portal circulation without relying on bile acids for micelle formation or pancreatic lipase for digestion. Because of these benefits, MCTs may be provided to patients experiencing fat malabsorption caused by various conditions. They are provided to the patient in either enteral formula or through the intake of MCT oil. - -

Which of the following statements is TRUE regarding medium-chain triglycerides (MCTs)? a. MCTs are rapidly hydrolyzed and absorbed in the absence of bile acids. b. MCTs require pancreatic lipase for digestion. c. MCTs must be administered intravenously in PN. d. MCTs are contraindicated in steatorrhea and other malabsorptive conditions.

c. Fundoplication (CH 27 Krause, pp. 512 -13) In a fundoplication surgery, the fundus of the stomach is wrapped around the lower esophagus to limit reflux. Billroth I, Billroth II, and the roux-en-Y procedure are different gastric surgeries that involve either gastrectomy or gastric partitioning. - -

Which of the following surgeries would be used to treat a patient with GERD? a. Billroth I b. Billroth II c. Fundoplication d. Roux-en-Y procedure

c. Steatorrhea (CH 29 Krause, p. 571) Steatorrhea occurs as a result of inadequate production of pancreatic lipase. This results in malabsorption of fats, leading to excessive fat in the stool. Constipation may result from motility problems with the colon, but this is not affected by pancreatic function. Because vitamin A is a fat-soluble vitamin, deficiency caused by malabsorption is more likely to occur secondary to pancreatitis than a condition of hypervitaminosis. Serum ammonia levels increase as a result of impaired liver function. - -

Which of the following symptoms appears when pancreatitis progresses to where the ability to secrete a sufficient quantity of enzymes is impaired? a. Constipation b. Hypervitaminosis A c. Steatorrhea d. Elevated serum ammonia

c. Establish an action plan. (CH 14 Krause, pg 232) During the ready-to-change session, the goal should be to work with the patient to set goals for change that include a plan of action. Eliciting self-motivational statements is used for the patient in the precontemplation stage to help create awareness of an existing problem in the patient. Negotiation for change occurs in the unsure-about-change session. Developing discrepancy is an activity used by the counselor throughout the counseling process that helps the patient identify the benefits and disadvantages of the behavior change. -

Which of the following techniques is appropriate for use with a patient ready to change behavior? a. Elicit self-motivational statements. b. Negotiate change. c. Establish an action plan. d. Develop discrepancy.

b. Willingness to try new foods (CH 17 Krause, pg 323) Eating with other children affects food attitudes and choices. This can be both negative and positive. On the negative side can be the development of food refusals and behaviors associated with meal time. On the positive side, children who see other children trying different foods will be more willing to try the new foods as well. Children cannot differentiate foods based on nutritional value, so they cannot self-select nutritious foods over nonnutritious foods. Table manners are generally dictated by an adult influence. -

Which of the following tends to be a positive development in childhood eating habits that can result from peer influences? a. Selection of nutritious versus nonnutritious foods b. Willingness to try new foods c. Table manners d. Adequate time to eat

a. Genetic theory (CH 20 Krause, pg 369) Predetermination theories of aging focus on a built-in mechanism that determines when aging and death will occur. The genetic theory explains that life span is determined by heredity, and thus this falls into the predetermination theory category. Another class of theories is the accumulated damage theories. These describe aging as a process in which the body's systems break down over time because of damage. Wear and tear, free radical, and somatic mutation theories identify different ideas behind how this damage occurs. - -

Which of the following theories of aging is a predetermination theory? a. Genetic theory b. Wear and tear theory c. Free radical theory d. Somatic mutation theory

d. Cross-link theory (CH 20 Krause, pg 369) Accumulated damage theories describe aging as a process in which the body's systems break down over time because of damage. The cross-link theory is one of these because it describes aging changes as results of inappropriate cross-links in protein, DNA, and structural molecules that inhibit the normal functions of the body. Predetermination theories of aging focus on a built-in mechanism that determines when aging and death will occur. Rate of living, pacemaker (or biologic clock), and immune system theory all focus on predetermined programming that leads to aging. - -

Which of the following theories of aging is an accumulated damage theory? a. Rate of living theory b. Pacemaker theory c. Immune system theory d. Cross-link theory

c. Supplements are used more frequently by people who read food labels. (CH 12 Krause, pg 195) Dietary supplement use is more frequent among people of better health, including those with BMIs below 25 kg/m2, nonsmokers, those who are physically active, and those who use food labels in making healthy eating decisions. Dietary supplement use increases with advancing age, white race, and women. -

Which of the following trends regarding use of dietary supplements is TRUE? a. Supplements are used more frequently by younger as opposed to older adults. b. Supplements are used more frequently by minorities than whites. c. Supplements are used more frequently by people who read food labels. d. Supplements are used more frequently by people with BMIs above 25 kg/m2.

d. Structure-function claims (CH 12 Krause, pg 196) Structure-function claims were developed under DSHEA to provide for label statements that address physiologic effects of product content in supplements. This was a compromise with the supplement industry so that the supplement producers did not have to provide documented evidence to support health claims. The problem with these claims is that consumers cannot distinguish between these and health claims. Authoritative statements were established under the FDA Modernization Act of 1997 to allow for health claims based on published statements from authoritative organizations. Both health and nutrition claims were established under the Nutrition Labeling and Education Act of 1990. -

Which of the following types of claims was first established under DSHEA? a. Authoritative statements b. Health claims c. Nutrition claims d. Structure-function claims

c. Replacing full calorie drinks with zero-calorie substitutes (CH 21 Krause, pg. 393) Artificial sweeteners and fat substitutes improve the acceptability of limited food intakes for some people (see Chapters 3 and 30). However, there is no evidence that using artificial sweeteners reduces food intake or results in weight loss, and they are not recommended by the NWCR. - -

Which of the following was NOT recommended by the National Weight Control Registry (NWCR) as a method to maintain weight? a. Eating a relatively low-fat (24%) diet b. Eating breakfast almost every day c. Replacing full calorie drinks with zero-calorie substitutes d. Engaging in high levels (60-90 min/day) of physical activity

a. Decreased serum hemoglobin (CH 15 Krause, pg 243) As the blood volume expands by 50% during pregnancy, this results in a dilution of blood constituents, such as serum hemoglobin, albumin, and other blood proteins. This also promotes an increase in glomerular filtration rate; however, the kidneys do not increase the volume of urine excretion. Proteinuria occurs when damage occurs to the basement membrane of the glomerulus. Some glucosuria may occur because of decreased efficiency in renal tubule reabsorption during pregnancy but not to the level of promoting hypoglycemia. Constipation is more likely to occur when inadequate water is taken in by the pregnant woman. -

Which of the following will result from the normal physiologic adaptation during pregnancy? a. Decreased serum hemoglobin b. Proteinuria c. Hypoglycemia d. Constipation

c. Fresh ground turkey (CH 11 Krause, pg 184) The Nutrition Labeling and Education Act excludes foods prepared on site such as bakery and deli items and raw foods such as meat and produce. -

Which of the following would NOT be required to have a nutrition label? a. Canned corn b. Hot dogs c. Fresh ground turkey d. American cheese

d. Excessive hepatic gluconeogenesis (CH 30 Krause, p. 615) Postprandial or reactive hypoglycemia occurs within 2 to 5 hours after eating. During this period of time, the body absorbs glucose and responds to increased blood glucose levels by secreting insulin. However, during this same time, as glucose is being provided by the diet, the liver is not stimulated to produce additional glucose through gluconeogenesis. A defect in glucagon response could contribute to the hypoglycemia by not triggering gluconeogenesis. - -

Which of the following would NOT result in postprandial (reactive) hypoglycemia? a. Rapid glucose absorption b. Excessive insulin secretion c. Insufficient glucagon secretion d. Excessive hepatic gluconeogenesis

ANS: B Krause CH 7 A CBC panel focuses on the analysis and descriptions of the red blood cells, including hemoglobin, hematocrit, and mean cell volume (the size of the red blood cells). Glucose, total cholesterol, and albumin are values on a common serum chemistry or blood panel. - -

Which of the following would be included in a complete blood count? a. Total cholesterol b. Mean cell volume c. Glucose d. Albumin

c. Large meals, fatty foods, and alcohol (CH 29 Krause, pp. 582-583) For both chronic and acute pancreatitis, the introduction of food should be done in a manner to minimize pain, nausea, and vomiting. Smaller meals with minimal fat content are better tolerated and less likely to promote symptoms. Alcohol is avoided because it is commonly associated with the development and progression of pancreatitis. - -

Which of the following, when consumed by a patient with chronic pancreatitis, may cause symptoms to worsen? a. Fried foods, foods with a high sucrose content, and caffeine b. Carbohydrates, low-fat dairy products, and coffee c. Large meals, fatty foods, and alcohol d. Red meat, sweets, and coffee

b. High-fat foods (CH 29 Krause, p. 577) A diet limited in fat is used when a patient has cholecystitis because fat intake stimulates the contraction of the gallbladder. When this occurs, the patient experiences pain. Presence of fat in the bowel stimulates release of cholecystokinin, which triggers the release of bile acids from the gallbladder. - -

Which of the following, when eaten by a patient with cholecystitis, may cause pain? a. High-protein foods b. High-fat foods c. Complex carbohydrates d. Simple carbohydrates

It increases as a result of fetal growth and maternal cardiac output. (CH 2, Krause) pg 18 Hormonal changes that occur during pregnancy support the changes in the maternal body to support the growth of the fetus. These changes include the growth of metabolically active tissue in the uterus, placenta, and fetus. Additionally, blood volume is increased, and cardiac workload increases. Because the metabolic rate is dependent on metabolically active tissues, as these increase and the heart's work increases, the overall metabolic rate increases. -

Which of these best describes the change in the metabolic rate during pregnancy? a. It decreases as a result of a decrease in maternal physical activity. b. It increases as a result of fetal growth. c. It increases as a result of fetal growth and maternal cardiac output. d. It decreases as a result of an increase in maternal adipose tissue.

Emergency Food and Shelter Program (CH 9 Krause pg 146-147) The Emergency Food and Shelter Program is funded through the Federal Emergency Management Agency to aid those in need in response to disaster. This program provides persons in need with funds to acquire food and temporary shelter. The CACFP, CSFP, and TEFAP are all FNS programs through USDA. -

Which of these food programs is NOT funded through the USDA Food and Nutrition Service? a. Child and Adult Care Food Program b. Commodity Supplemental Food Program c. Emergency Food Assistance Program d. Emergency Food and Shelter Program

ANS: C Reverse T3 (rT3) is derived from T4 through the action of deiodinase. It is an isomer of T3 but is not itself produced by the thyroid. REF:p. 619 (Krause CH 31) - -

Which of these is NOT a product of the thyroid? a. Thyroxine (T4) b. Calcitonin c. Reverse T3 (rT3) d. Triiodothyronine (T3)

c. Add peanut butter when eating crackers.(CH 25 Krause, p. 471) Xerostomia is a condition of dry mouth. Usually this results from a lack of or decrease in salivation that can occur from the use of medications or from medical conditions such as uncontrolled diabetes mellitus and several autoimmune conditions. Sticky foods such as peanut butter should be avoided. - -

Which of these is NOT a recommendation for people with xerostomia? a. Drink water with lemon. b. Chew xylitol-containing gum. c. Add peanut butter when eating crackers. d. Eat moist foods.

a. Hispanic ethnicity (CH 24 Krause, p. 459) People of European and Asian ethnicity experience more osteoporotic fractures than Blacks or Hispanics, who usually have a greater bone density. Cigarette smoking is a risk factor, probably because of toxic effects on bone. Young women with the female athlete triad of disordered eating, amenorrhea, and low BMD are at increased risk of having fractures. - -

Which of these is NOT a risk factor for developing osteoporosis? a. Hispanic ethnicity b. European or Asian ethnicity c. Cigarette smoking d. Female athlete triad

a. It is higher in fat than mature milk. (CH 15 Krause, pg 286) Colostrum is the thin, yellow, milky fluid that is the first milk available after birth. It is higher in protein and lower in fat and carbohydrates than mature milk. It is also lower in lactose and higher in immunoglobulins than mature milk. -

Which of these statements is NOT true about colostrum? a. It is higher in fat than mature milk. b. It is higher in immunoglobulins than mature milk. c. It is higher in protein than mature milk. d. It is lower in lactose than mature milk.

b. Peripherally inserted central catheter (CH 13 Krause, pg 218) PICC lines are inserted into a vein in the antecubital area of the arm and threaded into the subclavian vein, allowing greater mobility for the patient and decreased infection risk. Tunneled versions of single- or multiple-lumen catheters into the subclavian or the external jugular vein are also types of long-term access. Peripheral IV catheters and single-lumen, nontunneled catheters placed in the subclavian or internal or external jugular veins are also short-term catheters. -

Which one of these routes of parenteral access is considered long term? a. Peripheral intravenous catheter b. Peripherally inserted central catheter c. Single-lumen catheter inserted into the subclavian vein d. Catheter inserted into the external jugular vein

a. Isothiocyanates (CH 19 Krause, pg 361) Isothiocyanates promote the production of phase II enzymes in the liver biotransformation system. Carotenoids, phytic acid, and phenols all have antioxidant properties that are protective of different tissues and cells throughout the body. -

Which phytochemicals are thought to promote the detoxification systems of the body? a. Isothiocyanates b. Carotenoids c. Phytic acid d. Phenols

b. ODS (CH 12 Krause, pg 197) The Office of Dietary Supplements has prepared information available to assist in educating consumers about various dietary supplements. The National Center for Complementary and Alternative Medicine promotes research into alternative medicine. The Food and Drug Administration has published tips for supplement users in regard to making informed decisions when selecting supplements. The Agency for Healthcare Research and Quality maintains a database of evidence-based practices in treatments of various conditions, and some of these include aspects of CAM. -

Which program has developed fact sheets about dietary supplements that can be used by health professionals to educate clients? a. NCCAM b. ODS c. FDA d. AHRQ

b. Contemplation (CH 14 Krause, pg 233) Ambivalence occurs when the patient has mixed feelings and is considering the "pros" and "cons" of behavior change. This is characteristic of the stage of contemplation. The precontemplation stage is characterized by the need for awareness of a problem existing. Preparation involves the need to identify acceptable change strategies. Maintenance has the challenge of sustaining an accomplished change without occurrence of relapse. -

Which stage of change is characterized by the patient's feelings of ambivalence? a. Precontemplation b. Contemplation c. Preparation d. Maintenance

c. Agreeing with a twist (CH 14 Krause, pg 234) When dealing with clients during the precontemplation stage, the goal is to facilitate their ability to consider change. For this to occur, strong communication skills are necessary. These include asking open-ended questions, listening reflectively, providing affirmation, and summarizing the client's statements. Agreeing with a twist is a means of dealing with resistant behavior by agreeing with the client and then redirecting the focus of the conversation. -

Which technique is NOT useful in working with clients during a not-ready-to-change counseling session? a. Asking open-ended questions b. Affirming c. Agreeing with a twist d. Summarizing

Pharmacokinetics (Ch 8 Krause, page 122) Pharmacokinetics focuses on the time course of a drug through the body. Pharmacodynamics focuses directly on the drug's effect on the body. Pharmacogenomics involves the study of genetic variations that are affected by drugs. Polypharmacy refers to the use of multiple drugs, which could lead to interactions and have additional consequences in regard to health. REF: p. 122 -

Which term is used to describe the movement of a drug through the body by absorption, distribution, metabolism, and excretion? a. Pharmacodynamics b. Pharmacogenomics c. Pharmacokinetics d. Polypharmacy

B- Heart (Beerman, CH 12) - -

Which tissue is especially susceptible to the effects of magnesium deficiency? a. Liver b. Heart c. Kidneys d. Lungs

a. An adherence ruler (CH 14 Krause, pg 235) An adherence ruler or scale used in coordination with the question, "On a scale from 1 to 12, how ready are you right now to ___?" is adequate to establish readiness to change. A food diary and food models can help identify eating habits, but in the case of a food diary, often clients do not have the training necessary to produce one for the first session. Weighing the patient provides a baseline, and repeated weighings can help demonstrate progress, but weight change is not always the desired result of nutrition counseling. -

Which tool can be used to assess a client's readiness to change? a. An adherence ruler b. A food diary c. A weight scale d. Food models

d. Phosphorus (CH 13 Krause, pg 222) When refeeding syndrome occurs, serum levels of phosphorus, potassium, and magnesium fall below normal levels, resulting in the electrolyte imbalances associated with refeeding syndrome. As a patient's stores of these nutrients may be depressed after extended time without eating, infusion of glucose contributes to further decreasing blood levels of these minerals as they are shifted into cells with glucose. Potential cardiac and pulmonary complications can be avoided by supplementing the patient with phosphorus, potassium, and magnesium and by limiting the initial amounts of glucose administered when feeding resumes. -

Which trace element needs to be initially given at higher levels when there is a risk of refeeding syndrome? a. Zinc b. Copper c. Chromium d. Phosphorus

Autosomal recessive diseases (ch 5, krause) Autosomal recessive diseases are the most common as two copies of an abnormal autosomal allele are necessary for the phenotype to be expressed. Both autosomal and X-linked dominant diseases only need one abnormal allele for expression to occur. An X-linked recessive condition could only occur in a woman if she inherits two copies of the recessive trait. Because males only have one X chromosome, if the recessive trait is present, it will be expressed. REF: p. 75 -

Which type of disorders are the most common of the mendelian inherited disorders? a. Autosomal recessive diseases b. Autosomal dominant diseases c. X-linked dominant diseases d. X-linked recessive diseases

c. Traditional Chinese medicine (CH 12 Krause, pg 192) Qi or chi refers to the life-force or energy that is the center of the body functions. Health and wellness are assumed when chi is balanced and in harmonious flow with the body. Illness occurs when there are disturbances in this flow. -

Which type of holistic therapy is based on the belief that Qi is the balancing force of the body? a. Homeopathy b. Naturopathy c. Traditional Chinese medicine d. Phytotherapy

a. Homeopathy (CH 12 Krause, pg 192) Homeopathy helps the body heal itself through the law of similars in which remedies to symptoms are diluted substances that cause the symptoms. Naturopathy focuses on nature as the provider of healing and uses phytotherapy, treatment with plant-based preparations, as one aspect of treatment. Traditional Oriental medicine focuses on establishing balance in regard to life-force energy and uses a variety of techniques in health care. -

Which type of holistic therapy is based on the belief that you treat like with like? a. Homeopathy b. Naturopathy c. Traditional Oriental medicine d. Phytotherapy

c. Glucocorticoid (CH 29 Kruase, p. 575) Use of glucocorticoids has the side effects of sodium retention, hyperglycemia, hyperlipidemia, false hunger, protein wasting, and decreased absorption of calcium and phosphorus. Use of azathioprine can cause macrocytic anemia, mouth sores, nausea, vomiting, diarrhea, anorexia, sore throat, stomach pain, and decreased taste acuity. Cyclosporine can cause sodium retention, hyperkalemia, hyperglycemia, hyperlipidemia, decreased serum magnesium, hypertension, nausea, and vomiting. Tacrolimus promotes hyperglycemia, hyperkalemia, nausea, and vomiting. - -

Which type of immunosuppressant drug used in the treatment of liver transplant is most likely to promote sodium retention, hyperglycemia, and impaired absorption of calcium? a. Azathioprine b. Cyclosporine c. Glucocorticoid d. Tacrolimus

ANS: B Before the 2000 version, the focus of the Dietary Guidelines for Americans was on the prevention of chronic disease; however, because of concerns regarding foodborne illness outbreaks, food safety was included in the 2000 and 2005 versions. Food safety continues to be a concern because many Americans eat away from home, and inappropriate food handling is the means by which food becomes contaminated. REF: p. 145 Ch 9 Krause - -

Which version of the Dietary Guidelines for Americans was the first to include food safety? a. 2005 b. 2000 c. 1980 d. 1977

c. Vitamin D (CH 16 Krause, pg 303) The breastmilk of an adequately fed, lactating woman provides all vitamins except vitamin D. Human milk provides only about 0.5 mcg of vitamin D, which is one-tenth of the amount recommended by the AAP to prevent vitamin D deficiency and rickets. Supplementation with 5 mcg of vitamin D daily is recommended for breast-fed infants. -

Which vitamin is NOT adequately provided by breastmilk? a. Vitamin C b. Niacin c. Vitamin D d. Vitamin A

CDC (CH 9 Krause) The Centers for Disease Control and Prevention (CDC) collects information regarding morbidity and mortality, as do state and local public health agencies. The Census Bureau collects demographic information useful in describing the population base of a community. The CDC and the Administration on Aging are two agencies within the U.S. Department of Health and Human Services. The local Area Office on Aging is able to provide information about the elderly population in the local community. -

While performing a community assessment, you determine that you need information about local cardiovascular morbidity and mortality. Which of the following would be an appropriate resource for this information? a. Census Bureau b. Centers for Disease Control and Prevention c. Department of Health and Human Services d. Area Office on Aging

c. Malabsorption of calcium, zinc, and magnesium (CH 28 Krause, p. 550) The increase in renal oxalate stone formation resulting from ileal resection occurs because of impaired bile salt reabsorption. The ileum is the primary site of bile salt reabsorption, and without it, bile salts are eliminated instead of recycled. This leads to a decrease in bile secretion, which impairs lipid digestion and absorption. The resultant increase in fatty acids in the bowel can combine with divalent cations such as calcium, zinc, and magnesium to form insoluble soaps. Normally, these cations would be free to bind with oxalate to prevent colonic absorption of oxalate. However, with the divalent cations already bound as soaps, the colonic absorption of oxalate increases. - -

Why do renal oxalate stones form as a consequence of ileal resection? a. Inadequate intake of dietary fiber and water b. Malabsorption of fat-soluble vitamins c. Malabsorption of calcium, zinc, and magnesium d. Malabsorption of vitamin B12 and intrinsic factor complex

C- Those who use medications containing magnesium for treatment of heartburn, indigestion, and constipation (Beerman, CH 12) - -

Who is at the greatest risk of developing magnesium toxicity? a. Those who consume "farmed" seafood as opposed to non-farmed seafood b. Those who consume foods grown near electrical power plants c. Those who use medications containing magnesium for treatment of heartburn, indigestion, and constipation d. Those who consume foods grown near nuclear power plants

d. Men in their late 40s (CH 19 Krause, pg 352) According to the Dietary Guidelines for Americans, men older than 45 years of age and women older than 55 years of age receive the most benefit from limited alcohol intake. Among younger adults, the benefit of reducing risk of cardiovascular disease is offset by the increased risk of accidents resulting from alcohol abuse. -

Who may benefit the most from the light to moderate intake of alcohol in reducing the risk of cardiovascular disease? a. Men in their late 30s b. Women in their 20s c. Women in their late 40s d. Men in their late 40s

b. They promote saliva flow. (CH 25 Krause, p. 470) Foods that require chewing stimulate saliva production, which then reduces the cariogenicity of the meal by cleansing the teeth. This leads to a decrease in fermentable carbohydrate and reduced oral acidity. - -

Why are foods that require a lot of chewing effective in promoting good dental health? a. They cleanse the tooth surface. b. They promote saliva flow. c. They lower the acidity level of the mouth. d. They decrease the fermentation of carbohydrate.

c. Their renal concentrating ability is less than that of older children. (CH 16 Krause, pg 301) Under normal conditions, breastmilk and formula provide infants with adequate water. However, if formula is boiled, water evaporation can lead to concentration of formula solids. This places stress on the immature kidneys of the infant. This is also why providing commercially prepared adult foods to an infant could be problematic—the higher sodium concentration of these foods could exceed the infant's renal solute load. Infants should be monitored when they live in hot, humid conditions and when they have episodes of diarrhea or vomiting. Watching for the number of wet diapers is one of the ways to do this. -

Why are infants particularly susceptible to developing dehydration? a. Their mass-to-surface ratio is low. b. Their insulation is poor, resulting in increased water loss. c. Their renal concentrating ability is less than that of older children. d. The increased liquidity of their stools results in greater fluid loss.

d. Decreased motility in the large intestine (CH 20 Krause, pg 371) Constipation involves a reduction of bowel movements, and one potential reason is a decrease in the motility in the large intestine. The longer intestinal contents remain in the colon, the more water is absorbed and the harder the stool is to transit and eliminate. Delayed transit time, medications, and reduced stimulation of the gut through reduced physical activity all contribute to the development of constipation. - -

Why does constipation occur in elderly adults? a. Increased motility in the small intestine b. Decreased motility in the small intestine c. Increased motility in the large intestine d. Decreased motility in the large intestine

d. The infant has no immunity to the botulism spore that may be present. (CH 16 Krause, pg 302) Honey may be a carrier of Clostridium botulinum spores, which can reproduce and make toxins that can cause death. The spores are resistant to heat treatment and are not destroyed by current processing methods. Infants are born with a preferential taste for sweets; therefore, providing sweet foods will not necessarily exert a craving for more. The simple carbohydrates in honey are digestible by infants. Only water, breastmilk, and rice cereal should be mixed with infant formula—nothing else. -

Why should honey NOT be used in home-prepared formulas for infants? a. The added sweetener will cause the infant to crave sweets. b. The sugars present cannot be digested by the infant. c. The honey will settle out in the formula and cause a curdled appearance. d. The infant has no immunity to the botulism spore that may be present.

d. It is associated with lower intakes of iron and linoleic acid. (CH 16 Krause, pg 303) Cow's milk is a poor source of both iron and linoleic acid. Protein and other nutrients in cow's milk increase the renal solute load, placing stress on the infant's kidneys. Also, the protein in cow's milk may cause gastrointestinal bleeding in the infant, promoting an increased need for iron. Use of whole milk for feeding young children has not been demonstrated to promote obesity. -

Why should whole cow's milk NOT be given to infants younger than 1 year of age? a. It decreases the renal solute load. b. It can lead to obesity in toddlers. c. It lacks the anti-infective quality of formulas. d. It is associated with lower intakes of iron and linoleic acid.

a. Impaired thermoregulation (CH 23 Kruase, p. 437) Adequate fluid replacement is necessary to maintain body temperature. When fluid is not replaced, the body's core temperature can increase, promoting heat stress. Replacement of electrolytes should accompany fluid replacement because sodium and potassium are both lost through sweat. Calcium balance is maintained through adequate intake of calcium from food sources. Cerebral edema and bradycardia occur in situations of fluid overload. With decreases in body water, the kidneys conserve fluid as opposed to promoting elimination. Osmotic diarrhea may occur if highly osmotic fluids, such as apple juice, are used for fluid replacement. - -

________ can be the result of inappropriately low fluid replacement after athletic competition. a. Impaired thermoregulation b. Decreased calcium levels and fatigue c. Cerebral edema and bradycardia d. Diuresis and osmotic diarrhea

Cytosine-guanine (ch 5, Krause) ANS: D In DNA, only two potential base pairings are possible: adenine with thymine and cytosine with guanine. In RNA, uracil may take the place of thymine to be paired with adenine. -

Within a strand of DNA, which of the following nucleotide base pairing is possible? a. Adenine-guanine b. Cytosine-thymine c. Adenine-uracil d. Cytosine-guanine

a. Within 2 hours (CH 26 Krause, p. 486) Anaphylactic reactions to food usually occur within the first 2 hours after exposure. These are usually IgE-mediated allergies. Chronic and relapsing forms of food allergies may take more than 2 hours because they tend to be either a mix of IgE- and cell-mediated reactions or nonIgE- and cell-mediated reactions. A food-dependent, exercise-induced anaphylaxis is a particular type of physical allergy that occurs within 2 hours of rigorous activity after eating a specific food that is normally tolerated. - -

Within what frame of time is the onset of symptoms related to a food allergy most likely to occur? a. Within 2 hours b. Within 2 to 14 hours c. More than 24 hours d. More than 48 hours

b. Conformational epitopes (CH 26 Kruase, p. 480) Recent studies suggest that 70% to 80% of young children allergic to milk or eggs can tolerate baked (heat-denatured) forms of the protein but not the unbaked form. It is suggested that these children make IgE antibodies primarily to conformational epitopes (antigenic determinants on the surface of the food proteins that are recognized by the immune system). Haptens are small molecules that can elicit an immune response only when attached to a large protein carrier. Mast cells release histamines. Cytokines are proteins released by cells and serve as communicators with the immune system. - -

_________ are antigenic determinants on the surface of food proteins that are recognized by the immune system. a. Cytokines b. Conformational epitopes c. Mast cells d. Haptens

b. Loss of thirst sensation (CH 30 Krause, p. 588) The common symptoms of type 1 diabetes mellitus include hyperglycemia, polyuria, polydipsia, weight loss, dehydration, electrolyte disturbance, and ketoacidosis. Type 1 DM tends to increase the thirst sensation as opposed to blocking it. - -

__________ is NOT a symptom of type 1 DM. a. Hyperglycemia b. Loss of thirst sensation c. Weight loss d. Polydipsia

b. Glutamine (CH 28 Krause, p. 551) Glutamine is the preferred fuel for small intestinal enterocytes and thus may be valuable in enhancing adaptation. Short-chain fatty acids (e.g., butyrate) produced from microbial fermentation of carbohydrate and fibers are the major fuels of colonic epithelium. - -

__________ is the preferred fuel for the small intestinal enterocytes and is considered important to adaption after bowel resection. a. Glucose b. Glutamine c. MCT d. Butyrate

b. Headache, flushing, abdominal pain, and nausea (CH 26 Kruase, p 488) Adverse reactions to MSG include headache, nausea, flushing, abdominal pain, and asthma occurring several hours after ingestion. Symptoms related to nonimmunologic reactions to other food additives include hives, rash, and asthma in response to FD&C yellow #5, benzoic acid, and sodium benzoate and acute asthma and anaphylaxis or loss of consciousness in response to various sulfite additives - -

hat symptoms are likely to occur in the person with a perceived intolerance to monosodium glutamate (MSG)? a. Decreased heart rate, constipation, and hives b. Headache, flushing, abdominal pain, and nausea c. Steatorrhea, edema, proteinuria, and nausea d. Anaphylaxis, toxic shock syndrome, and headache

c. maintain water balance in the body as a whole and within cells. (CH 23 Krause, p. 430) Depending on the type of physical activity performed, athletes need to have adequate water intake to prevent dehydration. Fluid loss through sweating is affected by the length and intensity of activity as well as by the environment. Sweating provides a cooling mechanism for the body; however, if fluid is not replaced, dehydration will impair the body's thermoregulation and the athlete's ability to perform. - -

thletes need to increase their water intake to a. enhance renal excretion of waste products from muscle activity. b. improve intestinal absorption of nutrients after digestion. c. maintain water balance in the body as a whole and within cells. d. increase skin evaporation of water as a cooling mechanism.

Lipitor (CH 8 Krause pg 129) Statin drugs, such as Lipitor, are also known as HMG-CoA reductase inhibitors. They affect the formation of CoQ10 and the result is decreased circulating levels. Methotrexate is a folic acid antagonist. Cholestyramine absorbs fat-soluble vitamins, and carbamazepine affects the metabolism of biotin, vitamin D, and folic acid. -

upplementation of coenzyme Q10 (CoQ10) should be considered in a patient taking which drug? a. Carbamazepine b. Methotrexate c. Lipitor d. Cholestyramine


Conjuntos de estudio relacionados

BI218 Chapter 11 Rocketmix Questions

View Set

Skin Integrity and Wound Care - Basics

View Set

Midterm Material: Language Disorders in Children

View Set